Skip to main content

Full text of "High school mathematics"

See other formats


L  I  B  RAR.Y 

OF  THE 

U  N  I  VERS  ITY 

or    1LL1  NOIS 

<V  510.7 
\S59 


Digitized  by  the  Internet  Archive 

in  2012  with  funding  from 

University  of  Illinois  Urbana-Champaign 


http://www.archive.org/details/highschoolmathem07auniv 


HIGH  SCHOOL  MATHEMATICS 


Unit  7. 

MATHEMATICAL  INDUCTION 


UNIVERSITY  OF  ILLINOIS  COMMITTEE  ON  SCHOOL  MATHEMATICS 

MAX  BEBERMAN,  Director 
HERBERT  E.  VAUGHAN,  Editor 


UNIVERSITY  OF  ILLINOIS  PRESS      ■      URBANA,  1961 


HIGH  SCHOOL  MATHEMATICS 


Unit  7. 

MATHEMATICAL  INDUCTION 


UNIVERSITY  OF  ILLINOIS  COMMITTEE  ON  SCHOOL  MATHEMATICS 

MAX  BEBERMAN,  Director 
HERBERT  E.  VAUGHAN,  Ed/for 


UNIVERSITY  OF  ILLINOIS  PRESS      •      URBANA,  1961 


©  1961  by  the  Board  of  Trustees  of  the  University  of  Illinois. 
Manufactured  in  the  United  States  of  America. 


[7-iii] 


PREFACE 

In  Unit  2  you  formulated  some  of  your  knowledge  about  the  real 
numbers  in  ten  basic  principles.     [These  principles  are  repeated  on 
page  7-17  of  the  present  unit.]    You  found,    as  you  derived  theorems 
from  these  basic  principles,  that  the  latter  contained,  at  least  implicitly, 
most  of  what  you  knew  about  the  real  numbers  and,  perhaps,  some  things 
you  hadn't  known.     In  this  unit  you  will,    for  one  thing,    discover  a  few 
additional  basic  principles  which  you  can  use,    as  in  Unit  2,    to  organize 
more  of  your  knowledge  about  real  numbers  and  to  learn  more  about  them. 
For  example,    you  will  discover  simple  basic  principles  which  you  can 
use  to  derive  the  transformation  principles  for  inequations  [see  page  7-36] 
that  you  learned  to  use  in  Unit  3.     Knowledge  of  these  new  basic  principles 
will  also  help  you  to  discover  new  things  about  inequations,    and,    in  par- 
ticular,   about  the  positive  numbers. 

The  fact  just  mentioned- -that  some  of  the  new  basic  principles  will 
imply  new  theorems  about  positive  real  numbers- -should  remind  you 
that  the  basic  principles  and  theorems  of  Unit  2  formulate  only  properties 
which  hold  of  all  real  numbers  [or,    in  the  case  of  division,  of  all  nonzero 
real  numbers].      Special  kinds  of  real  numbers --for  two  examples,   the 
positive  numbers  and  the  positive  integers  - -have  special  properties 
which  do  not  hold  of  all  real  numbers.      For  instance,    although  it  is  the 
case  that  between  any  two  real  numbers  there  is  a  third  real  number, 
there  is  no  positive  integer  between,    say,   the  positive  integers  5  and  6. 
Among  the  new  basic  principles  which  you  will  discover  in  this  unit  there 
are  some  which  will  take  account  of  the  basic  differences  between  the 
set  of  all  real  numbers  and  the  set,    I+,    of  positive  integers.     Using  these 
you  will  be  able  to  discover  interesting  properties  peculiar  to  the  positive 
integers.     In  particular,    you  will  learn  a  special  method--proof  by  mathe- 
matical induction- -for  proving  generalizations  about  positive  integers. 
You  will  find  this  method  of  proof  very  useful,    not  only  here,    but  in  all 
your  later  work  in  mathematics.     [Incidentally,    you  will  find  many 
opportunities  in  this,  as  well  as  in  later  units,  to  practice  the  techniques 
you  learned  in  Unit  6  for  writing  column  proofs  and  paragraph  proofs.] 

Just  as  the  theorems  you  proved  in  Units  2  and  6  justified  techniques 
which  were  helpful  in  applying  your  knowledge  of  real  numbers  and 
geometric  figures  to  the  solutions  of  various  sorts  of  problems,    so  the 
theorems  you  learn  in  this  unit  will  help  in  solving  other  kinds  of  prob- 
lems.    However,    as  you  should  have  learned  by  now,    mere  knowledge 
of  the  theorem  which  justifies  a  technique  does  not  guarantee  that  one  is 
adept  at  using  the  technique.     Such  techniques --simplification  of  algebraic 
expressions  is  an  example- -must  be  practiced,    both  by  themselves  and 
as  steps  in  the  solutions  of  problems,    if  one  is  to  learn  to  make  good 
use  of  them.     The  Miscellaneous  Exercises  throughout  this  unit  and  the 
Review  Exercises  at  the  end  furnish  opportunity  for  the  practice  of  both 
old  and  new  techniques. 


[7-v] 


TABLE  OF  CONTENTS 


Preface 


[7-iii] 


Introduction 


Some  problems 


[7-1] 
[7-1] 


7.01    The  real  numbers  [7-3] 

Addition  [7-3] 

The  addition  table  [7-4] 

Algorithm  [7-4] 

A  proof  of  the  theorem  '2+2=4*  [7-6] 

Multiplication  [7-7] 

The  multiplication  table  [7-7] 

A  proof  of  the  theorem  42»  4  =  8'  [7-9] 

Miscellaneous  Exercises  [7-10] 

Division-with-remainder  algorithm  [7-12] 
Using  the  division-with-remainder  algorithm  to  find 

the  value  of  a  function  for  a  given  argument  [7-14] 

Other  computing  facts  [7-16] 
The  10  basic  principles  for  real  numbers  used 

in  Unit  2  [7-17] 

The  eleventh  basic  principle:    1^0  [7-18] 
The  notations  '=^>*  for  conditionals  and  '<==>,  for 

biconditionais  [7-19] 

Miscellaneous  Exercises  [7-20] 


7.02    The  positive  numbers 

Basic  principles  (Px)  and  (P2)  for  the  set  of  positive 

numbers 
Closure  principle,    (P3)>   for  addition  of  positive 

numbers 
Closure  principle,    (P4),   for  multiplication  of 

positive  numbers 


[7-22] 
[7-22] 
[7-23] 
[7-24] 


[7-vi]  [CONTENTS] 


A  principle,    (N),   which  defines  the  negative  numbers 

Miscellaneous  Exercises 

Exploration  Exercises -- 

Investigating  the  relation  {(x,  y) :   y  -  x  €  P} 

7.  03    Inequations 

Basic  principle  (G)  for  the  relation   > 

Inequality  theorems 

Two  proofs  of  the  theorem: 

VVV    [x  +  z  >  y  +  z  =>  x  >  y] 
x    y    z  l  3  7J 

The  inference  scheme:    — —M- +* 

q=5>  not  p 

The  transformation  principles  for  inequations 

Addition;   multiplication;   factoring 

Proving  part  of  the  factoring  transformation  principle 

Inequality  theorems  on  the  squaring  function 

A  geometric  application  of  'VXVV  x2  +  y2  >  2xy* 

The  reciprocating  function 

Inequality  theorems  on  the  reciprocating  function 

Monotonicity  of  functions 

Graphing  inequations  on  a  number  line  picture 

Systems  of  linear  equations  in  two  variables 

Miscellaneous  Exercises 

Exploration  Exercises -- 

Discovering  whether  a  given  set  is  an  inductive  set 
Proofs  that  2,    3,   and  4  belong  to  I* 

7.04    The  positive  integers 

Assumptions,   used  in  Unit  4,   about  positive  integers 
Defining  the  set  of  all  positive  integers 
Basic  principles,   I-+,  l*t  I*  tor  the  positive  integer 
Exercises  on  inductive  sets 
Proofs  by  mathematical  induction 


7-25] 
7-27] 

7-29] 
7-29] 

7-30] 
7-30] 
7-32] 

7-34] 

7-35] 

7-36] 
7-36] 
7-37] 
7-38] 
7-39] 
7-39] 
7-40] 
7-41] 
7-41] 
7-42] 

7-43] 

7-45] 
7-45] 
7-46] 

7-47] 
7-47] 
7-48] 
7-49] 
7-50] 
7-51] 


[CONTENTS]  [7-vii] 

A  proof  of  'V    3  +  n  €  I+'  by  mathematical  induction  [7-51] 
r                    n 

The  principle  of  mathematical  induction  [PMI]  [7-53] 
Inductive  proof:    initial  step,   inductive  step 

(using  the  inductive  hypothesis),   final  step  [7-53] 

A  short  cut  in  writing  inductive  proofs  [7-54] 

Closure  of  I+  with  respect  to  +  and  X  [7-56] 

Two  methods  for  proving  generalizations  [7-56] 

Combining  both  methods  to  prove  * Vm  Vn  m  +  n  e  I+*  [7-57] 

Miscellaneous  Exercises  [7-59] 

Reflection  of  a  point  with  respect  to  a  point  [7-60] 

Recursive  definitions  [7-61] 

Computing  triangular  numbers  [7-61] 

Recursion--recursive  definition  [7-62] 

The  function  T  [7-62] 

Explicit  definition  [7-62] 

Square  numbers --S  [7-62] 

Finding  an  explicit  definition  for  T  [7-63] 

Proofs  of  'Vn       Tn  =  SjE  +  iT 

Column  [7-64] 

Paragraph  [7-65] 

Tree-diagram  [7-65] 

Even  and  odd  positive  integers  [7-66] 

Finding  that  S  is  a  function  of  T  [7-66] 

Practice  with  recursive  definitions  [7-68] 

Compound  interest  formulas  [7-68] 

Graphing  recursively  defined  functions  [7-69] 

Oblong  numbers  [7-69] 

Pentagonal  numbers --polygonal  numbers  [7-70] 

A  pattern  for  recursive  definitions  of  polygonal 

number  functions --the  notation:    P  [7-70] 

n  *•  J 

The  number  of  p-membered  subsets  of  an  n-membered 

set--the  notations:    C(n,  p)  and  :       C  [7-71] 

x       tr'  n    p  l  J 

Finding  a  recursive  procedure  for  computing  values 

of  lC(n,  p)'  [7-72] 

Applications  [7-74] 


[7-viii] 


[CONTENTS] 


"The  arithmetic  of  the  positive  integers  [7-76] 
Recursive  definitions,  I4+,   and  I5+,   for  addition  and 

multiplication,    respectively,    of  positive  integers  [7-76] 

Derivation  of  the  apa  for  positive  integers  from  I4*  [7-78] 

Derivation  of  the  cpa  for  positive  integers  from  I4  [7-79] 

Miscellaneous  Exercises  [7-80] 

7.05  The  relation  greater  than  for  the  positive  integers  [7-84] 

For  each  positive  integer  there  is  a  next  one: 

V     V    [n>m=>n>m+l]  [7-84] 

m    n  L                          ~              J  L          J 

Lower  bounds  and  least  members  [7-87] 

Definitions  of  4a  lower  bound'  and  of  *a  least  member'  [7-87] 

The  least  number  theorem  [for  I+]  [7-88] 

The  cofinality  principle  [7-89] 

Exercises  on  bounds  [7-90] 

Misuses  of  induction  [7-90] 

Miscellaneous  Exercises  [7-91] 

7.06  The  integers  [7-94] 

A  definition  (I)  for  the  set  of  integers  [7-94] 
Closure  theorems  for  integers  :    opposition,   addition, 

subtraction,   and  multiplication  [7-95] 

Exploration  Exercises--  [7-96] 
Investigating  the  translating  function,  ti,   which  maps 

each  integer  k  onto  k  -  j  [7-96] 
Investigating  the  reflecting  function,    r:,  which  maps 

each  integer  onto  its  opposite  [7-97] 

The  least  number  theorem  for  integers  [7-98] 

Induction  over  {k:    k>j}  [7-100] 

Induction  over  the  set  of  all  integers  [7-100] 

Miscellaneous  Exercises  [7-101] 

The  greatest  integer  function  [7-102] 

The  notation  '[  .  .  .  ]  *  for  'the  integral  part  of  .  .  .  *  [7-102] 


[CONTENTS)  [7-ix] 

{(x,  y) :    y  =  [xj}  is  called  4the  greatest  integer 

function'  [7-104] 

A  proof  of  ,VxVk  [k  =  [x]  <=>  k  <  x  <  k  +  1]'  [7-104] 

Division -with-  remainder  [7-106] 

The  least  integer  function  [7-106] 
The  notation  '{{...  J'  for  'the  fractional  part  of  .  .  .'  [7-107] 
{(x,  y) :  y  =  {[  x  }}  is  called  'the  fractional  part 

function'  [7-107] 
^Discovering  a  technique  for  "splitting'*  each  positive 
integer  into  multiples  of  powers  of  a  given 

integer  >    1  [7-107] 

Partial  quotient  and  remainder  [7-111] 

Miscellaneous  Exercises  [7-112] 

Divisibility  [7-115] 
A  theorem  stating  that   j  is  transitive,   and  one  stating 

that   |  is  antisymmetric  [7-115] 

The  divisibility  relation  [7-116] 

Similarities  and  differences  between  J  and  <  [7-116] 
Examining  the  way  in  which   |   orders  the  positive 

integers  [7-117] 

Prime  numbers  [7-118] 

"^Highest  common  factor  [7-119] 

The  symbols  'HCF'  and  'GCD'  [7-120] 

Integral  linear  combinations  [7-121] 
Euclid's  algorithm  for  finding  greatest  common 

divisors  [7-123] 

Exploration  Exercises--  [7-125] 
Discovering  whether  a  graph  of  a  linear  equation 

contains  a  point  with  integral  coordinates  [7-125] 
Finding  pairs  of  integers  which  satisfy  a  linear 

equation  [7-125] 

Finding  all  integral  solutions  of  '6x  +  lOy  =  0'  [7-125] 

Diophantine  problems  [7-127] 
Finding  all  the  integral  solutions  of  a  linear 

equation  in  two  variables  [7-128] 


[7-x]  [CONTENTS] 

Relatively  prime  integers  [7-129] 

Solving  linear  Diophantine  equations  [7-130] 

Review  Exercises  [7-133] 

Basic  Principles  and  Theorems  [7-145] 

Basic  principles  for  the  real  numbers  [7-145] 

Theorems  from  Unit  2  [7-145] 

Basic  principles  for  P--basic  principle  for  >  [7-149] 

Theorems  about  >  [7-149] 

Basic  principles  for  I+  [7-151] 

Theorems  about  positive  integers  [7-151] 

Cofinality  principle  and  basic  principle  for  I                           [7-152] 

Theorems  about  integers  [7-152] 
Definitions  of  the  greatest  integer  function  and  the 

fractional  part  function  [7-153] 

Theorems  involving  *|[ .  . .  J'  and  '|  .  . .  }'                                 [7-153] 

Definition  of  the  divisibility  relation  [7-153] 

Theorems  about  |  [7-153] 

Table  of  Trigonometric  Ratios  [7-155] 

Table  of  Squares  and  Square  Roots  [7-156] 


[Introduction]  [7-1] 

Some  problems. --Try  to  solve  these  problems: 

I.  Al  Moore  is  trying  to  decide  between  two  job  offers.     Each  position 
starts  at  the  rate  of  $4000  per  year.     The  first  offer  promises  a 
salary  increase  of  $100  each  six  months.     The  second  assures  him 
of  an  annual  increase  of  $400.     If  he  plans  on  staying  with  one 
position  for  five  years,  which  one  will  pay  him  more  money? 

II.  Stan  Brown  also  has  a  choice  between  two  jobs.     Each  job  is  to  last 
for  30  days.     The  first  job  pays  $100  per  day.    The  second  job  starts 
with  1  cent  the  first  day,    2  cents  the  second  day,   4  cents  the  third 
day,   etc.  doubling  each  day.     Which  job  pays  more?    By  how  much? 

III.  Study  the  following  sentences  and  complete  the  last  two. 

Vl  +  3  =  2 

Vl  +  3  +  5  =  3 


Vl  +  3  +  5  +  7  =  4 

Vl  +  3  +  5  +  7  +  9  =  5 

•       •       • 

V1  +  3  +  5  +  ...  +87    = 
V1  +  3  +  5  +  ...  +999  = 


IV.    Study  these  sentences  and  complete  the  last  one. 

2<-L+J-+JL+_L<3  4<JL+JL  +  ..#  +  -L« 


<  7  8  < 


18  <  —  +  —  +  ...   +  —i—  <   19 


VT 

Vz 

V3 

VT 

JL  + 
VT 

vr 

•    •   * 

+  -2_ 
VTe 

-L  + 

VT 

-L  + 

vr 

•   •   • 

+  » 
Vioo 

VT 

VI 

V? 

-L  + 

VT 

-L  +  ... 

V2 

V25 

< 

9 

-L  + 

VT 

•n 

+       i 

■fioi 

< 

Use  the  facts  that  VI  =  1.414,    V3  =  1.7  32,   and  VT  =  2.236  to  check 
that 

2/5  -2<   -i.+  J__+J_+JL+J-.<  zJl  .  1 

VT      VI      V3      V4      VT 


[7-2] 


[Introduction] 


V.  You  can  easily  check  by  computing  that  (1.0 1)2  >    1.02,   ( 1.  0  I)3  >  1 .  03, 
and  that  (1.0  l)4  >    1.04.     Do  so.     Do  you  think  that  ( 1.  0  l)100  >  2? 
That  (1.05)100  >  6? 

VI.  You  can  probably  expand  '(x  +  y)2'  without  much  thinking: 

(x  +  y)2  =  x2  +  2xy  +  y2 
Other  equations  like  this  are  below.     Complete  the  last  two. 

(x  +  y)3  =  x3  +  3x2y  +  3xy2  +  y3 

(x  +  y)4  =  x4  +  4x3y  +  6x2y2  +  4xy3  +  y4 

(x  +  y)5  = 

(x  +  y)6   = 

VII.  Whatever  route  Milton  takes  in  going  from  his  home  to  Zabranch- 
burg  High  School  he  has  to  walk  at  least  nine  blocks.     How  many 
routes  are  there  which  are  just  nine  blocks  long? 


r 

—     — 

i 
1 

L 

r 

__  _ 

i 
.j 

i 
1 

r 

_  — 

j 

1 

| 

l 

-\ 

i 
i 

! 

J 

l/ 


HOME 


Z.H.S. 


You  could  certainly  solve  each  of  the  foregoing  problems  by  doing 
enough  computation.     But,   as  you  will  learn  in  this  unit  and  the  next 
one,  there  are  easier  ways. 

Also,   each  problem  suggests  a  generalization  which  you  couldn't 
prove  by  any  amount  of  computation.     In  fact,  the  proofs  depend  upon  ad- 
ditional basic  principles  for  real  numbers  which  you  will  study  in  this 
unit. 


[7.01]  [7-3] 

7.01    The  real  numbers.  --If  you  were  to  write  a  proof  of  a  theorem  that 
tells  you  that  the  expanded  form  of  4(x  +  2)2'  is  *x2  +  4x  +  4',   you  would 
find  yourself  making  use  of  basic  principles,   previously  proved  theorems, 
and  computing  facts  such  as  that  2  +  2  =  4  and  that  2*2  =  4.    The  theorems, 
of  course,  are  consequences  of  the  basic  principles,   but  where  do  the 
computing  facts  come  from?     [There  is  a  list  of  theorems  on  real  num- 
bers starting  on  page  7-  145.    The  first  78  were  proved  in  Unit  2.] 

ADDITION 

Statements  of  computing  facts  of  the  kind  just  referred  to  are  conse- 
quences of  the  basic  principles  together  with  some  obvious  definitions. 
What  you  need  to  know  is  that  the  numeral  '2*  is  merely  an  abbreviation 
for  *1  +  T,    43*  is  an  abbreviation  for  '2  +  1',    '4'  for  43  +  l\  .  . . ,   and 
4 10'  is  an  abbreviation  for  '9  +  1*.     And,   as  you  have  known  since  you 
first  studied  place  value,    '347',   for  example,  is  an  abbreviation  for 
*3«  102  +  4«  10  +  7\ 

Now,   let's  prove  the  theorem  '2  +  2  =  4'. 

2  +  2   =  2  +  (l+l)  [definition] 

=  (2  +  1)  +  1  [apa] 

=  3+1  [definition] 

=  4  [definition] 

In  the  same  way,   you  can  prove,   in  succession,    *2  +  3  =  5*, 
42  +  4  =  6',    . .  . ,    and  42  +  8  =  10'.      Do  so.     If  you  try,   in  this  way,  to 
prove  '2  +  9  =  11',   you  need  an  extra  step,   and  a  definition  relating  to 
place  value: 

2  +  9   =    2  +  (8+1)    =   (2  +  8)  +  1    =    10+1    =    1-10+1    =    11 
A  A  A  A  A 

definition  apa  theorem     theorem        definition 

Which  step  is  justified  by  a  place  value  definition? 


[7-4]  [7.01] 

In  this  same  step-by-step  manner  you  could  establish  any  computing 
fact  concerning  sums  of  positive  integers.     For  example,    to  prove 
*54  +  368  =  422'    you  would  prove  a  sequence  of  theorems.     First,    you 
would  prove  '54  +  1  =  55'.     Once  you  had  proved  this  theorem,    you  would 
use  it  in  proving  454  +  2  =  56'.       Continuing  in  this  way,    you  would 
eventually  have  proved  *54  +  367  =  421',    and  would  use  this  theorem  in 
proving  454  +  368  =  422'.      Each  of  your  proofs  would  make  use  only  of 
definitions  [including  place  value],    of  the  apa,    and  [except  for  the  first 
proof]  of  the  preceding  theorem  in  the  sequence,      [instead  of  proving 
368  theorems,    you  could  get  by  with  proving  54  theorems  if  you  used  an 
additional  one  of  our  basic  principles  for  real  numbers.     Which  one?] 

Back  in  grade  school  you  learned  an  easier  way  to  establish  this  kind 

of  fact  of  arithmetic --that  is,    to  add  positive  integers.     You  first  learned 

the  sums  of  all  pairs  of  the  numbers  0,    1,    2,    3,    4,    5,    6,    7,    8,    and  9~~ 

that  is,    you  learned  the  addition  table.       Then,    you  learned  how  to  use 

this  in  adding  larger  numbers.     For  example,    to  add  675  and  237,    you 

placed  the  second  numeral  under  the  first  and,    after  adding  corresponding 

digit  numbers  and  "carrying'',    you  ended  up  with  something  like: 

/  i 

675 
237 

912 

This  procedure  for  finding  the  sum  is  an  example  of  an  algorithm.    Al- 
though we  shall  not  do  so  here,    the  steps  you  take  in  using  this  algorithm 
can  be  justified  on  the  basis  of  place  value  definitions,    the  addition  table, 
the  cpa,   the  apa,    and  the  dpma. 

There  is  a  similar  algorithm  for  simplifying  certain  types  of  alge- 
braic expressions.     Study  the  following  examples  of  its  use. 

Example  1.      Simplify:     (3x3  +  2x  -  7)  +  (2x3  +  x  +  5x2  -  9) 

Solution.     3x  +  2x  -  7 

2x3  +  5x2  +     x  -  9 


5x3  +  5x2  +  3x  -  16 
[Explain  how  the  principle  for  subtraction  is  used  here.] 


[7.01] 


[7-5] 


Example  2.      Simplify:    (a  +  3b  -  c)  +  (a  - 


Solution. 


5a  +   2b 


2c)  -  (b  -  3a  -  3c) 

v ^ ; 


Explain. 


EXERCISES 


A.      Simplify. 


1. 

3. 

5. 

6. 

7. 

8. 

9. 
10. 
11. 
12. 
13. 
14. 
15. 
16. 
17. 
18. 
19. 
20. 


3a  +  4b)  +  (7a  +  9b)  2.     (5x  -  6y)  +  (7x  +  3y) 

-6t  +  s)  +  (-4t  -  5s)  4.     (-9c  +  7d)  +  (3c  -  7d) 

8x  -  3y)  +  (-2x  +  3y)  +  (-6x  +  5y) 

y  +  7z)  +  (5y  -  z)  +  (-9y  -  3z) 

5a  -  6b  +  8c)  +  (3a  -  4b  -  3c) 

8x2  -  3x  +  7)  +  (5x2  +  9x  -  12) 

7y2  -  3  +  6y)  +  (2  -  5y2  +  8y) 

Ilk  +  7m  -  3n)  +  (8m  -  2k  +  4n) 

5t2  -  3t  +  7)  -  (2t2  +  8t  -  5) 

3x  -  5)  +  (-2x  +  3)  +  (2  -  x) 

3a  -  7b)  +  (-2a  +  5b)  +  (-a  +  8b)  +  (4a  -  6b) 

3x2  +  7  -  5x)  -  (9  +  4x  -  5x2) 

8t3  -  3t  +  7)  +  (9t2  -  5t3  +  t)  +  (3t  -  5t2  +  9) 

lis4  -  3s2  +  9)  +  (5s2  -  4s4  +  11)  -  (3s4  -  7  -  2s2) 

3a2  -  2ab  +  3b2)  -  (-a2  -  5ab  +  3b2) 

y2  +  2y  -  7)  +  (2y2  -  4y  +  3)  -  (-8y  -  10  +  3y2) 

3x3  -  2x  +  1)  +  (5x2  -  3x3  -  3)  -  (6x3  -  2)  -  (7x2  +  1  -  2x) 

8a3  -  3a2b  +  7ab2  -  b3)  +  (9b3  -  3a2b  +  7ab2  -  3a3)  -  (a3  -  b3) 


[7-6] 


[7.01] 


B. 


Sample.     If  f(x)  =  3x2  -  5x  +  7,    f(a)  +  f(3a)  = 


Solution.  f(a)  =  3a*  -  5a  +  7 

f(3a)  =  3(3a)2  -  5(3a)  +  7 
=  27a2  -   15a  +  7 
f(a)  +  f(3a)  =  30a2  -  20a  +  14 


1.  If  f(x)  =  4x2  -  8x  -  3,    f(2a)  +  f(a)  = 

2.  If  g(x)  =  2x3  -  x2  +  7,    g(3x)  -  g(-x)  = 

3.  If  g(x)  =  x3  -  x2  +  7,     g(3x)  =  ,     3g(x)  = 

[3g](x)  =  ,     [3o  g](x)  = 

4.  If  f(x)  =  7x2  +  2x  +  9,    f(3)  =  and  f(10)  = 

5.  If  f(x)  =  8x3  +  7x2  +  5x  +  6  and  g(x)  =  4x2  +  3x  +  9,    f(10)  +  g(10)  = 

6.  Suppose  that,    for  each  x,    f(x)  =  x     -  2x  -  3.     What  is  the  slope  of 
the  line  containing  the  points  (2,  f(2))  and  (2  +  h,    f(2+h)),    where 

h  ^  0? 


C_.    Here  is  a  column  proof  of  the  theorem  '2  +  2  =  4'.     Fill  in  the  mar- 
ginal comments. 

(1)  V    x  =  x 
x    '  x 

(2)  2+2=2+2 

(3)  2  =  1  +  1 

(4)  2  +  2  =  2  +  (1  +  1) 

(5)  VxVyX  +  (y  +   1)  =  (x  +  y)  +   1 

(6)  2  +  (1  +  1)  =  (2  +  1)  +  1 

(7)  2  +  2  =  (2  +  1)  +  1 

(8)  3=2+1 

(9)  2+2=3+1 

(10)  4  =  3+1 

(11)  2  +  2=4 


[7.01]  [7-7] 

MULTIPLICATION 

Just  as  statements  of  the  computing  facts  of  addition  are  really 
theorems  derivable  from  the  basic  principles  and  definitions  of  certain 
numerals  and  of  place  value,    so  are  statements  of  the  computing  facts 
of  multiplication.     As  an  example,    here  is  a  proof  of  the  theorem 
•2«3  =  6\ 

2*3    =  2(1  +  1  +  1)  [definition] 

=  2  •  1  +  2  •  1  +  2  •  1  [idpma] 

=  2  +  2  +  2  [pml] 

=  4  +  2  [addition  table] 

=  6  [addition  table] 

If  we  had  previously  proved  that  2*2  =  4,    we  could  have  given  a  shorter 
proof.     [Explain.  ] 

In  filling  out  the  rows  of  the  addition  table  you  could  use  the  theorem: 

V  V    x  +  (y  +  1)    =   (x  +  y)  +  1, 

■*■    y 

which  is  a  consequence  of  the  apa.  In  filling  out  a  row  in  the  multipli- 
cation table,  the  first  entry  is  given  by  the  pml  and,  to  find  the  others 
it  is  convenient  to  use  a  theorem: 

V  V    x(y  +1)    =  xy  +  x 
x    y     w  7 

[What  basic  principles  do  you  need  to  refer  to  in  order  to  prove  this 
theorem?]     For  example, 

2-4   =   2(3  +  1)  [definition] 

=    2*3  +  2  [theorem] 

=   6  +  2  [theorem] 

=   8  [addition  table] 

Now,    prove  42-  5  =  10',     '2-6  =  12',     ...,     andl2-9  =  18\ 

As  in  the  case  of  addition,    the  multiplication  table  gives  us  theorems 
which  are  used  in  carrying  out  the  steps  in  the  multiplication  algorithm 
which  you  learned  in  grade  school. 


[7-8] 


[7.01] 


Again,    there  is  a  similar  algorithm  for  simplifying  certain  types  of 
algebraic  expressions.     Study  the  following  examples  of  its  use. 

Example   1.      Simplify:      (3x2  +  2x  +   l)(7x  +  3) 


Solution. 


3x'  +  2x  +   1 

7x  +  3 

9x2  +  6x  +  3 
Zlx3  +  14x2  +  7x 


21x3  +  23x2  +  13x  +  3 


Example  2.      Simplify:      (5y     -  3y  +  2)(2y     -  7) 


Solution. 


5yJ  -     3y  +  2 

2 


2/ 


-  7 


-  35y* 
10y5    -     6y3  +  4y2 


+  21y  -   14 


lOy5    -  41y3  +  4y2  +  21y  -   14 

EXERCISES 

A.    The  procedure  used  in  Example   1  above  is  like  that  used  in  multi- 
plying a  3 -digit  number  by  a  2 -digit  number.     The  procedure  used 
in  Example  2  is  like  multiplying  a  (  ? )  -digit  number  by  a  (  ? ) -digit 
number. 


B^.    Simplify. 

1.      (2t2  -  3t  +  5)(5t  +  3) 
3.      (8y2   -  3)(5y2  +  y  -   1) 
5.     (6b3  -  3b  +  7)(4b  +  5) 
7.     (8k  +  2k3  -  3)(5  -  7k2) 
9.     (5  -  2x2  +  3x4)(7  +  5x) 


2.  (7x2  +  3x  -  5)(-x  +  7) 

4.  (2a2  -  3a  +  7)(5a  +  4a2  -   1) 

6.  (7r3  +  2r2  +  r)(3r2  -  2) 

8.  (lis2  -  s   -  3)(2s2  +  3) 

10.  (8y3   -  2y  +  7y2)(4y2  -  y  +  3) 


[7.01] 


[7-9] 


11.    (2x4  -  3x2  +  l)(x  -  4)  12.    (3y2  -  y  -  l)(2y2  -  2y  -  1) 


13.    (-a3  -  a2  +  2a)(a2  -  a)        14.    (~a4  -  |a3  +  a2 


l)(fa3  -  -U2  +  1) 


15.  [5(q  -  l)3  -  2(q  -  l)2  +  7(q  -  1)  -  8]  [2<q  -  l)2  -  3(q  -  1)  +  7] 

16.  [3x(2x2  -  x  +  1)  -  5x2(3x-2)][4x(x-  1)  -  3x(2  -x)] 

<3.    Complete  each  of  the  following. 
1.     V    (  )(2x)  =  6x2  +  lOx 


2.     V    ( 
x  x 


3.     V    ( 
x 


)(x  +  2)  =  x3  +  2x2  +  3x  +  6 


)(4x  +  1)  =  20x3  +  17x2  +  19x  +  4 


D.    Here  is  a  column  proof  of  the  theorem  '2  •  4  =  8'.     Fill  in  the  marginal 
comments. 


(1) 

(2) 

(3) 
(4) 
(5) 
(6) 
(7) 
(8) 
(9) 
(10) 

(11) 


V    x   =  x 

X 


2*4    =   2-4 


3  +  1 


2-4   =   2(3  +  1) 


V    V    x(y  +1)    =   xy  +  x 
x    y       7  } 

2(3+1)    =    2-3  +  2 


2-4    =   2  •  3  +  2 
2-3=6 
2-4    =   6+2 
6  +  2    =   8 
2-4    =    8 


[7-10] 


[7.01] 


MISCELLANEOUS  EXERCISES 
A.     1.     If  f(x)  =  3x2  -  5x  +  7  and  g(x)  =  2x2  +  3x  -  5  then  [fg](x) 

2.  If  f(x)  =  5x2  -   1  and  g(x)  =  3x  -  7  then  [fg](x)  = 
and  [gf](x)  = 

3.  If  f(x)  =  4x2  +  2x  -  3  and  g(x)  =  2x  +  1  then  [f°  g](x)  = 
and  [g°  f](x)  = 

B_.    Simplify. 

1.  (x  +  2)(x  -  3)  +  (x  +  7)(x  -  5) 

2.  (y  -  5)(y  -  7)  +  (y  -  3)(y  +  5) 

3.  (2y  -  4)(y  +  8)  +  (3y  -  7)(2y  +  5) 

4.  (a  -  2b)(3a  +  b)  +  (7a  -  b)(5a  +  3b) 

5.  (3x  -  y)(2x  +  5y)   -  (6x  +  y)(3x  +  y) 

6.  (7x  -  4)(15x  -  9)  -  (8x  +  5)(13x  -  7) 

7.  (5y  -  7)(y+  3)  +  (5y  -  7)(y  -  3) 

8.  (2x  +  3y)(2x  +  7y)   -  (2x  +  3y)(x  -  4y) 

9.  (3x  -  4y)(x  -  2y)   -  (4y  -  3x)(3x  +  5y) 
10.  (5a  -  2b)(7a  -  3b)  +  (3b  -  7a)(5a  -  2b) 


C.     1 


If  the  perimeter  of  this  rectangle  is 
34,    what  is  the  area-measure? 


5x  +  2 


i 


If  the  area-measure  of  this   rectangle 
is  33,    what  is  the  perimeter? 


3x  +  5 


3.     Find  the  measures   of  the  diagonals  of  the  rectangles  in  Exer- 
cises  1  and  2. 


[7.01] 


[7-11] 


D.    Simplify. 

1.     (x  -  4)(x  -  7)(x  +  2)(x  -  3) 

3.     (a  +  l)(a  +  2)(a  +  3)(a  +  4) 

5.     (a  -  6)(a  +  7)(a  -  8)(a  +  9) 


2.  (y  -  3)(y  +  8)(y  -  2)(y  +  6) 
4.  (b  +  2)(b  -  2)(b  +  3)(b  -  3) 
6.     (2x  -  l)(3x  -  7)(5x  -  3)(4x  +  7) 


E.     1.     Suppose  that  f(x)  =  3x3  -  5x2  -  2x  +  8.      Compute   f(2),     f(-2), 
£(j),    f(l),    f(0),    f(-|),   and  £(0.05). 

2.     Suppose  that  g(x)  =  5x4  -  3x    +  x    -  x  +  1.     Compute  g(6),    g(~5), 
and  g(9). 


F_.    Replace  the  letters  by  digits. 

1.     1ABCDE 
*   3 

ABCDE1 


2.         SEND 

+  MORE 

MONEY 


G.    Solve  these  problems. 

1.  The  weight  of  a  circular  disc  varies  jointly  as  the  square  of  the 
radius  and  the  thickness.     If  the  ratio  of  the  thicknesses  of  two 
discs  is  9  to  8  and  if  the  weight  of  the  first  disc  is  twice  that  of 
the  second,    what  is  the  ratio  of  their  radii? 

2.  Two  numbers  are  in  the  ratio  5  to  37.     What  number  should  be 
added  to  each  so  that  the  sums  are  in  the  ratio  1  to  3  ? 


3.  Find  three  consecutive  even  numbers  whose  sum  is  204. 

4.  How  many  pounds  of  a   12%  salt  solution  are  required  to  yield 
84  pounds  of  salt  ? 

5.  A  printer  used  1890  digits  in  numbering  the  pages  of  a  certain 
book.     How  many  pages  are  in  the  book? 

6.  Prove  that  none  of  the  numbers    11,     111,     1111,     11111,    ...    is 
the  square  of  an  integer. 


[7-12] 


[7.01] 


DIVISION-WITH-REMAINDER  ALGORITHM 

The  addition,    subtraction,   and  multiplication  algorithms  are  pro- 
cedures for  simplifying  indicated  sums,    differences,   and  products  of 
positive  integers  named  by  decimal  numerals.     We  can  extend  these 
algorithms  to  apply  to  all  integers  by  taking  account  of  various  theorems 
about  opposites. 

The  situation  with  division  is  not  so  simple.     Subtraction,    which  in 
some  ways  is  analogous  to  division,    always  "comes  out  even".     But,   as 
you  know,    division  doesn't.     In  earlier  grades  you  learned  an  algorithm 
for  finding  the  quotient  of  a  first  number  by  a  second  when  the  second  is 
a  factor  of  the  first.     Take  the  example  of  simplifying  '867  t  17'. 


51 

17/867 

1 

50-17    = 

I         850 
17 

1  •  17    = 

I            17 

867 

=    51-17 

1             0 

The  content  of  the  dashed  box  suggests  what  is  behind  the  algorithm.     The 
algorithm  shows  that  867  -  50  •  17  -   1  •  17  =  0.     So,    867  =  50  •  17  +  1  •  17  = 
51 •  17. 

As  you  know,  this  algorithm  can  still  be  used  to  yield  important 
information  in  cases  in  which  the  quotient  is  not  an  integer.  Look  at 
this  example : 


51 


r 


17/868 

50-17    = 

850 

18 

1  •  17    = 

17 

868   =   51 • 17    + 


L--n 


868 


The  result  of  applying  the  algorithm  in  this  example  shows  us  that  — r= 


868 


is  not  an  integer.     More  specifically,   it  tells  us  that      .  _    is  between  the 

86  8  1 

integers   51  and  52,     Further,    it  tells  us  that      .  ^     differs  from  51  by  y=- 

All  this  information  is  conveyed  by  the  equation  4868  =  51-17  +   1*. 


[7.01] 


[7-13] 


In  fact,   the  purpose  of  the  algorithm  is  to  arrive  at  just  such  an  equation 
So,   the  algorithm  is  called  the  division-with-remainder  algorithm. 

This  algorithm  has  an  analogue  used  for  transforming  certain  alge- 
braic expressions.     For  example,    consider  the  expression: 

20x3  +  17x2  +  19x  +  6 

and  the  problem  of  transforming  it  to  an  expression  of  the  form: 

( )(4x+   1)  +(__.) 

Here  is  a  way  of  finding  expressions  to  fill  the  blanks: 

5x2  +      3x    +     4 


4x  +  1        /20x3  +  17x2  +  19x  +  6 


5x2(4x  +  1)  =!     20x3  +     5x2 


3x(4x  +  1)  = 


4(4x  +  1)  = 


1 2x2  +   1 9x 
12x2  +     3x 


I6x  +  6 
I6x  +  4 


20x3  +  17x2  +  19x  +  6  =  (5x2  +  3x+4)(4x  +  1)  + 


"2_n 


Let's  do  another  problem: 


V     5x4+  2x2  +  3x  +  5    =    (_?_)(x  -  3)  +  (_?_) 


5x3  +   15x2  +  47x    +   144 


-   3/5x' 


+      2x2  + 


3x  + 


5x4  -    15x: 


15xJ  +     2x^ 
15x3  -  45x2 


47  x2  + 


3x 


47x,:   -    141x 


144x  +        5 
144x  -  432 

437 


So,    V     5x4  +  2x2  +  3x  +  5  =  (5x3  +   1 5x2  +  47 x  +   144)(x  -   3)  +  437. 


[7-14]  [7.01] 


EXERCISES 

A.    Use  the  division-with-remainder  algorithm  to  complete  the  following 
sentences. 

1.  379  =  -15+                                           2.     5032  =  •  28  + 

3.  V  2x3  +  x2  +  x  +  16  =  (  )(x  +  2)  +  (_      _) 

4.  V  15x4  +  5x3  -  3x2  +   llx  +  4  =  ( )(3x+l)  +  (_      _) 

5.  V  6x4  +   17x2  +  7x  +  25  =  (  )( 2x2  -  2x  +  5)  +  (  _      _) 

6.  V  3x4  -  8x3  -  7x  +  2  =  (  )(x2  +  2x  -  5)  +  (  ) 

7.  V  x4  -  2x2  -  35  =  (  )(x2  -  7)  +  (  ) 

8.  V  27x3  +  64  =  ( )(3x+4)  +  (  ) 

9.  V  x3  -  8  -  6x2  +  12x  =  (  )(x  -  2)  +  (  ) 

J^  ■    ' '  '  ""  ™  ™  ™" 

10.  V     2x4  -  3x  +  7  =  (  )(x  -  5)  +  (  ) 

J^  '       '  "■  ™  "  ~ 

11.  V     2x4  -  3x  +  7  =  (  )(x  +  4)  +  (  ) 


vU       vO       *«, 

•"l"     *"l%     *v 


The  second  example  on  page  7-13  told  us  that,   for  each  x, 

(*)  5x4  +  2x2  +  3x  +  5  =  (5x3  +  15x2  +  47x  +  144)(x  -  3)  +  437. 

Suppose  that  f  is  a  function  such  that  f(x)  =  5x4  +  2x2  +  3x  +  5,    and  sup- 
pose that  we  wish  to  find  f(3).     One  way  to  do  this  is  to  substitute  *3* 
for  4x'  in  the  expression  '5x4  +  2x2  +  3x  +  5'  and  then  simplify.    Another 
way  is  to  use  (*).     Do  you  see  how? 

Now,    suppose  that  we  wish  to  find  f(-7).     We  can  use  the  division- 
with-remainder  algorithm  to  show  that,    for  each  x, 

5x4  +  2x2  +  3x  +  5  =  (5x3  -  35x2  +  247x  -  1726)(x  +  7)  +  12087 

From  this  it  is  easy  to  see  that  f(-7)  =   12087. 


o^     O,     vi. 
,,x      ^x      ,t> 


[7.01] 


[7-15] 


B.    1.     Suppose  that  f(x)  =  6x4  +  2x3  -   I6x2  -   lOx  +  1.     Use  the  division- 
with-remainder  algorithm  to  compute  f(2),   f(6),   f(-7),    and  f(2/3) 


2. 


(a)  Suppose  that  g(x)  =  x4  -  14x3  +  71x2  -  154x  +  120.  Use  the 
division-with-remainder  algorithm  to  compute  g(2),  g(3), 
g(4),   and   g(5). 

(b)  Solve  the  equation  4x4  -  14x3  +  71x2  -   154x  +  120  =  0'. 


3.  Show  that  4x  -  5'  is  a  factor  of  4x3  -  9x2  +  23x  -   15'.     You  could 

use  the  division-with-remainder  algorithm.    [If  4x  -  5*  is  a  factor, 
what  would  the  remainder  be?]    Do  the  problem  a  different  way. 

4.  Show  that  4x2  +  x  -  2'  is  a  factor  of  4x5   -  x4  +  9x2  -  7x  -  2'. 


5. 


Which  of  the  following  fractions  can  be  reduced  to  lower  terms? 


(a) 
(c) 
(e) 


(x 

+  5)(x 

-  2) 

x  -  2 

x2 

+  4x  - 

21 

X2 

+  x  - 

12 

X3 

-  6x2  - 

•  7x  + 

60 

x*  -  3x  -  18 


(b) 
(d) 
(f) 


x2  +  7x  -   18 
x  -  2 

x2  +  lOx  -  21 
x2  -  7x  +  12 

x2  -  2x  -  3 
x3  +  3x2  -  25x  +  21 


C.     1.     If  an  edge  of  cube  is  3x  -  2  inches  long,   then  the  surface  area  is 
square  inches  and  the  volume  is  cubic  inches. 

2.     If  the  surface  area  of  a  cube  is  ( 3x  +  12)(2x  +  8)  square  inches, 
what  is  its  volume? 


3. 


If  the  volume  of  the  pictured  rectangular 
solid  is  2x3  +  x2  -  8x  +  21  cubic  feet,   what 
is  the  area  of  the  shaded  face? 


4.     If  the  volume  of  a  cube  is  x3   -   1 5x2  +  75x  -   125  cubic  inches,    how 
long  is  a  diagonal  of  a  face?     How  long  is  a  diagonal  of  the  cube? 


[7-16]  [7.01] 

OTHER  COMPUTING  FACTS 

In  order  to  establish  a  computing  fact  such  as,    for  example,    that 

5  5 

j-  •  2  =   -_-,     we  use  the  principle  of  quotients  and  the  division  theorem. 

Here  is  a  column  proof: 

(1)  V    V    V    xyz    =    x(yz)  [apml 

Xyz  Lr-j 

(2)  |*2"3    =   |(2'3)  [(!)] 

(3)  2*3    =    6  [theorem] 

(4)  VxVy^0   7^    =    X  fp^ 

(5)  6/0  [     ?     ] 

(6)  | -6    =    5  [(4)  and  (5)] 

(7)  |-2"3    =    5  I<2>'   <3>'    a^d(6)] 

(8)  VV     /  rt  V    if  zv    =    x  then  z  =   —  [theoreml 

xy/Oz  y 

(9)  3/0  [     ?     ] 

(10)  if  |-  •  2  •  3    =    5  then  |  •  2  =  |-  [(8)  and  (9)] 

(11)  |-2    =   |  [(7)  and  (10)] 

In  the  foregoing  proof,    in  addition  to  basic  principles  [apm  and  pq]  and 
theorems  derived  from  them  ['2  •  3  =  6'  and  the  division  theorem],    we 
used  some  inequations  ['6  /  0'  and  '3  /  0'].     Similarly,    in  Unit  2,    in 
proving  the  important  theorems: 

V      -r-  =  x       and:         V      — r  =   -x, 
x     1  x     - 1 

we  used  41  /.  0'  and  '  — 1  /  0'.     Now,    it  is  easy  to  see  that  none  of  these 

inequations  can  be  derived  from  our  ten  basic  principles.     All  you  need 

do  is  to  imagine  there  are  no  real  numbers  except  0,    and  that  '1'  is  just 

another  name  for  0.      Nov/,    look  at  each  of  the  ten  basic  principles: 


[7.01] 


[7-17] 


Commutative  principles 

VV    x  +  y  =  y  +  x 
x    y  ' 

V   V    xy  =  yx 
x    y     7       7 


Associative  principles 

V  V    V    x  +  y+z  =  x  +  (y+z) 
x    y    z  '  7 

V  V    V    xyz  =  x(yz) 
xyz7  7 


Distributive  principle 

V   V   V    (x  +  y)z  =  xz  +  yz 
xyz  7  7 


Principles  for  0  and  1 


V    x  +  0  =  x 
x 


V    x  •  1  =  x 

X 


Principle  of  opposites 


V    x  +  -x  =  0 
x 


Principle  for  subtraction 

VV    x-y  =  x+  — y 
x    y  7  ' 


Principle  of  quotients 
:    y^O 


VV     /  „  (x  t  y)  •  y  =  x 


Since  0  +  0  =  0  +  0   and  0*0  =  0  '0,    the  commutative  principles  are 
certainly  true  for  the  number  system  you  are  imagining. 

If  you  also  imagine,   as  definitions  of  addition  and  multiplication, 
that  0  +  0  =  0  and  0*0  =  0  then  you  can  check  at  once  that  the  associative 
principles  and  the  distributive  principle  also  hold. 

Remembering  that  *1'  is,   for  the  moment,   just  another  name  for  0, 
you  see  that  the  principles  for  0  and  1  also  hold. 

Defining  oppositing  and  subtracting  by:    -0  =  0,  and:    0-0  =  0/   you 
see  that  the  principle  of  opposites  and  the  principle  for  subtraction  hold. 

Finally,    since   there    aren't  any  numbers  except  0,   the  principle  of 
quotients  is  true,   too.     [If  you  don't  believe  this,   try  to  find  a  counter- 
example! ] 

Now,   if  we  could  prove  from  the  ten  basic  principles  that,   for  example, 
1^0  then,    since  the  basic  principles  are  true  for  your  imagined  number 
system,    41  /  0'  would  also  have  to  be  true  of  this  system.      But,   it  isn't. 


[7-18] 


[7.01] 


So,    if  we  want  to  have  enough  basic  principles  to  reaLly  prove  all  the 
theorems   of  Unit  2,    we  should  add,    at  least,    41  /  0'  to  our  list  of  basic 
principles.     [We  have  left  space  below  the  pml  on  page  7-17  so  you  can 
do  this.     Do  it  now.] 

How  about  the  other  inequations?     For  example,    do  we  need  to  take 
4-l  /  0'  as  a  basic  principle?       The  answer  to  this  question  is   'no';    we 
can  prove : 


Theorem  79. 

V     if  x  i  0  then 

X 

-x  ^  0 

[From  this  and  the  basic  principle  '1  fi  0'  it  follows  that  -1^0.    Why?] 


(1) 
(2) 
(3) 
(4) 
(5) 
(6) 

(7) 

(8) 

(9) 

(10) 

(11) 

(12) 


-a  =  0 


V    x  =  x 

X 


* 


a  +  -a  =  a  +  -a 
a  +  0  =  a  +  -a 


V    x  +  0  =  x 
x 


a  +  0  =  a 


V    x  +   -x  =  0 
x 


a  +  -a  =  0 
a  =  0 
if  -a  =  0   then   a  =  0 
if  a  fi  0   then    -a  £  0 
V    if  x  £  0    then    -x  /  0 


[assumption]: 
[Identity] 

[(2)1 

[(1)  and  (3)] 

[paO] 

[(5)] 

[po] 

r'.7)] 

[(4),    (6),    and  (8)] 

[(9);    *(D] 

[(10)] 

[(H)] 


You  can  also  derive  Theorem  80  ['-0  =  01]  from  our  eleven  basic 
principles.     Do  this  now. 

Notice  that  Theorem  79  is  equivalent  to: 

V    if  -x  =  0   then   x  =  0, 
x 

and  that  Theorem  80  is  equivalent  to: 


V    if  x  =  0    then    -x  =  0 
x 


[7.01]  [7-19] 

[To  see  this  last  equivalence,    note  that  '-0=0'  can  be  deduced  from 

'V    if  x  =  0  then  -x  =  0'    and  the  principle  of  identity.     And,    on  the  other 
x 

hand,    notice  that  from  the  assumption  'a  =  0'  and  the  theorem  4-0  =  0' 
one  can  infer  '-a  =  0'.     Complete  the  derivation  of  4V    if  x  =  0  then  -x  =  0' 
from  4-0=0'.]     So,    Theorem  79  and  Theorem  80  can  be  combined  into 
a  single  statement: 

(*)  •         V     [x  =  0  if  and  only  if  -x  =  0] 


Ox  Ox  Ox 

'i^      T      *V 


In  writing  conditional  and  biconditional  sentences  we  shall  find  it 

convenient  to  abbreviate  the  connectives  'if  ...   then '  and  4.  .  .   if  and 

only  if '  by  4.  .  .  '==> '  and  4.  .  .    <^^> ',    respectively.     Thus, 

Theorem  79  can  be  written: 

V  [x  /  0  ^    -x  /  0] 

x     L  /J 

and  (*)  can  be  written: 

V  [x  =  0  <=>    -x  =  0] 
x  L  J 

The  brackets  indicate  the  scope  of  the  quantifier.     Why  aren't  brackets 

necessary  when  we  use  the  4if  ...   then '  notation? 

Of  course,    whichever  way  you  write  Theorem  79,    you  still  read  it 

as   'for  each  x,    if  x  is  not  0  then  the  opposite  of  x  is  not  0'.     If  it  is 

written  with  the  i^=^>\    it  is  sometimes  convenient  to  read  it  as   4for  each 

x,    x  is  not  0  only  if  the  opposite  of  x  is  not  0'. 


Ox       O,       Ox 
"C      "Is      "JN 


Our  eleven  basic  principles  are  enough  to  justify  the  first  80  theo- 
rems  on  pages  7-145  through  7-  149.     However,    it  is  not  possible  to 
prove,    from  these  alone,   that  2^0,      [You  can  show  this  impossibility 
by  the  same  kind  of  argument  we  used  to  show  that  41  p  0'  cannot  be 
derived  from  the  ten  basic  principles   of  Unit  2.     Imagine  that  there  are 
no  real  numbers  other  than  0  and  1  and  that  0  -f   1  .     Then,  define  addition, 
multiplication,    opposition,    subtraction,    and  division  in  a    "natural" 
way.     (Hint :     Let  1   +   1=0.)     Now,    show  that  all  eleven  basic  principles 
are  true  statements.] 

Since,    for  the  real  numbers,    Z  f-  0 ,    it  follows  that  we  need  more 
basic  principles.     In  the  next  section  we  shall  introduce  additional  basic 
principles  which  will  enable  us  to  prove  not  only  '2  £  0',    '3  fi  0',    etc., 
but  will  also  give  us  a  basis  for  proving  theorems  about  the  relation  >. 


[7-20]  [7.01] 

MISCELLANEOUS  EXERCISES 

A.  Expand  and  simplify. 

1.    (5a  +  b)(5a-b)  2.     (x  +  7)(x  +  8)  3.     (y  +  9)(y  +  9) 

4.  (y  +  9)(y  -  9)  5.     (3x  -  4)2  6.     (3z  -  4)(3z  +  4) 

7.  (3z  +  4)2  8.     (7a+2)(5a-6)  9.     (ax  +  b)(ax  -  c) 

10.  [(a  -  b)  +  c][(a  -  b)   -  c]  1 1 .    [x  -  y  +  z][x  -  y  -  z] 

12.  5pq(p2   -  pq  -  q2)  13.     7x2y3(x4  -  x2y  +  y4) 

14.  (2a  -  3b)(5a2  -  2ab  +  7b2)  15.    (3x2  -  2x  +  l)(5x2  -  7x  -  2) 

B.  Simplify. 

1.     81-53+81-47  2.    93-67  +  67-7  3.    8-19*12i 

4.     (7-33-j)  -(9-3)  5.     12  •  87  i  +  12j  •  12         6.    ( 1 1  • 16  |)  •  (5  • 6) 

C.  Simplify. 

x  -  3        2_  x  +  4        5_  8        y  -   1 

x+8x  x  -  3        x  'yy+2 

4.  ^  7  x  +   1         x  -   3  ,       x  -  4        x  -  5 

*x-4        x+5  'x+4        x  -  5  *x+4        x+5 

7      x  +  Z   +    x  -  8  8      2x~  3    +    3x~  7  q      7y  +  3   +    3y  -  2 

*x-5        x+5  *5x+l         5x-l  y-2  3y  -  6 

5x2y3        7a3b  „       3m3n3         8pg4  8(a  -  b)3   v   3(a  +  b)4 

1U*     3ab2  8xy4  L1"     2p2q3    *  12mn4         **'     9(a  +  b)*         2(a  -  bp 

(x-  3)(x  +5)    y(x.-  7)(x  -  5)  x2  +  2x-  15        x2  -  12x  +  35 

(x  +  4)(x-  5)         (x-  5)(x  +  5)  x*  -  x  -  20         x2  +  2x  -   15 

IS      x2  -  x  -    12         x2  +  2x  -   15  .,         6x2  +  7x  -  5  32x2  -  12x+  1 

x2  +  3x-  10        x^  +  7x  +  12  b*     16x2  -  26x  +  3         12x2+  17x-5 

17  x2  -  5  x  +  1         x  -  4  R  x2  -  5  2x+  1         5x  -  3 

*    (x-  3)(x-  2)        x  -  3        x-  2  x2  -  5x  +  6    "    x  -  3         x  -  2 


[7.01]  [7-21] 

ia      3x2  +  x  -  1           4x  +  1        2x-7  .  5x+7           2           x2  -  3x  -  7 

19'    x2  +  5x  -  24   "   ~x~^8~  +  ~ZTT  CK)-  3x  -  3        x  -  9    "   x*  -  10x  +  9 

xz  +  1               8x  +  3        4x+8  ??  x2  +  4x-  21          x2  4  3x  -  10 

Z1"    2x2  +  7x-  15    "    x  +  5       8x-  12  ""  x*  +  llx  +28        x2  +  9x+20 

23.    (x.  *+*)-<*£!. y)  24.  (5-^l  +  (7-^) 


£.     X 

a 

-  1 

2a- 

-i 

-  1 

a  • 

.1 

8 
x  +  y  a  -  1  x  +  2        x  +  3 

D.  Solve  each  system  of  equations. 

1.  x  -  y  =  2      ^  2.     4x  +  3y  =  10   ^  3.       x  +  3y  =  5      ^ 
2x  +  y  =  5       J                         2x  -  2y  =  5      J  2x  +  4y  =  7 

4.     2x  -  y  =  5       ^  5.     3x  -  2y  =  5      ^  6.     2x  -  y  =  -2 

2x  -  3y  =  5    J  6x-4y  =  9      J  5x  +  1  =  -2y 

E.  1.     Find  two  consecutive  integers  the  difference  between  whose  squares 

is  23. 

2.  If  A  can  do  a  certain  job  in  6  days,   and  if  B  can  do  it  in  8  days, 
how  long  will  it  take  them  working  together? 

3.  Mr.  Allen  drives  to  Zabranchburg--a  100-mile  trip- -at  45  miles 
per  hour.     In  order  to  average  55  miles  per  hour  for  the  entire 
trip,   how  fast  should  he  drive  during  the  return  trip? 

4.  Bill  has  an  average  grade  of  74  after  the  first  two  tests.     To  bring 
his  average  up  to  80,   what  must  he  get  on  the  third  test? 

5.  A  car  travels  north  at  45  miles  per  hour,   and  2  hours  later  another 
car  leaves  the  same  place  and  travels  north  at  55  miles  per  hour. 
How  long  will  it  take  the  second  car  to  pass  the  first? 

6.  Rectangle  A  is   14  feet  longer  than  it  is  wide.     Rectangle  B  is  4 
feet  smaller  in  width  but  16  feet  longer  in  length.     If  the  rec- 
tangles have  the  same  area,   what  is  it? 

7.  Given  a  circle  of  radius  10.     Is  the  measure,    i,    of  a  chord  of  this 
circle  a  function  of  its  distance,    d,   from  the  center- -that  is,   is 
there  a  function  f  such  that  i  =  f  «d?     If  so,    give  such  an  f. 


[7-22]  [7.02] 

7.0  2    The  positive  numbers.  --We  concluded  the  last  section  by  pointing 
out  that  our  eleven  basic  principles  for  real  numbers  are  not  sufficient 
to  prove,    among  other  things,   that  2/^0.      Evidently,    we  need  more 
basic  principles.     Let's  find  some  by  trying  to  prove  that  2^0. 

Now,    by  definition,   this  amounts  to  proving  that  1  +  1^0.     Since, 
by  the  principle  of  opposites  and  the  0-sum  theorem,    1  +  1  =  0  if  and 
only  if  -1  =  1,   what  we  need  to  prove  is  that  -1^1.     [Explain.] 

One  way  to  see  that  -1  and  1  are  different  is  to  recall  that  -1  =  "1 
and,   of  course,   that  1  =  *1.      So,    -1  is  a  negative  number,   and  1  is  a 
positive  number.     Hence,  since  the  same  number  cannot  be  both  positive 
and  negative,    1  and  -1  are  not  the  same  number --that  is,    -1^1. 

Now,    several  of  the  things  we  said  in  the  last  paragraph  suggest 
that  in  proving  that  -1/^1  [and,   so,   that  2^0]   it  will  be  useful  to  have 
some  basic  principles  which  deal  with  positive  and  negative  numbers. 
Actually,    we  may  as  well  concentrate  on  the  set  P  of  positive  numbers 
since  we  can,   if  we  need  to,   define  the  set  N  of  negative  numbers  to 
consist  of  the  opposites  of  the  members  of  P.     What  we  need  to  express 
in  the  new  basic  principles  is  how  the  operation  oppositing  and  the  property 
of  being  positive  are  related.     The  following  two  basic  principles  do  this. 

(Px  )     V    [x  ^  0  =>  either  x  €  P  or  -x  €  P] 

(P0  )     V    not  both  x  e  P  and  -x  e  P 

Basic  principle  (Px)  says  that,   no  matter  what  nonzero  real  number  you 
pick,    either  it  or  its  opposite  is  positive.     What  does  basic  principle 
(P2)  say? 

Using  (Px)  and  (P2),   and  the  basic  principle  '1  j£  0',    it  is  easy  to 
prove  that  -1^1.      For,   since  1^0,   it  follows  from  (Px )  that  either 
1  €  P  or  -1  e  P.    But,  this  being  so,  if  - 1  =  1  then  both  1  e  P  and  -1  €  P. 
By  (P2),   this  is  not  the  case.     Hence,    -1^1. 

Since  -1  ■£■  1,    it  follows  from  the  0-sum  theorem  that  1  +  1/^0.    That 
is,    2^0. 

However,    even  with  (Pjl)  and  (P2),   we  cannot  prove  that  3^0.     But, 
it  seems  likely  that  we  are  on  the  right  track,   and  that  what  we  need  are 
more  basic  principles  about  the  set  P.      For  example,   we  could  prove 
that  3  ^  0  if  we  could  show  that  3  e  P  and  0  ^  P.     Now,   we  can  prove  that 


[7.02] 


[7-23] 


0  i  P.       This  follows  from  (P2)  and  the  theorem  [Theorem  80]   that 
-0=0.      So,    we  have: 


Theorem   81. 


0  4  P 


Let's  see  how  we  might  show  that  3  e  P. 

By  definition,    3  =  2+1.     Also,    as  you  probably  recall,    the  sum  of 
two  positive  numbers  is  a  positive  number.     If  we  adopt  this  as  another 
basic  principle  then  we  can  show  that  2  +   1  e  P  by  showing  that  2  e  P  and 
1  6  P.     So,    as  a  third  basic  principle  for  P  we  shall  take: 

(P3)      V    V     [(x  €  P  and  y  e  P)  ==>   x  +  y  e  P] 

Now,    let's  tackle  the  job  of  showing  that  2  e  P  and  that   1  6  P. 

Evidently,    using  (P3)  you  can  prove  that  2  e  P  once  you  have  proved 
that  1  e  P.     [Explain.]    So,    our  only  problem  is  that  of  proving: 


Theorem  82. 
1  6  P 


Do  we  know  enough  basic  principles  to  prove  this?     We  can  see  that  we 
don't  by  noticing  that  if,    in  each  of  the  principles  (P^,    (P2)»    and  (P3)» 
we  replace  4P'    by    *N\    we  get  true  statements,    (Nx),    (N2),    and  (N3), 
about  the  set  of  negative  numbers.     Now,    if  we  could  derive  '1  e  P' 
from  (P^,    (P2)»    and  (P3),    and  our  eleven  other  basic  principles,    then 
we  could,    in  just  the  same  way,    derive  41  e  N'  from  (N1),    (N2),    and  (N3), 
and  our  eleven  other  basic  principles.     Since   1  is  not  a  negative  number, 
we  can't  do  this  . 

Our  trouble  is  that  none  of  the  basic  principles  which  we  have 
accepted  really  distinguishes  between  positive  numbers  and  negative 
numbers.     We  need  a  basic  principle  which  states  some  property  of  P 
which  is  not  also  a  property  of  N.     What  haven't  we  considered?      The 
most  obvious  omission,    so  far,    is  any  consideration  of  multiplication- - 
and,    with  respect  to  multiplication  there  is  a  difference  between  P  and 
N.     The  set  P  is  closed  with  respect  to  multiplication,    but  the  set  N  is 
not.     So,    let's  take  the  following  as  a  fourth  basic  principle  for  P: 


[7-24]  [7.02] 

(P4)     VxVy  [(xe  P  andyeP)  =>  xyeP] 

Now  we  can  prove  Theorem  82.     Here  is  a  proof: 

Since   1/^0,   it  follows  from  (P1)  that  either  1  e  P  or  -1  e  P.     Sup- 
pose that  -1  e  P.     Then,    by  (P4),    -1  •  -1  e  P.     But,    by  Theorem  23  and 
the  pml,    -1  •  -1  =  1*1  =  1.       So,    if  -1  €  P  then  1  e  P.      Since  either 
1  6  P  or  - 1  eP  and  since,  as  we  have  just  shown,   if  - 1  e  P  then  1  6  P, 
it  follows  that,    in  any  case,    1  e  P. 

So,   we  have  proved  that  1  e  P.     Now  [using  definitions  and  (P3)]  it 
is  easy  to  prove  that  2  e  P,    3  e  P,    4  €  P,   etc.     [Incidentally,    we  can  use 
(P2)  to  show  that  -1  f(  P.]      In  addition,   using  Theorem  81,    we  have  a 
new  proof  that  2^0,    and  can  prove  '3  ^  0',     '4  fi  0',    etc. 

We  can  also  prove  theorems  like  '5  £  2'.     For,    if  5  =  2  then  5-2  =  0. 
But,    5  -  2  =  (3  +  2)   -  2  and,    by  Theorem  30,    (3  +  2)  -  2  =  3.     So,    if  5  =  2 
then  3  =  0.     Since  3  /  0,    it  follows  that  5/^2. 

'2  * 

We  can  also  prove  theorems  like     -=-  ^  0  .     The  key  theorem  here  is 

Theorem  54: 

VV     ,[^  =  0=>x  =  0] 

x  y  t  o    y 

?  2 

Since  3^0,    it  follows  that  if  j  =  0  then  2  =  0.     But,    2^0.     So,    j  ^  0. 

EXERCISES 

A.  1.     In  proving  the  theorem  45  fi  2'  we  used  the  theorem: 

***  VxVy  tx  =  y=^  ^  -  y  =  0] 

Prove  (*)  by  first  proving  'V    x  -  x  =  0*. 

2.     Prove  the  converse  of  (*). 

B.  Prove  each  of  the  following. 

1.     4  e  P  2.     If  3  3.     -j  £  1 

4.  No  positive  number  is  its  own  opposite. 

5.  P  is  not  closed  with  respect  to  subtraction. 


[7.02]  [7-25] 


C.    Prove  the  following  theorems. 

Sample  1.      V   V    [(x  e  P  and  y  e  P)    =>  x2  +  2xy  +  y2  e  P] 
x    y 

Solution.     Suppose  that  a  €  P  and  b  e  P.     By  (P3),    a  +  b  €  P. 

So,   by  (P4),   (a  +  b)2  e  P.     But,  (a  +  b)2  =  a2  +  2ab  +  b2. 
Hence,    a2  +  2ab  +  b2  €  P.       So,   if  a  e  P  and  b  e  P 
then  a2  +  2ab  +  b2  e  P.      Consequently, 

V  V     [(x  e  P  and  y  e  P)  =>  x2  +  2xy  +  y2  e  P]. 
x    y 

1.  V   V    [(x-yeP  and  y  e  P)  =>  x  e  P]  [Hint.    Look  at  Theorem  32.  ] 

2.  V    V     [(x  -  y  e  P  and  y  e  P)  =>   x2  -  y2  e  P] 

Sample  2.      V    V     [x  ^  y  ==>   either  x-yeP  or  y-xeP] 
x    y 

Solution.     Suppose  that  a  fi  b.     Then  a  -  b  fi  0    [Why?]   and,    by 
(Px),    either  a  -  b  e  P  or  -{a  -  b)  e  P.       But,    by 
Theorem  33,    -(a  -  b)  =  b  -  a.     Hence,   if  a  ^  b  then 
either  a  -  b  6  P  or  b  •  a  £  P,       Consequently, 

V  V     [x  ^  y  =>   either  x-yePory  -xeP]. 
x    y 


3.     V    V    not  both  x  -  y  e  P  and  y  -  x  e  P 

x    y  J  J 


4.     V    V    V     [(x  -  y  e  P  and  y-zeP)=>x-zeP] 


5.     V    V    V    [x  -  y  e  P  =>   (x  +  z)   -  (y  +  z)  €  P] 
x    y     z  '  w  J 


6.      V    V    V     [(z  €  P  and  x  -  y  e  P)  =£>   xz  -  yz  e  Pi 
x    y     z  L  7  7  J 


«.•-       vl-       k", 

*.,%      '»^     -V 


We  have  defined  the  negative  numbers  to  be  the  opposites  of  the  pos- 
itive numbers.      If  we  are  to  prove  theorems  about  the  set  N  of  negative 
numbers  it  is  convenient  to  incorporate  this  definition  in  a  basic  principle 


[7-26]  [7.02] 

(N)  V    [-xeN   <=>    x  €  P] 

We  shall  not  include  (N)  in  our  "official"  set  of  basic  principles.     How- 
ever,   you  will  be  asked  to  prove  some  theorems  about  the  set  N  in  the 
next  set  of  exercises,    and  for  these  proofs  you  will  need  to  use  (N)  as  a 
basic  principle. 

vl-     o^     o^ 

^,x        ^,<s        ,,v 

D.    Prove  each  of  the  following. 

1.    V    [xe  N  <=>    -xe  P] 
x  L  J 

Sample.     V    [x  £  0  =>   either  x  e  P  or  x  e  N] 

Solution. 

(1)  V    [x  £  0  =>   either  x  €  P  or  -xe  P]  [basic  principle] 

(2)  a  /  0  =>  either  a  e  P  or  -a  €  P  [(1)] 

(3)  V    xeN    <=>    -x  e  P  [theorem--Ex.  1] 

(4)  ae  N   <=>    -aeP  [(3)] 

(5)  a  £  0  ^=>   either  a  e  P  or  a  e  N  [(2)  and  (4)] 

(6)  V    [x/0=>  eitherxePorxeN]  [(1)  -  (5)] 


x 


[Note  that  step  (5)  follows  from  step  (4)  and  step  (2)  by  virtue  of  the 
substitution  rule  for  biconditional  sentences.     This  principle  of 
logic  tells  us  that,   given  a  biconditional  sentence  [(4)]  and  a  second 
sentence  [(2)],   if  either  component  of  the  biconditional  sentence  [in 
this  case,   the  component  4-a  e  P']  is  replaced  by  its  other  compon- 
ent ['a  e  N']  in  the  second  sentence,   the  new  sentence  so  obtained 
[(5)]  is  a  consequence  of  the  given  sentences  [(4)  and  (2)].] 

Here  is  a  paragraph  proof  for  the  same  theorem: 

Since,   by  (PL),   if  a  -f  0  then  either  a  e  P  or  -a  e  P,   and,    since  -a  e  P 
if  and  only  if  a  e  N,    it  follows  that  if  a  fi  0  then  either  a  e  P  or  a  e  N. 
Consequently,    V    [x  £  0  =>   either  x  e  P  or  x  e  N]. 


[7.02] 


[7-27] 


2.     V    not  both  x  e  P  and  x  €  N 
x 


3.     V    [x  £  0  =>  either  x  e  N  or  -x  €  N] 


4.     V    not  both  x  e  N  and  -x  e  N 
x 


5.     V   V    [(x  e  N  and  y  e  N)  =>  x  +  yeN] 


6.     V    V    [(x  €  N  and  y£N)=>  xy  6  P] 


7.     V    [x  f  0  =>  x2  e  P] 


MISCELLANEOUS  EXERCISES 


A.     1. 

2. 

3. 

4. 

5. 

6. 

7. 

8. 

9. 
10. 
11. 
12. 
13. 


f  g(x)  =  x2  -  3x  -  28  then  g(17)  =        ,    g(27)  =        ,    and  g(6)  =        . 

f  f(x)  =  (x  -  7)(x  +  4)  then  f(  17)  =        ,    f(27)  =        ,   and  f(6)  = 

f  h(x)  =  x2  +  x  -  30  then  h(15)  =        ,   h(-6)  =        ,   and  h(35)  = 

f  f((x,  y))  =  6x2  -  y2  then  f(3,  2)  =         and  f(2,  3)  = 

i  g(x,  y)  =    {-^  -   ^-  then  g(3,  2)  =         and  g(2,  3)  =        . 

f  h(x,  y)  =  (5x2)  -  (5  +  x)2  +  xy2  then  h(3,  2)  =       and  h(2,  3)  = 

f  f(a,  b,  c)  =  -3ab  +  5a2c  then  f(-4,  -12,  3)  = 

f  g(a,  b,  c)  =  18  -  a  -  6c  +  2ab  then  g(-4,  -12,  3)  = 

f  h(m,  n)  =  mn2  +  m2n  then   h(3,   -6)  = 

£   f(x)  =  3(x  -  2)(x  +  2)   -  4(x2  -  5)  +  (x  -  8)(x  +  1)  then  f(-5)  = 

f   f(x)  =  8X  then  f(l)  =        ,   f(2)  =        ,   and  f(3)  = 

13  1 

£  g^X*  =    x~^2    +   x~^T  then  Si1)  =        »    g(3)        »    and  g(  j)  = 

f  f(x)  =  x2  +  1  and  g(x)  =  9  -  x2  then  for  what  argument  do  f  and  g 
have  the  same  value? 


[7-28]  [7.02] 

B.    Tell  which  expressions  are  equivalent  to  the  given  one. 

1.  mx  +  my  +  nx  +  ny 

(A)    (m  +  n)(x  +  y)         (B)    m(x  + y)  +  (x  +  y)n       ( C)    m(y  +  n)  +  x(m  +  y) 

2.  xy  +  3x 

(A)    y(x  +  3)  (B)    x(y  +  3)  (C)    x(y  +  2)  +  x 

3.  xy  +  xz  +  yz 

(A)    x(y  +  z)  +  yz  (B)    y(x  +  z)  +  xz  (C)    x(y  +  z  +  x) 

4.  T  +  Ts 

(A)    T(l  +  s)  (B)    s(l  +  T)  (C)   (T  +  1)(1  +  s) 

5.  y(l  +  x)2  +  y(l  +  x)2x 

(A)    y(l  +  x)3  (B)    y(l  +  x)(  1  +  x2)  (C)    yx  +  y(  1  +  x)3 

6.  Kk  -  K 

(A)   k(K  -  1)  (B)   K(k  -  1)  (C)   k(K  -   1)  +  k  -  K 

7.  tx  -  ty  +  sx  -  sy 

(A)    (t  -  s)(x  +  y)  (B)    (t  +  s)(x  -  y)  ( C)    (t  +  x)(s   -  y) 

8.  xy  -  xz  +  yz 

(A)    x(y  -  z)  +  yz  (B)    z(x  -  y)  +  xy  (C)    -y( -x  -  z)    -  xz 


C.     Compute. 

1.     23  -32  2.      89   -f    87  3.      (-3)2-  (-2); 

4.    (23)4-r  2U  5.      2-32  6.      5  +  2-  3Z 


_D.    True-or -false  ? 

1.     32  +  33  =  35  2.      32  +  33  =  65  3.      32  •  33  =  36 

4.     32-33  =  95  5.      38   ~    32  =  34  6.      (34)2=36 


[7.01] 


[7-29] 


E.    Solve  these  equations. 
1 .     5x  -  2  +  7(  3  -  x)  =  1 2 

3.     x  +  2(5x  -   1)  =  -x  -  3 


2.     2  -  (1   -  x)  +  7x  =  5  -  3(x  -  6) 


4.     6  +  (x  -  3)2  =  (x  +   l)2  -  2 


F.     1.     Draw  a  square  and  mark  a  point  in  its  plane.     Draw  a  line 

through  this  point  such  that  the  line  cuts  the  square  region  into 
regions  of  equal  area-measure. 

2.  Repeat  Exercise  1  for  a  regular  hexagon. 

3.  Given  an  equilateral  triangle  AABC.     Find  the  radius  of  the  cir- 
cular arc  with  center  at  A  which  bisects  the  triangular  region. 

4.  Draw  a  square  ABCD  and  draw  perpendicular  lines  Jt  and  m  such 
that  &  intersects  each  of  a  pair  of  opposite  sides  and  m  intersects 
each  of  the  other  pair  of  opposite  sides.     Show  that  the  segment 
of  i  determined  by  the  intersection  points  is  congruent  to  the 
segment  of  m  determined  by  m's  intersection  points. 

5.  Two  players  at  a  circular  table  take  turns  in  putting  pennies  on 
the  table.     The  pennies  may  touch  but  not  overlap.     The  loser 
is  the  player  who  can't  find  a  place  to  put  down  a  penny.     The 
first  player  can  always  win.     How? 

6.  Remove  two  opposite  corner  squares  from  a  chessboard.     Can 
the  remainder  of  the  board  be  covered  without  overlapping  by 

•  rectangular  cards,    each  card  covering  exactly  two  squares? 


EXPLORATION  EXERCISES 
1.     Graph  the  sentence     ly  -  x  e  P'     on  a  picture  of  the  number  plane 


2.     What  name  is  customarily  given  to  the  relation  {(x,  y) :    y  -  x  e  P} 
which  you  pictured  in  Exercise   1? 


[7-30] 


[7.03] 


7.0  3    Inequations.  --The  principles  (P1),    (P2),    (p3)>    a^d  (P4)  for  the  set 
P  of  positive  numbers  are  also  basic  for  establishing  theorems  about 
the  relation  >.     In  fact,    you  may  recall  from  your  work  in  Unit  2  that 
we  defined  4>'  by: 


(G)        V    V     [y>  x   <=>    y  -  xeP] 
x     y 


We  shall  now  accept  (G)  as  a  new  basic  principle. 

(G)  tells  us  what  is  meant  by  statements  of  inequality  such  as   '7  >  2* 
and  '5  >   9'.     The  statement  47  >    2'  is  just  another  way  of  saying  that 
7  -  2  is  a  positive  number.     Similarly,    '5  >   9'  is  another  way  of  saying 
that  5   -  9  is  a  positive  number.     [Since  7   -  2  is  a  positive  number,    we 
know  that  47  >    2'  is  true.     How  about  '5  >   9'?] 

Of  course,    since  the  relation  <   is  merely  the  converse  of  >,    we 
could  get  along  without  the  symbol  '<'.     However,    we  shall  use  it  occa- 
sionally.    When  we  do,    you  should  think  of  a  sentence  of  the  form 
'.  .  .    <  'as  merely  another  way  of  writing  one  of  the  form  '  >    .  .  .  * 

[Similarly,    a  sentence  of  the  form  '.  .  .    >    '  is  just  an  abbreviation  for 

one  of  the  form  ' .  .  .    >  or   .  .  .    =  ' .  ] 

Another  thing  that  (G)  tells  us  is  that  the  real  numbers  which  are 
greater  than  0  are  precisely  the  positive  numbers.     We  state  this  as: 


Theorem  83. 

x  e  P] 

V      x  >   0    <^ 
x  u 

— > 

Do  you  see  how  to  derive  Theorem  83  from  our  basic  principles?     Try 
to  do  so  before  reading  the  column-proof  which  follows. 

=>    y  -  x  €  P]  [basic  principle] 


(1) 
(2) 
(3) 
(4) 
(5) 
(6) 


V   V    [y  >  x 
x    y 


a  >  0    <^=>    a  -  0  e  P 


V      x 
x 


0  =  x 


a  -    0 


>   o    <^=>    a  e  P 


V     [x  >   0 
x  L 


xe  P] 


[ID] 
[theorem] 

[(3)] 

[(2),    (4)] 
[(1)  -(5)] 


[7.03] 


[7-31] 


In  paragraph  form  we  could  write: 

By  (G),  a  >  0  if  and  only  if  a  -  0  €  P.  But,  by  Theo- 
rem 43,  a  -  0  =  a.  So,  a  >  0  if  and  only  if  a  e  P.  Hence, 
V    [x  >  0  <=>    x  €  Pi. 

Theorem  83  is  useful  in  that  it  enables  us  to  replace  any  sentence 
of  the  form  *.  .  .  £  P'  by  one  of  the  form  *.  .  .   >  0'.    So,  for  example,  we 
can  restate  the  theorem  41  €  P*  as  41  >  0'.     Theorem  81--'0  i  P'--can 
be  restated  as  '0  /  0'.      Also,   since  Theorem  81  is  equivalent  to  the 
statement: 

Vx  [x  =  0  =>  x  4   P] 


and,   so,   to  the  statement 


V    [xe  P 
x  l 


/o], 


Theorem  83  can  be  used  to  translate  Theorem  81  to: 

V    [x  >  0  =>  x  £  0] 


Other  convenient  ways  of  saying  the  very  same  thing  are: 

and: 


x  c  P      r 


\>o^° 


EXERCISES 

A.      1.     Use  Theorem  83  to  translate  our  basic  principles  (P.^,   (P2), 
(P3),    (P4),   and  (G),   into  statements  which  do  not  contain  *P*. 
The  last  of  these  five  statements  we  shall  call  'Theorem  84'. 

2-.     Can  you  use  Theorem  83  to  translate  the  five  statements  you 
obtained  in  Exercise  1  back  into  the  basic  principles  (Pj^), 

(P2),    (P3),    (P4),   and  (G)? 

O,      s.K      *U 


Theorem  84. 


V   V    [y  >  x   <=>    y-x  >   0] 


J^         m.h         O, 


[7-32] 


[7.03] 


B.    One  useful  corollary  of  Theorem  84  is: 


Theorem  85. 

-x  >  0] 

V    [x  <  0   <r-^> 

x  l 

Prove  this  theorem.     [Compare  this  theorem  with  Exercise  1  of 
Part  D  on  page  7-26.] 

C.     1.     Now  look  at  the  theorems  in  Sample  2  and  Exercises  3,   4,    5, 

and  6  of  Part  C  on  page  7-25.     With  the  help  of  (G)  and  Theorem 
83  we  can  translate  these  five  theorems   into  useful  theorems 
about  >.      Do  so,     [These  five  new  theorems  are  parts  a,   b^,   c, 
d,   and  e  respectively,    of  Theorem  86.      These  five  theorems 
could  be  taken  as  a  set  of  basic  principles  for  >.     See  Part  ^D 
on  page  7-35. ] 

2.     Give  a  column-proof  to  show  that  Theorem  86b  is  a  consequence 
of  (G)  and  the  theorem  of  Exercise  3  on  page  7-25.      [See  the 
solution  of  the  Sample  on  page  7-26.] 

INEQUALITY  THEOREMS 

In  Part  C  of  the  preceding  exercises  you  discovered  Theorem  86. 
This  theorem  is  an  immediate  consequence  of  earlier  theorems  on 
positive  numbers,   and  basic  principle  (G). 


Theorem  86. 

a. 

V   V    [x  /£  y  =>   either  x  >  y  or  y  >  x] 
x    y 

b. 

V    V     not  both  x  >  y  and  y  >  x 
x    y                                '             7 

c. 

V    V    V    [(x  >  y  and  y  >   z)  =^  x  >   z] 

x    y    z  L            7            7                                J 

d. 

V    V    V     fx  >   y  =>   x  +  z  >   v  +  zl 

x    y    z                                           '          J 

e. 

V    V    V    [(z  >   0  and  x  >  y)  =>  xz  >   yz] 

x    y    z 

[7.03] 


[7-33] 


Other  theorems  on  inequations  can  be  proved  in  a  simiiar  manner,   but 
it  is  often  easier  to  deduce  such  theorems  from  Theorem  86  and  our 
first  eleven  basic  principles. 
For  example,    consider: 


Theorem  87. 


V     x  f  x 
x 


As  it  happens,   there  is  a  previously  proved  theorem  about  P  which  can 
be  used  in  proving  Theorem  87.       [Before  reading  further,   try  to  guess 
which  theorem  this  is,   and  use  it  in  proving  Theorem  87.]    However,   it 
is  much  simpler  to  derive  Theorem  87  from  one  of  the  parts  of  Theorem 
86.     Do  so  now.     [Hint.     Recall  that  *x  f  x'  is  short  for  'not  x  >  x'.] 

Theorem  87  is  equivalent  to: 

V    V     [x  =  y  =>  x  f   y] 
x    y  L  J  ii* 

[Explain  why.]    This,    and  another  consequence  of  Theorem  86b: 

(*)  V    V     [y  >   x=>   x^   y] 

x     y 

are  very  useful.     We  can  combine  them  into  a  single  theorem: 


Theorem  88a. 


V    V     [y  >  x  =>   x  f   y] 

x     y  L/   —  i      i  i 


[Explain.  ] 

To  see  how  Theorem  88a  is  used,  consider  the  following  theorem 
which  you  probably  recognize  as  the  addition  transformation  principle 
for  inequations  : 


Theorem  89. 

w      W       W        I            i              \               i               ^- 

^       i 

V    V    V      x+z>y+z<= 
x    y     z  L                   ' 

=>    x  >   y] 

The  if -part  of  this  theorem  is  Theorem  86d.     So,   to  prove  Theorem  89, 
all  we  need  do  is  prove  its  only-if-part : 


[7-34]  [7.03] 

(**)  VVV    [x+z>y+z=>x>y] 

x    y     z 

This  can  be  proved  in  a  considerable  number  of  ways.     Probably  the 
simplest  of  these  ways  is  the  following: 

Suppose  that  a  +  c  >  b  +  c.     Then,  by  (G),    (a  +  c)   -  (b  +  c)  €  P  and, 
since  (a  +  c)  -  (b  +  c)  =  a  -  b,    a  -  b  €  P.     So,    by  (G)  again,    a  >  b. 

However,    for  purposes  of  illustration,    consider  the  following  proof 
of  (**): 

Suppose  that  a  +  c  >  b  +  c.     Since  a  =  b  or  a  /  b,   it  follows  from 
Theorem  86a  that  either  a  =  bora>   b  or  b  >  a- -that  is,   that  either 
a  >  b  or  b  >  a.     Suppose  that  b  >  a.     If  b  =  a  then  b  +  c  =  a  +  c,   and 
if  b  >  a  then,    by  Theorem  86d,    b  +  c  >  a  +  c.      So,   if  b  >_  a  then 
b  +  c  >  a  +  c.    But,  by  Theorem  88a,  ifb  +  c>a  +  c  then  a  +  c  f  b  +  c. 
Hence,   if  b  ^  a  then  a  +  c  p  b  +  c.     Since,  by  hypothesis,  a  +  c  >  b  +  c, 
it  follows  that  b  ^  a.      But,   as  shown  above,    either  a  >  b  or  b  •;  a. 
So,   a  >  b.     Therefore,    if  a  +  c  >   b  +  c  then  a  >  b.       Consequently, 

VVV     [x  +  z  >   y  +  z  =>  x  >  y] . 

x    y    z    L  '  7J 

One  advantage  of  this  method  of  proof  is  that  it  can  be  used  to  derive: 
(***)  VVV     [(z  >   0  and  xz  >   yz)  =>  x  >   y] 

Jt         y  Z 

from  Theorem  86e.     Do  this  now. 


EXERCISES 
A.    On  page  7-33  we  mentioned  that  the  theorem: 

(*)  VV[y>x=>x/y] 

x  y 

is  an  immediate  consequence  of  Theorem  86b: 

V    V     not  both  x  >   y  and  y  >  x 

x    y  7  7 

By  common  sense,   it  sounds  as  though  anyone  who  accepted  Theo 
rem  86b  would  agree  to  (*).     That  is,    it  looks  as  though  each 
inference  of  the  form: 


[7.03]  [7-35] 

not  [p  and  q] 
q   =>    not  p 

is  a  valid  inference.     Let's  use  the  rules  of  reasoning  we  studied  in 
Unit  6  to  show  that  this  is  the  case: 

*  t 

e a 

pands * 

p  =>  [p  and  q] not  [p  and  q] 

not  p 
q  =^  not  p 

1.  You  can  use  this  kind  of  argument  to  show  that:    b  >  a  =>  a  ^  b 
is  a  consequence  of  the  single  premiss:    not  both  a  >  b  and  b  >  a 
What  assumptions  would  you  use  and  discharge  during  the  proof? 

2.  As  shown  in  the  tree -diagram,  the  proof  would  contain  four  infer- 
ences.    Give  the  rules  of  reasoning  which  justify  the  inferences. 

B.  1.     Prove  the  converse  of  Theorem  88a  from  Theorem  86a.     [See 

second  sentence  of  the  second  proof  of  (**)  on  page  7-34.] 

2.     Find  a  third  proof  for  (**). 

C.  Prove  each  of  the  following. 

1.  V    x  +  1  >  x  [Theorem  90] 

2.  V   V   V   V    [(x  >  y  and  u>v)=>x  +  u>y  +  v]       [Theorem  91] 

3.  V   V   V    [(x  >  y  and  y  >  z)  =>  x  >  z]  [Theorem  92] 

x     y      z 

4.  VxV    [(x>yandy>x)    r=>  x  =  y]  [Theorem  93] 
[Hint.    Use  Theorem  86a  on  page  7-29.] 

5.  VxV    [-x  >   -y  <=>    y  >  x]  [Theorem  94] 

^"JD.    Show  that  the  five  parts  of  Theorem  86  constitute  an  adequate  set  of 
basic  principles  for  >  by  deriving  from  them  the  five  statements  you 
obtained  in  solving  Exercise  1  of  Part  A  on  page  7-31. 


[7-36]  [7.03] 

THE  TRANSFORMATION  PRINCIPLES  FOR  INEQUATIONS 

In  Unit  3  you  studied  theorems   which  justified  the  various  steps  taken 
in  solving  equations  and  inequations.      As  you  learned  there,    the  trans- 
formation principles  for  equations  are  derivable  from  the  first  ten  of  our 
basic  principles.       [See  Theorems  6  and  7,    Theorems   11  and  48,    and 
Theorems  56  and  15.]    You  are  now  ready  to  consider  the  proofs  of  the 
transformation  principles  for  inequations: 

The  addition   transformation  principle  for  inequations 

VVV     [x+z>y+z  <==>  x  >   y] 
x    y    z  J  J 

The  multiplication  transformation  principle  for  inequations 

a.  V    V    V     .     _    [xz  >  yz  <=>  x  >   y] 

—  xyz>0L  7  7J 

b.  VVV.,,    [xz  <  yz  <=>  x  >  y] 

—  x    y    z  <   0    L  J  '  J 

The  factoring  transformation  principle  for  inequations 

a.  V    V    [xy  >  0  <=>  ([x  >  0  and  y  >  0]  or  [x  <   0  and  y  <  0])] 

b.  V    V     [xy  <   0  <=>  ([x  >   0  and  y  <   0]  or  [x  <   0  and  y  >   0])  ] 

x    y 

[These  principles  are  Theorems  89,    95,    and  96.] 

The  addition  transformation  principle  is  an  immediate  consequence 
of  Theorem  86d  and  (**)  on  page  7-34. 

Part   a_  of  the  multiplication  transformation  principle  is  a  conse- 
quence of  Theorem  86e  and  (***)  on  page  7-34.       [Recall  now  how 
Theorems  86a  and  88a  are  used  in  deriving  (***)  from  Theorem  86e.] 

It  is  easy  to  derive  part   b_  of  the  multiplication  transformation 
principle  from  part    a_   by  using  Theorem  85,    Theorem  20,    and 
Theorem  94.     Here  is  a  proof: 

By  Theorem  85,   for  c  <   0,    -c  >   0.     So,    by  part   a    of 
the  multiplication  transformation  principle, 

a  •  -c  >  b  •  -c    <=>    a  >  b. 

Hence,    by  Theorem  20, 

-(ac)  >    -(be)    <=>    a  >   b. 


[7.03]  [7-37] 

So,    by  Theorem  94   [Exercise  5  of  Part  C  on  page  7-35], 

ac  <  be    <=>    a  >  b. 
Consequently,    V    V    V    <   _  [xz  <  yz   <==>    x  >  y]. 

Parts    a   and   b    of  the  factoring  transformation  principle  for  inequations 
will  be  proved  in  Part  A  of  the  following  exercises. 

EXERCISES 

A.     1.     Prove  the  if -part  of  part   a   of  the  factoring  transformation  prin- 
ciple.    Do  this  by  first  proving: 

if  (a  >  0  and  b  >  0)  then  ab  >  0 
and  then  proving: 

if  (a  <  0  and  b  <   0)  then  ab  >  0 

2.     Now,    let's  prove  the  only-if-part  of  part   a   of  the  factoring  trans- 
formation principle. 

Suppose  that  ab  >  0.     By  Theorem  81,    ab  £  0.     So,    by  the 
principle  for  multiplying  by  0,    b  ^  0.     Hence,    by  Theorem  86a, 
either  b  >   0  or  b  <   0. 

Suppose  that  b  >  0.     By  part   a   of  the  multiplication  transfor- 
mation principle,   it  follows  that 

ab  >  0b  only  if  a  >  0. 
Since  0b  =  0,    it  follows  that,   for  b  >  0, 

ab  >  0  only  if  a  >  0. 
Since  ab  >  0,   it  follows  that  a  >  0.     So,    for  b  >  0,   we  have 

both  a  >  0  and  b  >  0. 

Next,    suppose  that  b  <  0.     [Show  that  in  this  case  it  follows  that 
both  a  <  0  and  b  <  0.] 

Since  either  b  >   0  or  b  <   0,    it  follows  that  either 

(a  >  0  and  b  >  0)  or  (a  <  0  and  b  <  0). 

Hence,    if  ab  >   0  then  [(a  >   0  and  b  >   0)    or    (a  <  0  and  b  <   0)]. 
Consequently,    we  have  proved  the  only-if-part  of  part   a    of  the 


[7-38] 


[7.03] 


B. 


factoring  transformation  principle.     This  together  with  Exercise 
1  gives  us  part  a_. 

3.     Prove  part   b   of  the  factoring  transformation  principle.     [Hint. 
As  in  proving  part    t>    of  the  multiplication  transformation 
principle  on  page  7-36,   you  may  find  it  helpful  to  use  Theorem 
85;   then  use  part  a  of  the  factoring  transformation  principle.] 


1.  Prove  that  the  squaring  function  maps  each 
nonzero  real  number  on  a  positive  number. 
Interpret  this  graphically.   [Theorem  97a] 

2.  Suppose  that  a  and  b  are  arguments  of  the 
squaring  function  for  which  the  corres- 
ponding values  are  the  same.     What  do  you 
know  about  a  and  b?     Prove  it. 

3.  Suppose  that  a  and  b  are  nonnegative  argu- 
ments of  the  squaring  function  for  which 
the  corresponding  values  are  the  same. 
What  do  you  know  about  a  and  b?     Prove 
it.  [Theorem  98a] 


4.  Suppose  that  a  and  b  are  arguments  of  the  squaring  function  such 
that  the  corresponding  values  are  different.     In  particular,   sup- 
pose that  b2  >  a2.      Can  you  conclude  that  b  >  a?     If  you  suppose 
that  b2  >  a2  and  b  >  0,   can  you  conclude  that  b  >  a?     Can  a  be  -b? 
Can  a  be  less  than  -b? 

5.  Suppose  that  a  and  b  are  arguments  of  the  squaring  function  such 
that  b  >  a.     Does   it   follow  that   b2  >  a2?     Would  additional  knowl- 
edge about  b  allow  you  to  conclude  that  b2  >  a2?     How  about  more 
knowledge  about  a? 


6.     (a)    State  the  theorems  you  discovered  in  Exercises  4  and  5. 
(b)    Prove  them.  [Theorems  98b  and  c] 


[7.03] 


[7-39] 


C. 


Suppose  that  quadrilateral  ABCD  is  a 
square  and  that  quadrilateral  CDEF 
is  a  rhombus  whose  diagonals  are  2a 
units  and  2b  units  long. 

1.     Compute  the  area-measure  of 
the  square  and  the  rhombus. 


2.  Is  the  area-measure  of  the  square  ever  less  than  the  area-meas- 
use  of  the  rhombus?     Are  they  ever  equal? 

3.  State  and  prove  a  theorem  about  real  numbers  which  tells  you 
that  if  the  rhombus  is  not  a  square  then  the  area-measure  of  the 
square  is  greater  than  that  of  the  rhombus.      [Theorem  97b] 
[Hint.   You  may  find  it  helpful  to  use  the  theorem  *V     /  n  x2  >  0'.] 


x^  0 


Q(T'    2) 


D.  't  The  figure  at  the  left  shows  the 

graph  of  the  function 

«x,  y),    x  >   0:    y  =   i}. 

1.  What  are  the  side -measures  of 
the  rectangle  with  vertex  Q? 
What  is  the  area-measure  of 
this  rectangle? 

2.  What  is  the  area-measure  of 
the  rectangle  with  vertex  P? 

3.  What  is  the  perimeter  of  the  rectangle  with  vertex  Q? 

4.  Must  the  two  rectangles  shown  have  the  same  perimeter? 

5.  Could  you  choose  a  point  on  the  curve  such  that  perimeter  of  the 
corresponding  rectangle  is  greater  than  100?  If  there  are  such 
points,    give  the  coordinates  of  one  of  them. 

6.  Among  the  rectangles  we  have  been  considering,    which  has  a 
minimum  perimeter?      Which  a  maximum? 


[7-40]  [7.03] 

E.     1.     Graph  the  reciprocating  function  r  defined  by: 

r(x)    =    — ,      for  x   ^    0 
[Use  cross -section  paper  and  a  unit  length  of  about  1  inch.] 

2.  (a)    On  the  figure  you  drew  for  Exercise   1,    graph  the  equation 

'y  =  x\ 

(b)    Use  these  two  graphs  to  draw  the  graph  of  the  function  p 
where  p(x)  =  x  +  — ,    for  x  >  0.     ["addition  of  ordinates"] 

3.  Does  p  have  a  minimum  value? 

4.  State  a  theorem  which  tells  what  you  have  guessed  in  doing  Exer- 
cise 3.      [V    .    _    x  +   —  > 

L   x  >   0  x  —         J 

5.  Compare  the  theorem  in  Exercise  4  with  Theorem  97b.     Use  the 
latter  in  proving  the  former.  [Theorem  97c] 


o,     *■,     v^ 

T      T»      T 


As  you  have  seen  in  Exercise  1  of  Part  E,   the  reciprocating  function 
has  positive  values  just  for  positive  arguments.       That  is, 

V     /  n     [-  >   0    <=>    x  >   0]. 
x  f-  0     L  x  J 

Let's  prove  this . 

By  Theorem  97a,    for  a  f  0,    a2  >  0.     So,  by  the  multiplication 
transformation  principle  for  inequations,    for  a  f  0, 

I  >  0    <=>     -  «a2  >  0  -a2, 
a  a 

So,    since  0  •  a2  =  0  and  since,    for  a  ^  0,    —  •  a2  =  a, 

I  >   0    <=>    a  >   0. 
a 


Consequently,     V     /  _    f—  >   0    <=>    x  >   01. 

^  '  x  f-  0    L  x  J 


*!,       «JU       O^ 

•v     t    -v 


[7.03]  [7-41] 

F.    Prove  each  of  the  following. 

1.     VVV     ,  n[2L  >    i    <=>    xz  >  yz]  [Theorem  99a] 

xyz/=0Lzz 

2«     VxVy^0    [~>0    <=>    xy>0] 

3'     <*>      Vy  ^  Q  Vz  ^  Q  [yz  >   0  ->   (y  >    z   <^>     I  >    I)] 

(b)  V    .    .  V     /  .  V     /  n    [yz  >  0  =>   (y  >   z   <=>     -  >    -)] 
'        x>0y^0z^0L7  7  z         y   J 

(c)  V.nV/nV._[y>z->~>-]  [Theorem  100] 

x>0y/=0z>0L7  z         y 


G.    The  theorem  lV     /  _    [—  >   0  if  and  only  if  x  >   Ol'  tells  you  that  the 
—  x  f-  0    L  x  '  J  7 

reciprocating  function  has  positive  values  just  for  positive  argu- 
ments.     What  does  the  theorem  in  Exercise  3(a)  of  Part  F  tell  you 
about  the  values  of  the  reciprocating  function? 


H.    Suppose  that  f  is  a  real-valued  function  whose  domain  is  the  set  of 
real  numbers  and  is  such  that 

(*)  VxVy  [x>   y  =>  f(x)  >   f(y)]. 

Prove  each  of  the  following. 

1.  V    V     [£(x)   >   f(y)    =>   x>   y] 

x     y 

2.  V    V     [f(x)  =   f(y)   =>   x=    y] 

x.     y 


I_.      Graph  each  of  the  following  sentences  on  a  picture  of  the  number  line 
1.     5x  -  3  >   7  -  5x  2.     2(x  +  3)  >  4  +  6x 


~      x-  5   .     x  +  4  5x  -  2    -  , 

3.     -r-  >    -T-  4.      1  <    —3—I6 


5.     x(x  -  3)  >  0  6.     (2x  -   l)(x  +  3)  >  0 


[7-42] 


[7.03] 


7.     x(x  -  2)  >  x(x  -  3) 


8.     x2  -  x  -  6  <  0 


9.     -  >    10 
x 


11.     x+  i  >   % 
x         2 


13.      (x2  +  5x)2  <  36 


15.     <*  -  »><V  4>   >  0 

x  +  5 


10. 
12. 


x  -  1 

6x  -  5 


<   1 


>  7 


14.     (x  +  5)(x  -  l)(x  -  4)  >  0 


*16.     JLLL   >   2 


Vx*  +  1 


J.     Graph  each  of  the  following  sentences. 

1.     y  >  3x  -  5  2.     2x  +  y  -  3  >  0 

3.     x  -  2y  +  1  <  0  4.     y  -  3  <  0 

K,    Find  all  ordered  pairs  of  integers  which  satisfy  the  given  sentence 

1 .  2x  +  y  -  4  >  0  and  x  -  2y  +  3  <  0  and  y  -  4  <  0 

2.  4  -  y  <  4x  <  3y  <  9 

3.  4x  +  y  >  4  and  2x  -  y  <  0  and  x  +  2y  <  6 

4.  There  is  a  triangle  whose  sides  are  x  inches,   y  inches,   and  5 
inches  long  and  y  =  2x. 


JL.    1.     If  three  cans  of  peaches  and  four  cans  of  pears  cost  less  than 

$1.60,   and  four  cans  of  peaches  and  four  cans  of  pears  cost  more 
than  $1.80,   can  you  buy  a  dozen  cans  of  peaches  for  $2.40? 

2.     In  a  certain  candy  store,   a  lollipop  costs  less  than  a  piece  of 

bubblegum.     If  you  have  just  enough  money  to  buy  a  piece  of  bub- 
blegum  and  someone  gives  you  2  cents  more,   you  still  won't 
have  enough  to  buy  three  lollipops.     But,   if  you  have  8  cents,   you 
have  more  than  you  need  to  buy  a  piece  of  bubblegum  and  two 
lollipops.     What  is  the  cost  of  a  lollipop  and  the  cost  of  a  piece 
of  bubblegum? 


[7.03]  [7-43] 


MISCELLANEOUS  EXERCISES 


A.    Simplify. 

1. 

7         2 

11    "    3 

2. 

11         12 

3. 

5a  +  b        a  -  b 
a  +  b        a  +  b 

4. 

7x  +  y           x  -  y 
x  +  y         2(x+y) 

5. 

I    i    1 

a        b        c 

6. 

4              5 
a  -  b        b  -  a 

7. 

k                 k 

8. 

k              k 

9. 

x2  -  4y2  m         5x 

x  -  2y         2x  -  y 

x  -  2y      2x  -  y 

x  -  2y       (x  +  2y)2 

10. 

9mb          9m 
b+1        b+  1 

11. 

x2-16         3x+15 
x  +  5             x  -  4 

12. 

9mb  .    9m 

b+1      b+1 

13. 

x2  -  16      3x+15 
x  +  5           x  -  4 

14. 

1         2 

a-  b  +  V 

15. 

k2              1 

k  -  1        k2  -  k 

16. 

x2  -  4x  +  3           xy 
9  -  x2            x  -  3 

17. 

....*?.„  +  ! 

9  - 12        t  -  3 

18. 

t*        .       t 

9  - 12    *    t  -  3 

B^.    Graph  each  of  the  following  sentences. 

1.     y  =    |3x  -  4  |  2.y  =  x+|x|  3.    y  =  4x  -  x2 


4.     y  =  4  -    |x|  5.    y  =    |4  -  x|  6.    y  =    |4x  -  x 

7-  y  =  x~hr  8-  y  =  r^w  9-  y  =  x2  +  xTtt 


C.    Which  of  the  following  equations  have  no  roots? 
1.     x  +  x  =  x  2.    x+i=i  3. 


xx  x  +  2        x  +  3 


4x  -  8    _    3x  -   12  3  5 


x  +  4            x  +  4              *    x        x(x  -   1)  x(x  -   1) 

ID.    Solve. 

1.     (x  +  5)(x  -  4)  =  (x  +  5)(2x  -  9)  2.  y(y  -   3)  =  y(2y  +  5) 

3.     a2  -  5a  +  6  =  2a2  -  6a  4.  a2  -  5a  +  6  =  a2  -  3a 

5.     k(k-  l)(k-3)  =  k(2k  +   l)(k-  1)  6.  (y  +  2)(y-4)(y  +  5)  =  40(y  -  4) 

7.     x(x  +  7)  =  (x  +  7)(x  +  3)  8.  ^-i    =     3a  +  7 

3  3a 


[7-44]  [7.03] 

E.     1.     Find  a  point  on  the  y-axis  which  is  twice  as  far  from  (4,    0)  as 
from  (-1,    0). 

2.  What  point  on  the  graph  of  'y  =  x'  is  equidistant  from  (6,    0)  and 
(6,    8)? 

3.  A  point  moves  so  that  it  is  equidistant  from  (4,    0)  and  (0,    4). 
Find  an  equation  of  its  path.     [Hint.    Suppose  that  the  point  has 
coordinates  (x,  y).     Then,    .  .  .    ] 

4.  A  point  moves  so  that  its  distance  from  (0,    0)  is  twice  its 
distance  from  (3,   0).     Find  an  equation  of  the  path.     "What  is  the 
path? 

5.  Suppose  that  P  is  a  moving  point  whose  path  is  the  circle  with 
center  (0,    0)  and  radius  2.     Find  an  equation  of  the  path  of  M, 
the  midpoint  of  PA,    where  A  is  the  point  (8,   0).     Sketch  the  path 
of  M. 

6.  By  how  much  does  0.  3333  differ  from   -^  ?      By  how  much  does 
0.  142857  differ  from   j  ? 

7.  Mr.    Johnson  travels  by  car  between  Zilchville  and  Zabranchburg. 
One  tenth  of  the  trip  is  through  towns  and  the  rest  is  on  the  open 
highway.     If  the  speed  limit  in  towns  is   15  miles  per  hour  and  on 
the  open  highway  50  miles  per  hour,    can  he  average  40  miles  per 
hour  for  the  entire  trip  without  exceeding  the  speed  limits? 

8.  Which  is  the  larger,    -rj-    or    ■£-=-,     or  are  they  equal? 

9.  Mr.   Jones  starts  from  Zabranchburg  to  drive  to  Frampton,    24 
miles  away.     Fifteen  minutes  after  starting  he  meets  with  an 
accident.       After  wasting  5  minutes  trying  to  fix  the  car,    he 
proceeds  at  half  speed  to  a  service  station  2  miles  farther  along 
the  road.     Three  quarters  of  an  hour  later  he  starts  out  again, 
and  by  driving  12  miles  an  hour  faster  than  at  first,   he  manages 
to  get  to  Frampton  50  minutes  later  than  he  had  planned.     What 
is  his  usual  driving  speed? 


[7.03] 


[7-45] 


EXPLORATION  EXERCISES 
A.     Consider  the  eleven  sets  listed  below. 

(a)  the  set  of  all  real  numbers 

(b)  the  set  of  all  rational  numbers 

(c)  the  set  of  all  irrational  numbers 

(d)  the  set  of  all  negative  numbers 

(e)  the  set  of  all  nonnegative  numbers 

(f)  the  set  of  all  integers 

(g)  the  set  of  all  integers  greater  than  5 
(h)    the  set  I+  of  all  positive  integers 

(i)     {1,    2,    3,   4,    5,    6,    7,    8,    9,    10} 

(J)     {-y>   0,   y,    1,    l~,    2,    .  .  .  }  [What  does  4.  .  .  '  mean?] 


2  2  2  2 

(k)    {5,    5-j,     6,    6-j,    7,    7-j,    8,    8y,    ...} 


(1) 

(2) 

y 

1.  For  each  of  the  eleven  sets,    put  a  check  mark  in  column  (1)  if 
the  number   1  belongs  to  the  set. 

2.  For  each  of  the  sets,   put  a  check  mark  in  column  (2)  if,  for  each  x, 

if  x  belongs  to  the  set  then  x  +  1  belongs  to  this  set. 

3.  For  which  sets  do  you  have  check  marks  in  both  columns? 

4.  Name  five  other  sets  S  such  that 

1  e  S    and     V     [x  e  S    =>    x  +  1  e  Si. 
x  L  J 


B_.    Eet?7\=   {S,   S  a  set:     1  c  S  and  V    [x  €  S    =>  x  +  1  e  S]}. 

1.  Name  some  members  of77"\. 

2.  How  many  sets  belong  to/71? 

3.  Do  you  think  that  there  is  a  member  of  777  which  is  a  subset  of 
each  member  of 777? 


[7-46]  [7.03] 

C.    In  Part  A  you  noticed  that   I+  e#(--that  is,   that 
(i)  1  €  I+ 

and  that 

(ii)  V     [x  e  I+    =^  x  +   1  e  I*]. 

Here  is  a  proof,    using  (i)  and  (ii),   that  2  e  I+: 

(1)  2=1+1  [definition] 

(2)  V    [xel+=»x+le  I+]  [(ii)] 

(3)  1  e  I+  =>   1  +  1  €  I+  [(2)] 

(4)  1  e  I+  [(i)] 

(5)  1  +  1  €l+  [(3),  (4)] 

(6)  2el+  [(1),  (5)] 

1.  In  Part  A  you  also  noticed  that  the  set  R  of  all  rational  numbers 
belongs  toTTT.      How  could  you  easily  obtain  a  column  proof,   in 
which  you  used  no  other  fact  about  R,   for  the  theorem  '2eR'? 

2.  Suppose  that  all  you  are  told  about  some  set  S  of  real  numbers 
is  that  S  eTf\.      Does  it  follow  from  this  that  2  e  S?      [Why?] 

3.  Prove  that  3  €  I+.      [Hint.    You  may  use  the  previously  proved 
theorem  '2  e  I+  '.  ] 

4.  Repeat  Exercise  2  with  43  e  S'   in  place  of   42  e  S'. 

5.  Repeat  Exercises  3  and  4  with  '4'  in  place  of  '3'. 

6.  Do  you  think  that  there  is  a  positive  integer  which  you  could  not 
prove  belongs  to  I+  by  using  (i)  and  (ii)?      [Given  lots  of  time!] 

7.  In  view  of  your  answer  for  Exercise  6  [and  your  answers  for  the 
preceding  exercises],    what  are  some  of  the  numbers  which  you 
believe  belong  to  each  set  S  which  belongs  toT^T? 

8.  Consider  the  following  statement: 

(iii)        Vg  [( 1  €  S  and  Vx  [x  e  S  =>  x  +  1  e  S])   ==>  I+  C  S] 
Do  you  believe  what  this  says? 


[7.04]  [7-47] 

7.04    The  positive  integers.  --In  an  earlier  section  you  proved  that  1  €  P» 
that  2  €  P,  that  3  €  P,  and  that  4  €  P.     Clearly,   we  could  have  continued 
and  shown  that  5  e  P,  that  6  €  P,    .  •  •  until  we  got  tired.    As  you  know, 
the  real  numbers  1,   2,    3,   ...  are  called  the  positive  integers.     Could 
we  have  proved  that  each  of  them  really  is  positive?    The  answer  is  'no*. 
One  reason  for  this  is  that  a  generalization  with  an  infinite  number  of 
instances  cannot  be  proved  just  by  proving  instances  of  it.    Another 
reason  why  it  is  impossible  at  this  stage  to  prove  that  each  of  the  so- 
called  * 'positive'*  integers  belongs  to  P  is  that  our  basic  principles  say 
nothing  about  the  positive  integers.     In  order  to  prove  theorems  about  I+ 
--the  set  of  positive  integers --we  need  additional  basic  principles. 

Back  in  Unit  4  you  did  prove  some  theorems  about  I*,  and  in  order 
to  do  so  you  made  certain  assumptions.    They  were  the  following: 

(1)  The  positive  integers  belong  to  P,  that  is,  I*  C,  P. 

(2)  I+  is  closed  with  respect  to  addition  and  multiplication. 

(3)  For  all  positive  integers  m  and  n,  if  n  >  m  then  n  -  m  is  a 
positive  integer. 

(4)  1  is  the  only  positive  integer  between  0  and  2 

(5)  Each  nonempty  set  of  positive  integers  has  a  least  member. 

(6)  For  each  real  number  x,  there  is  an  integer  k  such  that 
k  <  x  <  k  +  1. 

(7)  Each  positive  integer  other  than  1  has  just  one  prime  factori- 
zation. 

So,  in  effect,  in  Unit  4  you  adopted  (l)-(7)  as  additional  basic  principles. 
Now,   certainly,  this  is  a  heterogeneous  lot  and  there  is  not  even  any 
reason  to  believe  that  they  are  adequate  for  proving  all  the  theorems  we 
may  want  to  prove  about  I*.     Let's  try  to  find  some  simpler  basic  prin- 
ciples which  [together  with  the  sixteen  we  already  have]  will  do  the  job. 

We  said  that  the  positive  integers  are  the  real  numbers  1,   2,   3,    . . .  . 
Our  problem  is  that  of  making  this  statement  more  precise- -actually,   of 
explaining  the  * ...  *  . 

Let's  suppose  that  someone  who  doesn't  know  asks  you  what  the 
positive  integers  are.     You  might  begin  by  saying  that  the  positive 


[7-48]  [7.04] 

integers  are   1,    2,    3,    and  so  on.     To  test  his  understanding  of  your 
answer,    he  might  then  ask,    "Is  4  a  positive  integer?'       You  would 
probably  tell  him  that,    yes,    4  is  a  positive  integer  and  so  is  5,    and  so 
on.     And  you  might  add  that,    for  each  positive  integer  x,    x  +  1  is  a  posi- 
tive integer.     So,    so  far,    you  have  told  him  two  important  things  about  I+: 

(a)  1  e  T 

(b)  V        ,+  x  +  1  €  I+ 

x  e  I 

Knowing  these,    he  has  a  method  for  picking  out  an  unlimited  number  of 
positive  integers.     But,    he  still  has  one  more  important  question: 
"Are  there  any  positive  integers  which  can't  be  gotten  this  way?"     To 
this  you  would  say,    "No,    those  are  all  the  positive  integers  there  are.  " 
This  last  bit  of  information  is  a  third  basic  principle  about  I+.     Let's 
try  to  restate  it  in  a  more  usable  form.     Let  K  be  the  set  of  all  those 
real  numbers  which  can  be  proved  to  be  in  I+  by  using  (a)  and  (b).    Then, 

1  e  K  and  V        v  x  +  1  e  K 

x  €  K 

Your  answer,    above- -that  K  contains  all  the  positive  integers  - -amounts 
to  saying  that  I+C_K.     To  see  what  this  entails,    suppose  that  S  is  a  set 
of  real  numbers  such  that 

(1)  1  e  S  and  V        c  x  +   1  e  S 

x  e  s 

If  a  real  number  c  e  K  then,    by  the  definition  of  K,    there  is  a  proof 
using  (a)  and  (b)  that  c  e  I+.      Consider  such  a  proof.     If  you  replace  all 
the  *I+ 's  in  it  by    4S's,    you  then  have  a  derivation  of  'c  e  S'  from  (1). 
So,    if  S  satisfies  (1)  then  each  member  of  K  is  a  member  of  S,    that  is, 
K  C^  S.      In  other  words, 

VQ  [(1  e  S  and  V        c  x  +  1  e  S)  =>   K  C  Si. 

So,    if  you  say  that  I+  C  K  then  you  are  implying  that 

(*)  VQ[(leSandV       c  x  +  1  e  S)  =>  I+  C   S]. 

o  x  £  o  — 

On  the  other  hand,    suppose  that  you  assert  (*).      Since   1  e  K  and 
V..X+   leK,    you  are  implying  that  I+CK. 


[7.04] 


[7-49] 


So,   (*)  says  precisely  that  I+  C  K.     In  other  words,    (*)  tells  us  that 
each  positive  integer  can  be  gotten  from  (a)  and  (b). 
Let's  use  (*)  to  prove  (1)  on  page  7-47: 


Since  P  is  a  set  of  real  numbers,   it  follows  from  (*)  that 
if  ( 1  €  P  and  V  x  +  1  e  P)  then  I+  C  P. 

Now,    by  Theorem  82,    1  e  P.      By  Theorem  82  and  the  basic  closure 
principle  (P3),    V        p  x  +  1  e  P.     So  [by  modus  ponens],   I+C_P. 

Although,    as  you  have  just  seen,    (*)  is  convenient  for  proving 
certain  theorems,    it  will  be  more  useful  to  take  as  our  third  basic 
principle  for  I+  the  following: 

(c)    Vs  [(l  e  S  and  Vx  e  I+  [x  e  S  =>  x  +  l  e  S])  ^Vx£l+xeS] 

Using  (a)  and  (b)  it  is  possible  to  show  that  (*)  and  (c)  are  equivalent. 

In  order  to  state  our  three  basic  principles  for  I+  in  a  simpler 
form,    and  to  simplify  the  statements  and  proofs  of  later  theorems, 
we  shall  (as  above]  use  lS'  as  a  variable  whose  values  are  sets  of  real 
numbers,    and  use  lm',    'n',    'p',    and  lq'  as  variables  whose  domain  is  I+. 
So,  for  example,  in  place  of  the  sentences   'V       T+  x  e  S',    we  shall  write 
'Vm  m  e  S'  or  4V    q  e  S'.     In  particular,    lVm  m  e  I+  '  is  an  abbreviation 


m 


for  the  trivial  theorem  'V        t+  x  e  I+' 


Using  these  conventions  about 
variables  we  can  restate  our  three  basic  principles  for  I+  as  follows: 


(I\) 

1  el+ 

(I  *2) 

V    n  +  1  e  I+ 
n 

»*3  ) 

Vs  [( 1  e  S  and  Vn  [n  e  S  =»  n  1  1  €  S])  =>  Vn  n  e  S] 

Note  that  when  (I+2  )  is  written  out  fully,    we  have  *VX  [x  e  I+  ==>   x  +   1  e  I*]1 

Also,    one  can  rewrite  Theorem   101  as   'V    ne  P'. 

n 


[7-50]  [7.04] 

EXERCISES 

A.    In  each  of  the  following  exercises  you  are  given  a  set  S.     Your  job 
is  to  prove  or  disprove: 

V    [n  €  S   =>  n  +  1  e  S] 
n 

Sample.     S  =  {m:    3  +  m  e  I  +  } 

Solution.     What  we  want  to  prove  [or  disprove]  is  the  following: 

V  [n  €  {m:    3  +  m  e  T}   =>    n  +  1  e  {m :    3  +  m  e  I+}] 
We  can  simplify  this  immediately  to: 

V  [3  +  n  €  T  =>    3  +  (n  +   1)  €  T] 

n     L  J 

In  order  to  prove  this  generalization,  we  proceed  in  the 
familiar  way  by  supposing  43  +  p  e  I+'  and  then  deriving 
43  +  (p  +  1)  eT\ 

Proof.  Suppose  that  3  +  p  e  T.     Then,  by  (I+2),    (3  +  p)  +  1  e  I+. 

But,    by  the  associative  principle  for  addition, 
(3  +  p)  +   i  =  3  +  (p  +  1).       So,    3  +  (p  +  1)  e  T. 
Therefore,   if  3  +  p  e  I+  then  3  +  (p  +  1)  e  I+. 
That  is,   if  p  e  S  then  p  +  1  e  S.     Consequently, 

V  [neS=>n+leS]. 
n  L  J 

1.  S  =  {m:  m  >    1} 

2.  S  =  {m:  m  -  5  e  I*} 

3.  S  =  {m:  m2  >  m} 

4.  S  =  {m:  m  <   4} 
*5.  S  =  {m:  m  <   0} 


B.     Prove  that  3  e  I+. 


[7.04]  [7-51] 

PROOFS  BY  MATHEMATICAL  INDUCTION 

On  page  7-47  we  stated  certain  assumptions  you  made  about  the 
positive  integers  in  an  earlier  unit.     The  first  of  these  is  Theorem  101 
which  we  have  already  proved.     The  other  six  assumptions  can  also  be 
proved  by  using  (l*^,    (I+2),    and  (I+3).     [To  prove  (6)  we  shall  need  one 
more  basic  principle.]    These  theorems  will  be  proved  in  this  and  later 
units . 

As  an  instructive  example  of  how  such  theorems  are  proved,    let's 
prove: 

(*)  V     3  +  n  e  I  + 

n 

[Notice  that  since  3  e  I+,   this  is  an  instance  of  a  theorem,  4V    V   m  +  n€  I+', 

-  m  n 

which  says  that  I+  is  closed  with  respect  to  addition.] 

To  prove  (*)  is  to  prove  that  each  positive  integer  has  the  property 
that  when  it  is  added  to  3,   the  sum  is  a  positive  integer.     This  amounts 
to  saying  that  {x:    3  +  x  e  I+}  contains  all  the  positive  integers.     In  other 
words,    (*)  is  equivalent  to: 

V     n  €  {x:     3  +  x  €  I+} 

n  l  J 

Looking  at  (I+3),   we  see  that  we  can  prove  this  if  we  can  prove: 

1  6  {x:    3  +  x  €  I+}  and  V     [n  e  {x:    3  +  x  e  T}  =>  n  +   1  e  {x:    3  +  x  e  T}] 

And,    we  can  prove  this  if  we  can  prove: 

(i)  3  +   1  e  I* 


and: 


(ii)  Vn  [3  +  n  €  I+  =>   3  +  (n  +  1)  e  I+]  [Explain.  ] 


Now,    let's  start  proving. 


[7-52] 


[7.04] 


Part  (i) : 


(1) 

V 

n 

n  +  1  e  T 

(2) 

3e  r 

(3) 

3  +  1  €  r 

Part  (ii) : 

(4) 
(5) 
(6) 


3  +  q  €  I+ 


V    n  +   1  6  I* 
n 


(3  +  q)  +   1  £  I+ 


(7)  VxVy  x+  (y  +  1)  =  (x+y)  +  1 

(8)  3  +  (q  +  1)  =  (3  +  q)  +  1 


(9)  3  +  (q  +  1)  €  I* 

(10)  3  +  q  £  I+  =>    3  +  (q  +   1)  e  I+ 


(11)  V    [3  +  n  £  I+=>   3  +  (n  +  1)  £  I+] 


Part  (iii) : 

(12)  1  £  {x:   3  +  x£  I+} 

(13)  Vn[n€  {x:    3  +  x  €  I+}  =>  n  +  1  €  {x:    3  +  x£  I+}] 

(14)  VJdeSandV   [n  £  S  =>  n  +  1  £  S])  =>   V   n  £  S] 


n 


n 


r 


(15) 


(1  €  {x:   3  +  x  £  I+}  and 
V    [n  £  {x:    3  +  x  £  I+}=>   n  +   1  £  {x:    3  +  x  €  I+}]) 


n 


(16) 
(17) 


V    n  £  {x:    3  +  x£  T} 


V    n  £  {x:    3  +  x  £  I+} 


V     3  +  n  £  I+ 
n 


J 


(i*2 )] 

theorem] 
(1),  (2)] 


assumption]* 

u +2)] 

(4),  (5)] 
theorem] 
(7)] 

(6),  (8)] 
(9);  *(4)] 
(4)  -(10)] 


(3)] 
(11)] 


(14)] 


(12), (13), (15)] 
(16)] 


[7.04] 


[7-53] 


This  proof  is  a  typical  proof  by  mathematical,  induction  [or  :    indue  - 
tive  proof].     The  basic  principle  (I+3 )    [or  some  equivalent  statement]  is 
called  the  principle  of  mathematical  induction.     As  illustrated  above, 
an  inductive  proof  of  a  generalization  about  positive  integers--that  is, 
of  a  sentence  of  the  form: 


V      F(n) 
n 


[V      3  +  n  c  T] 
1   n  J 


consists  of  three  parts. 

Part  (i)  [steps  (1)  -(3)]  is  a  proof  of  the  generalization's    "first 
instance"  : 

F(l)  [3  +  1  el+] 

[The  whole  of  part  (i)  is  sometimes  called  the  initial  step  of  the  proof.] 
Part  (ii)  [steps  (4)  -  (11)]  is  a  proof  of  a  sentence  of  the  form: 

V^  [F(n)  =>   F(n  +1)]  [V     [3  +  n  e  I+  =>    3  +  (n  +   1)  €  I+]] 


n 


n 


[The  whole  of  part  (ii)  is  sometimes  called  the  inductive  step;    and  the 
assumption  [step  (4)]  is  called  the  inductive  hypothesis.] 

Part  (iii)  [steps  (12)  -(17)],    which  is  sometimes  called  the  final  step, 
brings  in  (I+3).     This  part  is  much  the  same  in  all  inductive  proofs.     It 
consists  of  six  steps  of  the  form: 


(    ) 


1  €  {x:   F(x)} 


[ 


] 


(     )        Vn  [n  €  {x:   F(x)}=>  n  +   1  e  {x:    F(x)}] 


(    )        Vg  [(1  €  S  and  VR  [n  6  S=>  n  +  1  €  S])  =>  Vn  n  6  S]  [(I+3)] 


C  (1  €   (x:    F(x)}  and 

(     )  I    Vn  [n  €  {x:    F(x)}=>  n  +   1  e  {x:   F(x)}]) 


"> 


V. 


V    n  e  {x:   F(x)  } 
n  J 


J 


(     ) 


V    n  e  {x:    F(x)} 
n  ^  J 


[  , 


(     ) 


V    F(n) 
n 


[7-54]  [7.04] 

The  marginal  comments  for  the  first  two  steps  refer,    in  each  such 
proof,    to  the  conclusions  of  parts  (i)  and  (ii),    respectively.     The  mar- 
ginal comment  for  the  fourth  step  refers  to  the  third  step.     That  for  the 
fifth  step  refers  to  the  first,    second,   and  fourth  steps.      Finally,    the 
marginal  comment  for  the  last  step  refers  to  the  preceding  step. 

Obviously,  once  you  have  completed  parts  (i)  and  (ii)  of  an  inductive 
proof,  writing  part  (iii)  is  just  hack  work.    You  could  even  have  a  rubber 
stamp  made  which  would  print  a  form  like  the  one  above,  but  leave  blanks 
in  place  of  'F(x)'  and  'F(n)'.     Then  you  could  finish  any  inductive  proof 
by  printing  this  form  after  part  (ii),    writing  the  appropriate  sentences  in 
the  nine  blanks,    and  filling  in  the  numerals  at  the  left  and  the  marginal 
comments  on  the  right.    Instead  of  doing  even  this,    let's  agree  that  once 
we  have  completed  parts  (i)  and  (ii)  we  can  skip  the  first  five  steps  given 
in  part  (iii),    and  finish  the  proof  by  writing  a  single  step  of  the  form: 

(      )  Vn  F(n)  [       ,  ,    PMI] 

The  marginal  comment  will  refer  to  the  conclusions  of  parts  (i)  and  (ii) 
and  indicate  that  the  justification  of  this  step  makes  use  of  (I+3),   that  is, 
of  the  principle  of  mathematical  induction. 

With  this  agreement  for  shortening  part  (iii),  the  proof  of  theorem 
(*)  consists  of  twelve  steps --the  eleven  steps  which  make  up  parts  (i) 
and  (ii),    and  a  final  step: 

(12)  Vn  3  +  n  el+  [(3),   (11),   PMI] 

[We  could  also  shorten  the  proof  by  leaving  out  step  (5),    since  this  is  the 
same  as  step  (1).     Doing  so,  the  succeeding  steps  would  be  renumbered 
and  the  comment  for  the  conclusion  of  part  (ii)  would  be  4(1),  (4)  -(9)'.] 

As  usual,   paragraph  proofs  are  shorter.     Here  is  a  paragraph 
proof  for  (*)  : 

(i)    Since  3  e  I+,   it  follows  by  (I+2  )  that  3  +  1  e  I+  . 

(ii)    Suppose  that  3  +  q  e  I\     It  follows  by  (I+2)  that  (3  +  q)  +  1  6  I+ . 
But,    (3  +  q)  +   1  =  3  +  (q  +   1).     So,    3  +  (q  +  1)  €  I+.     Hence, 
Vn  [3  +  n  eI+=>   3  +  (n  +   1)  €  I4]. 

(iii)     From  (i)  and  (ii)  it  follows,  by  the  PMI,    that  V     3  +  n  e  I+. 

n 


[7.04]  [7-55] 

Before  writing  an  inductive  proof,    you  will  find  it  helpful  to  restate 
the  theorem  you  wish  to  prove  in  the  form: 

V    n  e  {x:    F(x) } 
n  *•  J 

and  to  write  the  sentences  of  the  forms: 

F(l)  and:     V    [F(n)  =>   F(n  +  1)] 

which  are  to  be  the  conclusions  of  parts  (i)  and  (ii) .     If  you  have  trouble 
in  seeing  what  these  last  two  sentences  are,    you  may  find  it  helpful  to 
write  a  sentence -form  with  a  blank  in  place  of  the  variable.     Thus,   if 
you  were  to  prove  (*): 

V    3  +  n  €  T 
n 

you  would,   before  beginning  the  proof,    write: 

V    ne  {x:    3  +  xc  T} 

n  v  J 

3  +  . . .  e  I+ 

(i)    3  +  1  e  I+  (ii)  V    [if  3  +  n  e  I*  then  3  +  (n  +  1)  €  I+] 


EXERCISES 

A_.     1.     Give  a  column  proof  of  Theorem  101  [*V   n  e  P']  by  mathematical 
induction.      [Before  stating  the  proof,    carry  out  the  suggestion 
made  in  the  preceding  paragraph.] 

2.     Write  all  six  steps  of  part  (iii)  of  the  inductive  proof  of  Theorem 
101. 


B.     1.     Give  a  column  proof  of  'V     3n  e  I+*. 

—  r  n 

2.     Give  a  paragraph  proof  of  the  same  theorem 


[7-56] 


[7.04] 


CLOSURE  OF  T  WITH  RESPECT   TO  +  AND  X 

Two  of  the  assumptions  about  positive  integers  which  you  made  in 
an  earlier  unit  amount  to  the  following  theorems: 


and: 


Theorem   102. 


V     V    m  +  n  €  I+ 

m     n 


Theorem   103. 


V      V     m  •  n  e  I+ 
m    n 


Theorem  (*)  on  page  7-51  is  the   "third  instance"   of  Theorem   102, 
and  its  proof  will  suggest  how  to  prove  Theorem   102. 

We  now  have  two  methods  for  proving  generalizations.     The  first 
method,    which  you   used  in  earlier  units,    depends  on  the  test-pattern 
principle.     To  use  this  method,    you  r.jd-ke  up  a  testing  pattern  for  the 
instances   of  the  generalization.     The  second  method  involves  the  PMI 
[and  applies   only  to  generalizations  about  positive  integers]. 

In  Unit  2  you  learned  that  you  could  often  find  a  test-pattern  for  a 
generalization  by  first  proving  an  instance  of  the  generalization.  For 
example,  in  order  to  find  a  proof  for  4V  x  +  x  =  x  •  2'  you  might  begin 
by  trying  to  prove,    say,    '3  +  3  =   3*2'.     Here  is   such  aproof: 


(1) 
(2) 
(3) 
(4) 
(5) 
(6) 
(7) 
(8) 
(9) 


H  +  12  =  Q]  +  m 


V       X-    1      : 
X 

-     X 

3    '    1      : 

-® 

a +  oi  = 

-  (II- 1  +  QJ. 

1 

x(y  +    z)     = 

-    xy  +  xz 

U(l     +     1)      : 

=  [H-i  +  GO- 

1 

EJ  +  [I]  = 

=  GOa  +  i) 

2     = 

=  i  +  i 

E]  +  ,3    -- 

=     3  -2 

[Identity] 
[pml] 
[(2)] 

[(1),  (3)] 
[idpma] 
[(5)] 

[(4),  (6)] 
[definition] 
[(7),  (8)] 


[7.04]  [7-57] 

Now,    in  order  to  obtain  a  proof  of  *V    x  +  x  =  x  •  2',   all  you  need  do 
is  replace  each  boxed  43'  by,    say,    an  'a',    and  add  one  more  step: 

(10)  V    x  +  x   =   x-2  [(1)  -  (9)] 

This  suggests  that,    in  order  to  prove  Theorem   102: 

V     V    m  +  n  €  I+  , 
m    n 

we  begin  by  trying  to  find  a  test-pattern  for  sentences  of  the  form: 

V  p  +  n  e  r 
n  r 

And,   it  suggests  that  a  proof  of  one  such  sentence,    say: 

(*)  V     3  +  n  e  I+ 

n 

may  indicate  how  to  write  such  a  test-pattern. 

Let's  look  at  the  proof  of  (*)  on  page  7-52.      This  is  an  inductive 
proof,    and  the  main  problem  was  to  prove: 

3  +  1  e  T,         and:        V     [3  +  n  e  1+  =>   3  +  (n  +  1)  e  I+  ] 

n     L  J 

What  we  now  want  to  do  is  to  get  test-patterns  ending  with: 

p  +   1  6  I+,         and:        V    [p  +  n  e  I+  =>  p  +  (n  +   1)  e  I+  ] 

For,   if  we  can  do  this,    we  can  conclude: 

(      )  Vn  p  +  n  e  T  [      ,       ,    and  PMI] 

Then,    we  shall  have  a  test-pattern  for  sentences  of  the  form: 

V  p  +  n  e  I+, 


n 

and,    by  the  test-pattern  principle,    can  complete  the  proof  of  Theorem 
102  by  writing : 

(      )  V     V    m  +  nel+  [(1)   -  (      )] 

m     n  l\    /        \       /j 


[7-58]  [7.04] 

EXERCISES 
A.     1.     Write  a  column  proof  of  Theorem  102. 
2.     Write  a  paragraph  proof  of  Theorem  102. 

vU    *t*    o„ 
"lN    *(*    *v 

If  you  followed  the  discussion  on  test-patterns,  and  looked  at  part  (i) 
of  the  proof  on  page  7-52,  your  answer  to  Exercise  1  of  Part  A  probably 
begins  like  this  : 

Part  (i): 

(1)  V    n  +  1    e  I+  [(!*)] 


n 


+  » 


(2)  p  e  T  [theorem] 

(3)  p  +  1    €   I+  [(1),    (2)] 

Parts    (i) ,    (ii),    and   (iii)    make  up  a  test-pattern  for  instances  of 

*  VL,  V_  m  +  n  €  I+  '--that  is,    for  sentences  of  the  form  'V    p  +  n  e  I+ '. 
m    n  *  n  r 

To  test  such  an  instance  we  substitute  for  'p'  a  numeral  for  some 
positive  integer.     So,    after  making  such  a  substitution,    step  (2)  will 
be  a  theorem.     We  can  somewhat  simplify  the  proof  by  taking  account 
of  our  convention  that  the  domain  of  *p'  is  I+.     Because  of  this,    'pel 
is  a  trivial  theorem  —  step  (3)  is  an  immediate  consequence  of  step  (1). 

K^      x»*     >.** 
■V     "V     'i% 

B_.     1.     Complete  the  following  proof  of  Theorem  10  3.     [Important: 
First  look  at  (5)  and  (13),    and  fill  in  (14)  and  the  marginal 
comment  for  (15).     After  you  have  seen  how  to  do  this,  com- 
plete part  (i)  and  part  (ii).] 

Part  (i) : 

(1)  V    n  e  I+  [trivial  theorem] 

(2)  P  e  I+  [(1)] 

(3)  [basic  principle] 

(4)  [       ] 

(5)  pi    e   T  [(2),    (4)] 


[7.04]  [7-59] 

Part  (ii): 

(6)  pq  e  l+  [inductive  hypothesis ]* 

(7)  V   V    x(y  +  1)  =  xy  +  x  [theorem] 

(8)  [       ] 

(9)  [  ] 

(10)  pq  +  p  e  r  [     ,  (9)] 

(11)  [  ] 

(12)  [(H);  *(6)] 

(13)  Vn  [pnel+  =>  p(n+  l)e  I+]  [       -       ] 

Part  <iii) : 

(14)  [      ,        ,   PMI] 


(15)  V     V    mn  €  I+ 

1      '  m    n 


2.     Write  a  paragraph  proof  of  Theorem  103. 

MISCELLANEOUS  EXERCISES 

1.  If  the  area  of  a  rectangular  enclosure  is  48  square  yards  and  the 
perimeter  is  28  yards,    what  are  the  length  and  the  width? 

2.  Mr.   Jones  is  24  years  older  than  Bill.       Eight  years  ago  he  was 
twice  as  old  as  Bill.     How  old  are  they  now? 

3.  A  can  do  a  certain  task  in  8  days.     If  B  works  along  with  A,   they  can 
do  it  in  4  days.     How  long  would  it  take  B  to  do  it  alone? 

4.  One  leg  of  a  right  triangle  is  7  feet  longer  than  the  other  leg.     If  the 
hypotenuse  is   13  feet  long,   what  is  the  area  of  the  triangle? 

5.  Suppose  that  the  vertices  of  the  acute  angles  of  a  right  triangle  are 
A(-5,    0)  and  B(5,    0).     Write    a    sentence   whose  graph  contains  just  the 
points  which  can  serve  as  the  third  vertex  of  this  triangle. 


[7-60]  [7.04] 

6.  The  average  velocity  of  molecules  of  air  at  room  temperature  is 
about  5  X  104  cm/sec.     About  how  many  miles  per  hour  is  this? 

7.  A  unit  commonly  used  in  measuring  atomic  particles  is  the  Angstrom 
unit.     It  is  a  unit  of  length,   and  is  defined  to  be  10~8  centimeters 

o 

long.    If  the  average  atom  is  3.  5A  in  diameter,  how  many  such  atoms 
placed  side  by  side  would  it  take  to  span  a  1 -inch  distance? 

8.  Physicists  in  the  18th  century  discovered  that  the  force  of  attraction 
between  two  particles  with  opposite  electrical  changes  is  inversely 
proportional  to  the  square  of  the  distance  between  them,   and  jointly 
proportional  to  the  magnitude  of  the  charges.     How  is  the  force  of 
attraction  between  the  charged  particles  changed  (a)  if  the  distance 
between  them  is  doubled?    (b)  if  the  charge  of  one  particle  is  doubled? 
(c)  if  the  charges  of  both  particles  are  doubled? 

9.  [A  point  Q  is  said  to  be  a  reflection  of  a  point  P  with  respect  to  a 
point  R  if  and  only  if  R  is  the  midpoint  of  PQ.  ]     Suppose  that  quad- 
rilateral ABCD  is  a  parallelogram  and  that  P    is  a  point  in  its  plane. 
Let  P2  be  the  reflection  of  Px  with  respect  to  A,   P3  be  the  reflection 
of  P2  with  respect  to  B,   P4  be  the  reflection  of  P3  with  respect  to  C, 
and  P5  be  the  reflection  of  P4  with  respect  to  D.    Prove  that  P5  =  Px  , 

10.  (a)     If  V    f(p)  =   *P  +  1  then  V    f(p  +  1)  =  and  V    f(p  -  1)  = 

P  ^P  "  3  P  P 

(b)  If  Vpf(p)  =  p(p-l)  then  Vpf(p  +  1)  =  andV    f(p-l)  = 

(c)  If  V    g(n)  =  n(n  +  l)(n  +  2)  then  V    g(n  +  1)  =         and  V    g(n  -  1)  = 

n  n  n 

(d)  If  V    g(p)  =  p2  -  (p  -  l)2  then  V    g(p  +  1)  =  and  V    g(p  -  1)  = 

(e)  If  V    S(n)  =  n2  then  V    S(n  +  1)  =  and  V    S(2n  +  1)  = 

n  n  n 

11.  A  student  looked  at  the  statement  *2X(3  +  4)  =  2  +  (3X4)'    and 
thought  it  was  false  because  it  looked  as  if  someone  were  trying  to 
use  a  commutative  principle  on  the  multiplication  and  addition  signs. 
But,    of  course,   the  statement  is  true.     Find  other  integers  x,    y, 
and  z  such  that  x  X  (y  +  z)  =  x  +  (y  X  z). 


[7.04]  [7-61] 

RECURSIVE  DEFINITIONS 

One  of  the  mathematical  pastimes  of  the  ancient  Greeks  was  the 
discovery  of  interesting  generalizations  concerning  whole  numbers. 
For  example,    consider  the  following  diagram: 


V 

V 

•\# 

A  • 

.  ,\. 

\ 

\ 

\* 

V 

•v 

••V 

•        •        •     y  • 

•    •     •  *  • 

*      •       •      •  v  • 

\ 

If  we  count  the  dots  in  each  of  these  "triangles"  and  in  those  we  would 
get  by  continuing  the  obvious  construction,    we  obtain  a  sequence  of  num- 
bers which  begins 

1,       3,       6,        10,       15,       21,       28,    ...    . 

The  Greeks  called  these  whole  numbers  triangular  numbers.      For  our 
purposes  it  will  be  more  convenient  to  regard  the  corresponding  positive 
integers  as  triangular  numbers. 

Suppose  that  we  use  'T    '  to  name  the  first  triangular  number.     Then 
T.    =   1.     Also,    T0  =  3,    T,  =  6,    T,  =   10,    and  T     =   15.     What  is  T,  ?    T_  ? 

1  '         2  '         3  '         4  5  6  7 

Suppose  you  are  told  that  T       is  55.      Can  you  use  this  information 
to  find  TX1  ? 

Given  that  T25  =  325,    find  T26. 

As  you  see, 

r  T1   =    1,    and 

(1)  IV        T     ,       =    T     +  (n  +   1). 

V      n         n  +  l  n 

These  formulas   give  a  way  of  computing  triangular  numbers.     For  example, 

T6   =  T5  +  6 

=  (T4  +  5)  +  6 

=  (T3  +  4)  +  5  +  6 

=  (T2  +  3)  +  4  +  5  +  6 

=  (T1  +  2)  +  3  +  4  +  5  +  6 

=  1  +  2+3  +  4+5  +  6  =  21. 


[7-62] 


[7.04] 


The  process   of  using  T5  to  compute  Tfe,    T4  to  compute  T5, ,    and  T 

to  compute  T    is  an  example  of  recursion.    Use  recursion  to  compute  T6. 

Evidently,    (1)   gives  us  a  way  of  computing  any  triangular  number. 
That  is,    (1)  gives  us  a  procedure  for  computing  the  successive  values  of 
a  function,    T,    whose  domain  is  the  set  of  positive  integers    and  whose 
range  is  the  set  of  triangular  numbers. 

Some   of  the  ordered  pairs  in  T  are  (1,    1),    (2,    3),    (3,    6),    and  (4,    10). 
What  is  the  ordered  pair  in  T  with  8  as  first  component?     With   9  as  first 
component?     [Notice  that  in  the  case  of  a  function  like  T  whose  domain  is 
1+,    it  is  customary  to  modify  the  usual  functional  notation  and  write,    say, 
4T10'  instead  of  'T(IO)'  .] 

The  pair  of  formulas  (1)  is  called  a  recursive  definition  of  the  func- 
tion T.     Here  are  recursive  definitions   of  two  other  functions: 


Vn+l=(n+1>fn 


rl  =   1 


V    r  =   |r     + 

n     n+  l       \  n       r 


n 


/2 


Compute  the  first  four  values  of  each  function.     Complete 


V    f 


n    n  +  2 


-<        >fn+1  =  <        ><        >£n 


In  working  with  functions  in  earlier  units,    you  became  accustomed 
to  a  different  way  of  defining  functions  -  -explicit  definition.     For  example, 
consider  the  sequence,    S,    of  square  numbers. 


•      • 


•      •      • 


•      •      •       • 


•      •      •      • 


An  explicit  definition  of  S  is 


n 


S     =  n2 
n 


With  this  kind  of  definition,    you  can  find  any  value  of  S  directly  without 
previously  computing  other  values. 

Can  we  discover  an  explicit  definition  for  T?     You  might  want  to 
play  with  this  problem  before  reading  further. 


[7.04] 


[7-63] 


Let's  reconsider  X.     The  following  figure  shows  that  T6  +  Tfe  =  6*7, 


and  suggests  that 
(2) 


\ 


V   T     =   n(n  +  *) 
n    n  2 


We  can  check  this  guess  against  the  recursive  definition  by  noting  that 

1(1  +  1)    =  j 


and  that,    for  each  n, 

(n  +   l)[(n  +!)+!]    =    n(n  +   1)    +  (n  +  x) # 

[Verify  this  last  generalization  by  transforming  the  right  side  of  the 
equation  into  the  left.  ] 

This  checking  procedure  shows  that  there  is  a  function- -namely, 
the  one  defined  by  (2) --which  satisfies  the  recursive  definition.     What 
we  want  to  do  now  is  to  show  that  there  is  only  one  such  function.     This 
we  can  accomplish  by  deriving  (2)  from  (1).      Let's  do  so. 


Our  job  is  to  prove: 


V     T         n(n  +  1) 


n      n 


We  shall  use  the  principle  of  mathematical  induction  to  prove  this 
generalization  about  positive  integers.     Restating  the  generalization, 
we  have : 

w        ,   (  -r  m(m  +   1)  ■» 

V     ne   {m:    T       =    — ■ = -} 

n  m  2 

As   a  help  in  formulating  the  proof  we  write: 

...(...    +   1) 


T 


<i)  Ti  =  Ki  +  D 


(ii)     V 


n 


T    =    n(n  +    *)    ==>    T 

2  n+  l 


_    (n-H)[(n-H)+l] 


n 


[7-64] 
Part  (i) 


(1) 

T1=  1 

(2) 

Kl  +  1)    .  , 
2 

(3) 

*    _    Id  +  1) 

Part  (ii): 

(4) 
(5) 
(6) 
(7) 


T     = 


V       T    , 
n        n+  l 


q(q+  1) 
^ 


T    +  (n  +  1) 
n 


T     L      =  T     +  (q  +  1) 
q+  l  q       VM  ' 


W-aafiUu  +  i) 


(8)    V     i<i  +  i>  +  (x  +  1)    =    <*+'>[<*+l>  +  l] 


{9)      ais^Ll)  +  (q  +  i)  =  la  +  m  +ILLU 


(10) 


_    (q  +   l)[(q  +   1)  +   1] 
q  +  1  "  2 


(in      t  =  q(q  +  l)  =>  t        =  (q  j  D[(q  UJLIJJ 

7            q               2  q+  l                             2 

(12)  V  (t     =   n(n  +  *>   =>  T            =  (n  +   l)Hn  +  l)  +   Ll 

n     n               2  n+  l                            2 


[7.04] 


[recursive 
definition] 

[theorem] 
[(1),  (2)] 


[inductive 
hypothesis]* 

[recursive 
definition] 


[(5)] 

[(4),   (6)] 
[theorem] 

[(8)] 

[(7),   (9)] 
[(10);    *<4)] 
[(4)  -(H)] 


Part  (iii) 
(13) 


V   T     =   n(n  +  *) 


n    n 


[(3),(12),PMI] 


In  the  future  we  shall  omit  steps  like  (5)  and  label  steps  like  (6)  '[recur- 
sive definition]'. 


[7.04] 


[7-65] 


In  practice,    one  could  replace  steps  (8),    (9),    and  (10)  by 


q(q 

+  1)  +  2 

(q 

+ 

1) 

2 

<q  + 

D[(q  + 

1) 

+ 

1] 

/    [algebra] 


You  might  want  to  put  in  more  manipulation  steps  than  these  to  convince 
a  reader  that  you  knew  what  you  were  doing. 
Here  is  a  paragraph  proof  of  (2): 


(i)     Since,    by  the  recursive  definition  of  T,    T     =   1,    and  since 

1(1  +  1)    _   ,       T  Kl  +  1) 

2  '        l  2 


(ii)  Suppose  that  T     =  ? -.      Since,    by  the  recursive  defi- 

nition,    T  =  T     +  (q  +  1),    it  follows  that  T     ,       =    q^q  *   l'    +  (q+  1). 

q+  l  q  q+  l  2  M        ' 

But,  stfL+Ji  +  (q  +  i)  =  aia +  U  +  2'q +  U  -  <q +  ^ +  z> .  s».  if 


T     =    q(q  +   1)    then  T  =    (q+l>[(q+l)    +   l] 


q+1 


Consequently, 


w    fT     _    n(n  +  1)  T  _    (n  +   l)[(n  +  1)  +   1]"| 

n  pi  '  2  n+i  2  J  ' 


(iii)  From  (i)  and  (ii)  it  follows,    by  the  PMI,    that   V^  T^  =    "^  ^ 


n     n 


Here  is  a  tree-diagram  of  the  proof  on  page  7-64. 

(5) 
(8)  (4)  (6) 

(9) 


(7) 


(1)  (2) 

(3) 


(10) 

(11) 

(12)         [PMI] 


(13) 


It  shows  clearly  that  (13)  is  a  consequence  of  the  theorems  (2)  and  (8), 
the  recursive  definition  consisting  of  (1)  and  (5),    and  the  PMI. 


[7-66] 


[7.04] 


Use  either  column  proofs  or  paragraph  proofs  for  the  following 
exercises . 

EXERCISES 

A.     1.      Consider  the  following  recursive  definition  of  a  function  E  whose 
domain  is  I+: 

E1  =  2 

V       E  =  E     +  2 

n         n  +  i  n 

(a)    Compute    Ep,     E    ,     E    ,     and   E    . 


(b)    Complete  the  following 

V       E 
n         i 

to  obtain  an  explicit  definition  of  E. 


V       E     = 
n        n 


(c)    By  mathematical  induction,    derive  the  explicit  definition  of 
E  from  its  recursive  definition. 

2.      Give  a  recursive  definition  for  the  function  O  whose  values  are 
the  successive  odd  positive  integers,    and  repeat  Exercise   1, 
using  your  recursive  definition  of  O  in  place  of  the  recursive 
definition  of  E. 


3.     Here  is   a  recursive  definition  of  a  function  S 

s1=  1 


V      S     ,       =  S     +  (2n  +   1) 

n       n  +  l  n 

Repeat  Exercise   1  for  the  function  S. 


B_.     1.      Consider  the  sequence   of  triangular  numbers.     Notice  that  the 


10         15        21         28 


T         T 

tP      /p  +  i 


4  9         16  25        36 

sum  of  each  two  consecutive  triangular  numbers  appears  to  be 


[7.04]  [7-67] 

a  square  number.     This  suggests,    as  a  theorem,    a  generalization 
which  begins  : 

V        S     ,       = 
n        n+  1 

Complete  the  generalization,    and  prove  it  by  mathematical  induction. 

2.       The  explicit  definition  of  S  which  you  obtained  in  Exercise  3  of  Part 
A  is: 

V        S     =  n2 
n         n 

(a)  Use  this  definition  to  complete  the  following  generalization: 

(*)  V       S       ,       = 

v    '  n         2n  +  l 

(b)  Use  this  result  and  the  explicit  definition  of  the  triangular 
numbers  to  discover  a  relation  between  the  odd  square  numbers 
and  the  triangular  numbers.       [Hint.     Transforming  the  right 
side  of  (*)  may  help.]      Express  your  discovery  as  a  generali- 
zation beginning  with: 

V        S        ,       = 
n         2n  +  l 

(c)  Give  a  second  proof- -this  time  by  mathematical  induction- -of 
the  generalization  you  discovered  in  (b). 


3. 


Consider  these  three 

se 

qu 

ences 

: 

1 

2 

3 

4 

P 

1 

3 

6 

10  .  .  . 

T 
P 

1 

4 

9 

16  ... 

S 
P 

Discover  how  to  find  a  term  of  the  second  sequence  from  the 
corresponding  terms  of  the  first  and  third  sequences.     Express 
this  discovery  in  a  generalization  and  prove  it  by  mathematical 
induction. 

4.       Prove  that,    for  each  n,    the  nth  even  positive  integer  is  the  average  . 
of  the   1st  and  the  2nth  odd  positive  integers. 


[7-68]  t7-04] 

C.     1.     Here  is  a  recursive  definition  of  a  function  f  whose  domain  is  I+: 

~~*  fn 

f1  =  3,    and   Vnfn+1   =   y^ 

Compute  the  sum  of  the  "first  ten"  values  of  f . 

2.  If  r±  =  3,    and  Vp  rp  +  ±  =  rp  +    ^p,    then  compute  r  1Q. 

3.  If   f3  =  7,    and  Vn   £n+i  =    fn  +  5  then   i±  =        . 

4.  If   i1  =  9,    and  Vm  f m  +  x  =    fm  +   10,    then   f  1T  -  f  16  =        . 

5.  If  gj.  =  4,   and  vn  gn  +  j_  =  §n  +  n>   then  §5  =         and  §9  "  §8  = 

6.  If  c,   =  1,    and  V     c     ,      =  c     +  (3n2  +  3n  +   1),    then  c5  = 

1  n     n  +  l         n  3 

7.  If  gl  =   1,    and  Vn>  ±  gn  =  gn_  ±  +  (2n+   1),   then  g4  =        . 


8.     If    f ,   =  2,    and  V      f       ,       =  f      -2,    then  f 
1  m    m  +  l         m 


lo 


9.     If   t1=   i,andVn>3tn_3=   tn_2-i,    then  1 10  =        . 


10.  If   f,    =   1,    and  V    f     ,       =   f     •  —rr>    then  f3  =  and   f, 

1  nn+i         nn+1  3  x 

11.  If  f,    =   -p,  and  V    f  =  f    •       "  ,    ,  then  f4  = 

1        1  nn+i         nn+1  4 


and  f 

1 


12.     Suppose  that  k1  =   1,    and  that  for  each  n,   k  -  k     +   — ^ 

(a)  Find  the  smallest  m  such  that  k       >    1.11. 

in 

(b)  Find  the  smallest  m  such  that  k       >    -^r-. 

m  9 


D.    Here  is  a  recursive  definition  for  a  function  A  -whose  domain  is  the 
set  of  nonnegative  integers. 

AQ  =   100 


V      A     =   1.03A 
n        n  n  -  i 


1.     Compute  A    . 


2.     How  much  does  $100  amount  to  if  it  is  deposited  in  a  savings  bank 
for  4  years  at  3%  interest  compounded  annually? 

^3.     Guess  an  explicit  definition  for  A. 


[7.04] 


[7-69] 


E.    Graph  each  function.     [Plot  at  least  5  ordered  pairs.] 

Sample,     f .  =  3,    and  V    f     ,       =  f     +  (5n  +  1) 
v    ■         i  n    n  +  l        n 

f 
Solution, 


n 
100  - 

80  - 

60 
40  - 

20  - 


fl   =    3'    f2   =   9»     f3  =    20 


f4  =  36,   f5  =  57,   f6  =  83 


[Note  that  the  graph  is 
a  set  of  discrete  dots.] 


n 


fi  =  ° 

V     f  =  f     +  1 

n    n  +  l        n 


fi  =  2 

V    f     .      =  f     +4 

n    n  +  l        n 


fi  =  ° 

V     f     ,       =  f     +  n 
n    n  +  i        n 


fi  =   l 

V    f     ,       =  f     +  n 

n    n+  l         n 


5. 


fi=   1 


n    n  +  l        n 


6. 


£!=    1 


n 


V    f     ,      =  f 
nn+i        n     n+1 


7.  r  £,  =  2 

v  f       =  l^lli4 

n    n  +  i  2f 


n 


i1  =  2 

V    f     ,  ,  =  2f 

n    n+  i  n 


F.     1.     Each  function  defined  explicitly  below  has  I+  as  domain.     Give  a 

recursive  definition  for  each  and  then  use  mathematical  induction 
to  derive  the  given  explicit  definition  from  the  recursive  definition. 


(a)    f     =  5n  +  1 
n 


(b)    f 


n 


-3n  +  1 


(c)    f     =  3n2  +  1 
n 


2.     The  oblong  numbers  are  defined  recursively  by 


B1  =  Z 

V„      Bn+1=Bn+2<n+1> 


Use  this  and  the  recursive  definition  of  the  triangular  numbers 

as  the  basis  of  an  inductive  proof  of  the  theorem:    V      B      =  2T_ 

r  n        n  n 


[7-70] 


[7.04] 


G.     1.     Besides  triangular  and  square  numbers,   the  Greeks  studied 
pentagonal  numbers,    as  well  as  polygonal  numbers  of  still 
higher  orders.       Here  are  pictures  showing  how  triangular, 
square,    and  pentagonal  numbers  are  generated  geometrically. 
Study  the  pictures  and  the  recursive  definitions  of  triangular 
and  square  numbers,   and  figure  out  a  recursive  definition  for 
pentagonal  numbers,   Pn. 


•         « 


V 


y 


\ 


'  \*  \#  \" 


r 


•    •    •!  • 

7  n  •  i  • 


pl  = 


V       P  =  P     + 

^      n        n  +  i  n 


2.     Use  mathematical  induction  to  prove: 


V        P  =  T    +  S 

n        n+i         n         n+i 


3.     Notice  that  the  explicit  definitions  for  T  and  S  can  be  written: 


T     =   _n(n  +  *) 


V      S     =   n(2n^+  °> 
n      n  "  2 


n      n  2  n      n 

Guess  an  explicit  definition  for  the  function  P  of  Exercise  1, 
and  use  mathematical  induction  to  derive  it  from  the  recursive 
definition  of  Exercise  1. 


4.     The  polygonal  numbers  of  order  3  are  the  triangular  numbers, 
those  of  order  4  are  the  square  numbers,   those  of  order  5  are 
the  pentagonal  numbers,    etc.     Let's  use  'P'3''  as  a  new  name 
for  T,    4p(4)'  for  S,    lP(5^'  for  P,    etc.     So,   for  example,    P(^ 
is  the  fourth  hexagonal  number.     From  your  work  on  Exercise  1 
you  should  see  how  to  write  a  pattern  for  recursive  definitions 
">f  polygonal  number  functions  of  all  orders. 

_,(m+2)  _ 


v        p(m+2)  =  p(m+2) 


n 


n+  l 


n 


[7.04]  [7-71] 

^5.     Guess  a  pattern  for  an  explicit  definition  of  polygonal  number 
functions  of  all  orders: 

V       p(m+2)  = 
n  n 

and  derive  it  from  the  recursive  definition  you  found  in  solving 

Exercise  4. 

H.    1.     If,   in  a  certain  football  league,    each  two  teams  play  just  one 

game  with  each  other,   how  many  league  games  are  played?     For 
example,  how  many  games  are  played  if  the  league  has  4  mem- 
bers?    5  members?     1  member? 

2.  Suppose  that,  for  each  n,  G  is  the  number  of  games  played  by 
an  n-teamed  league  in  which  each  two  teams  play  together  just 
once.  Find  a  recursive  definition  for  G.  [What  is  Gx?  How 
many  additional  league  games  must  be  played  when  a  new  team 
joins  a  league  which  formerly  had  n  members?] 

3.  Use  the  recursive  definition  to  compute  some  values  of  G.     Guess 

an  explicit  definition  for  G: 

V       G     = 
n         n 

and  prove  your  guess  correct  by  deriving  this  explicit  definition 

from  the  recursive  definition. 


v»,      v"^      o^ 

«V    *is    <v 


In  solving  Part  H,   above,    you  found  the  number  of  ways  in  which  2 
teams  can  be  chosen  from  a  league  with  n  members.     More  generally, 
you  learned  how  to  find  out,   for  any  n  €  I+,   how  many  2-membered  sub- 
sets an  n-membered  set  has.     It  is  customary  to  abbreviate  'the  number 
of  2-membered  subsets  of  an  n-membered  set'  by  'C(n,    2)'    [or,    some- 
times, by  'nC2'].     So,  the  result  of  Exercise  3  of  Part  H  could  be  written: 

Vn    C(n,    2)  =  n(ly   " 

In  the  next  exercise  you  will  work  on  the  problem  of  finding  the  number, 
C(n,    3),    of  3-membered  subsets  of  an  n-membered  set. 

vO       si,        si. 


[7-72]  [7.04] 

I.      Mark  6  points  A,  B,  C,  D,  E,  and  F,    no  3  of  which  are  collinear.     How 
many  triangles  are  there  each  of  whose  vertices  is  one  of  these  six 
points?     [This  is  C(6,    3).     Why?]    One  way  of  attacking  this  problem 
is  to  ask  how  many  of  the  triangles  have  A  as  one  vertex  and  how 
many  do  not.       The  sum  of  these  two  numbers  is  C(6,    3).       [Why?] 

Each  triangle  which  has  A  as  one  vertex  corresponds  with  some 
2-membered  subset  of  the  5-membered  set   {B,    C,    D,    E,   F}.      So, 
the  number  of  such  triangles  is  C(5,    2)--that  is,    10. 

The  number  of  triangles  which  do  not  have  A  as  vertex  is  the 
number  of  3-membered  subsets  of  {B,    C,    D,    E,   F}.       [Explain.] 
This  number  is  C(5,    3).       So, 

C(6,    3)  =  C(5,    3)  +  C(5,    2) 
=  C(5,    3)  +  10. 

Continue  this  recursive  procedure  until  you  have  computed 
C(6,    3). 


*!*      «^      «J. 

*-,>.         *,•«.         ^.v 


Now,    let's  find  a  recursive  procedure  for  computing  values  of 
'C(n,  p)'.      Suppose  that  S  is  a  set  with  n  members,    and  that  eQ  is  some 
particular  member  of  S.      Then  S  =  SQ  w  {eQ},    where  S0  is  a  particular 
(n  -  l)-membered  subset  of  S   [in  fact,    SQ  =    {eQ}]. 

Now,    SQ  has  C(n-  1,   p)  p-membered  subsets,    and  each  of  these  is  a 
p-membered  subset  of  S. 

What  other  p-membered  subsets  does  S  have?      Obviously,    each  of 
the  other  p-membered  subsets  of  S  contains  eQ  and,   in  addition,    contains 
p  -  1  members  of  SQ.       Since  SQ  has  n  -   1  members,    S  has  exactly 
C(n  -  1,   p  -  1)  p-membered  subsets  which  contain  eQ. 

So,   the  total  number  of  p-membered  subsets  of  S  is 


Hence, 


C(n-  1,    p)  +  C(n  -  1,    p  -  1). 


Vn  Vp     C(n,  p)  =  C(n-  1,    p)  +  C(n  -  1,   p-  1). 


j^    -j-    o- 


[7.04] 


[7-73] 


J.     Here  is  a  table  for  values  of  'C(n,  p)\       The  entry  in  the  6-row  and 


\  p 

0 

1 

2 

3 

4 

5 

6 

7 

8 

9 

0 

1 

2 

3 

4 

5 

6 

20 

7 

8 

9 

3-column  is  C(6,    3)  which,   as  you  discovered  in  Part  E,   is  20. 

Note  that,    for  the  table,   we  have  allowed  0  as  a  value  of  4n*  and 
*p'.    What  does  4C(5,    0)'  mean?      What  is  the  proper  entry  for  the 
5-row  and  0 -column? 

What  does  'C(0,    5)'  mean?      What  is  the  proper  entry  for  the 
0-row  and  5-column? 

What  is  the  proper  entry  for  the  0-row  and  0-column? 

The  recursion  formula: 

V    V       C(n,  p)    =    C(n  -  1,    p)  +  C(n  -  1,    p  -  1) 
n     p 

should  help  us  to  fill  in  the  table.     Try  to  use  it  to  fill  in  the  entries 

in  the  6-row.     In  that  case,  the  instance  of  the  formula  is: 

V       C(6,  p)    =    C(5,  p)  +  C(5,   p-  1) 

So,   to  find  an  entry  in  the  6-row,    you  need  entries  in  the  5-row. 


To  find  these,    you  need  entries  in  the  4-row.     Etc 
Complete  the  table. 


[7-74] 


[7.04] 


K.    Let's  recall  problem  VII  of  the  Introduction,    page  7-2: 

Whatever  route  Milton  takes  in  going  from  his  home  to  Zabranch' 
burg  High  School,  he  has  to  walk  at  least  nine  blocks.    How  many 
routes  are  there  which  are  just  nine  blocks  long? 


.Home 


ZHS 


r 

—     — 

1 

L 

p 

1         1 

r 

1 

i 

i 

r 

—  - 

_i 

"1 

N 


S 


-E 


During  each  of  his  9 -block  walks  to  school,    Milton  walks  south  on 
each  of  four  blocks  and  west  on  five.     He  has  a  special  page  of  his 
notebook  on  which  he  keeps  track  of  the  routes  he  takes.     The  page 
is  ruled  into  9  columns  headed  '1st  block',    '2nd  block',    etc.,    and 
his  record  of  the  route  pictured  above  looks  like  this  : 

s  1st  ^    2d  b    3d    ^  4th  ^    5th  ^  6th    >  7th  >  8th  )  9th 


W 


I 


w 


S 


w 


y 


w 


i 


I     I 


w 


s 


1.     (a)    Fill  in  the  next  line  in  Milton's  table  by  writing  4S's  in  four 
of  the  columns  [and  'W's  in  the  remaining  five]. 

(b)    Mark  the  route  you  have  described  on  the  figure. 


2.     (a)    In  how  many  ways  could  you  have  answered  Exercise   1(a)? 
(b)   How  many  9-block  routes  can  Milton  take? 


[7.04]  [7-75] 

^3.     How  many  10-biock  routes  can  Milton  take  from  his  home  to  Z.H.S.  ? 

"^4.     How  many  11 -block  routes  can  Milton  take  from  his  home  to  Z.H.S.  ? 

*5.     How  many  11 -block  routes  are  there  in  which  no  block  is  walked 
over  more  than  once? 


L.    Here  is  another  way  to  solve  Milton's  problem.     During  each  of  Mil- 
ton's 9-block  walks  to  school  he  has  to  walk  south  for  one  block  at 
each  of  4  different  times.     He  distributes  these  4  times  of  southerly 
walking  among  6  north-south  streets.     [For  the  route  marked  in  the 
figure  for  Part  G,   none  of  his  southerly  walks  took  place  on  the  1st 
north-south  street,    1  took  place  on  the  2nd,   none  on  the  3rd,    1  on  the 
4th,    2  on  the  5th,    and  none  on  the  6th.  ]    So,    Milton's  problem  can  be 
solved  by  finding  the  number  of  ways  in  which  4  things  of  the  same 
kind  can  be  put  into  6  boxes.     Exercise    1  will  help  you  do  this. 

1.  Find  a  formula  for  the  number  of  ways  of  distributing  p  things  of 
the  same  kind  in  n  boxes.       [Hint.     Think  of  the  p  things  as  laid 
out  in  a  row  on  your  desk.    Then,  putting  them  in  n  boxes  amounts 
to  erecting  n-  1  partitions.] 

2.  Use  your  answer  for  Exercise  1  to  solve  Milton's  problem. 

3.  In  how  many  ways  can  you  distribute  six  pennies  among  four 
pockets    [right  and  left  coat  pockets,    right  and  left  trouser 
pockets]? 

4.  In  how  many  ways  can  you  distribute  six  pennies  among  four 
pockets  so  that  there  is  at  least  one  penny  in  each  pocket? 

5.  How  many  selections  of  six  coins  can  be  made  if  each  coin  is 
either  a  penny,   a  nickel,    a  dime,    or  a  quarter? 

6.  How  many  selections  of  six  coins  can  be  made  if  each  coin  is 
either  a  penny,   a  nickel,    a  dime,    or  a  quarter,   and  each  selec- 
tion contains  at  least  one  coin  of  each  kind? 


[7-76]  [7.04] 

^THE  ARITHMETIC  OF  THE  POSITIVE  INTEGERS 

In  proving  Theorem  102  we  made  use  of  (1*^,   (I+2),    (I+3)  and  a 
theorem  about  real  numbers: 

V     V    x  +  (y  +   1)   =  (x  +  y)  +  1 
x     y  ' 

which  is  a  consequence  of  the  apa.  [We  used  this  same  theorem  when 
deriving  the  addition  table  in  section  7.01.]  Since  we  are  dealing  only 
with  positive  integers,   we  could  have  used  a  special  case: 

(I*   )  V     V    m  +  (n  +  1)  =  (m  +  n)  +  1 

\    4/  m    n 

of  this  theorem.     If  we  were  unacquainted  with  the  other  real  numbers, 
and  with  our  basic  principles  for  real  numbers,    we  might  adopt  (I+4)  as 
a  recursive  definition  of  addition  of  positive  integers.     In  fact,   using 
(I+  )  -(I+4),    alone,  it  is  possible  to  prove  that  addition  of  positive  integers 
is  commutative  and  associative. 

Similarly,    in  proving  Theorem   103  on  page  7-58  we  made  use  of 
Theorem  102,    (1^),    (I+2),    U+3)>    the  pml,  and  a  consequence  [step  (7)] 
of  the  idpma  and  the  pml.      [We  used  the  last  two  when  deriving  the 
multiplication  table  in  section  7.01.]    Again,    since  we  are  dealing  only 
with  positive  integers,    we  could  have  replaced  the  last  two  by: 


(i*.) 


Vm  m  1  =  m 


V     V    m(n  +  1)  =  mn  +  m 
m    n 


If  we  were  to  disregard  our  basic  principles  for  real  numbers,    we  might 
adopt  (I+5  )  as  a  recursive  definition  of  multiplication  of  positive  integers. 
Using  {l\)  -  U+5)»    alone,   we  could  prove  that  multiplication  of  positive 
integers  is  commutative  and  associative,    and  is  distributive  with  respect 
to  addition.     In  fact  the  whole  arithmetic  of  the  positive  integers  can  be 
derived  from  (1^ )  -  (I+5 )  together  with  one  other  pair  of  principles: 

V         m  +  1  i  1 
m 

V     V    if  m  +  1  =  n  +  1  then  m  =  n 
m    n 


[7.04] 


[7-77] 


^EXERCISES 
In  answering  the  following  exercises,   use: 


(I+2) 
(I+3) 

tf+4> 


l  el+ 


V    n  +  1  €  r 

n 


Vs  [(1  €  S  and  V    [n  €  S   =>  n  +  1  e  S])  then  Vr  n  €  S] 

V      V      m  +  (n  +   1)  =  (m  +  n)  +  1 
m     n 


and  the  logical  principle  'V      m  +  1  =  m  +  1'.     Of  course,    you  may  also 
use  Theorem  102    [Why?]. 


n 


^1.     Complete  the  following  proof  of  'V^   1  +  n 
Part  (i): 

(1) 
(2) 
(3) 


=  n  +  1\ 


1+1  =  1  +  1 


Part  (ii) 
(4) 
(5) 
(6) 
(7) 
(8) 
(9) 


(10)     V    [l+n  =  n+l=>    1  +  (n+  1)  =  (n  +  1)  +  1] 


1  +  (p  +  1)  =  (1  +  p)  +  1 


Part  (iii) 
(ID 


logical  principle] 

] 
.    1 

inductive  hypothesis] 

] 
] 

(5),   (6)] 
(4),    (7)] 

(9)] 


] 


^2.     Complete  the  following  proof  of  'V     V    V     m  +  (n  +  p)  =  (m  +  n)  +  p*. 

t-  t>r  mnp 

[Hint.     Up  to  the  last  step,   the  proof  is  a  test-pattern  for  sentences 
of  the  form  4V     in  +  (n  +  p)  =  (m  +  n)  +  p' .     In  completing  the  proof, 


[7-78] 


[7.04] 


you  will  find  it  helpful  to  look,   first,    at  (2)  and  (14).     Then  fill  in  (15) 
and  (16).     Look  at  ( 14)  again.     Then,   fill  in  ( 1  3)  and  ( 12),    except  for  the 
comment  on  (12).     Finally,    complete  part  (i),    and  part  (ii). 


Part  (i): 

(1) 
(2) 


m  +  (n  +  1)    =   (m  +  n)  +  1 


[U+J] 
[       1 


Part  (ii) 


m  +  (n  +  q)    =   (m  +  n)  +  q 


(3) 
(4) 

(5)  n  +  q  e    I+ 

(6)  m  +  ([n  +  q]  +   1)    =    [m  +  (n  +  q)]  +   1 
(7) 

(8)        m  +  (n  +  [q  +   1])    =    [m  +  (n  +  q)]  +   1 

(9) 
(10) 

(11)        (m  +  n)  +  (q  +   1)    =    [(m  +  n)  +  q]  +   1 
(12) 

(13) 


} 


[(12);    *      ] 


(14) 


V     [m  +  (n  +  p)    =  (m  +  n)  +  p  => 

m  +  (n  +  [p  +   1])  =  (m  +  n)  +  [p  +   l]] 


[(1), 


Part  (iii)  : 
(15) 


[       . 


(16) 


[(1)  -  (15)] 


[7.04] 


[7-79] 


3.  Complete  the  following  proof  of  4V  V  m  +  n  =  n  +  m'.  [Hint.  The 
proof  is  a  straightforward  inductive  proof.  First,  fill  in  (1).  Then, 
fill  in  (17),    (16),   (15),   (14),  and  (13),  except  for  the  comment  on  (13).] 


Part  (i): 


(1) 


[theorem] 


Part  (ii) 

(2) 
(3) 
(4) 
(5) 
(6) 
(7) 
(8) 
(9) 
(10) 

(11) 
(12) 
(13) 
(14) 
(15) 
(16) 


V    q  +  n  =  n  +  q 

n  n  n 

q  +  p  =  p  +  q 


q  +  (l  +  p)  =(q+  I)  +p 
1  +  p  =  p  +  1 


[         ] 
[theorem] 

[         ] 

I       •         1 
[         ] 
I      .         ) 


[(4) 


[ 
[(4) 


(5)] 


1 


(5)] 

[       .         ] 
[(1)  -(13)] 
[(14);  *      ] 
[(1)  -(15)1 


Part  (iii) 
(17) 


[7-80]  [7.04] 

MISCELLANEOUS  EXERCISES 

1 .  V    V    x6  •  y6   = 

x     y  '  

(A)     (xy)6  (B)     xy12  (C)     (xy)12  (D)     (xy)36 

2.  If  it  takes  x  minutes  to  walk  along  two  sides  of  a  rectangular  field 
30  feet  by  40  feet,    how  many  minutes  can  be  saved  by  walking 
diagonally  across  the  field? 

3.  Find  two  positive  integers  whose  sum  is  less  than  29  and  whose 
difference  is  less  than  7. 

4.  Solve  this  system  of  equations. 

3a  -  4b  +  1   =  0 

a  +  2b  -  3  =  0 

5.  Ten  pints  of  a  certain  liquid  contain    70%   disinfectant.       The  rest 
is  water.     If  2  quarts  of  this  liquid  are  drawn  off  and  replaced  by 

3  pints  of  disinfectant,    what  is  the  resulting  ratio  of  disinfectant  to 
total  mixture? 

6.  Simplify. 

(a)    Sx3--^  (b)    ab'-J^  (c)     (a  +  b)  •  -y^-- 

lbx*  a  b  a     -  b 

4    /IT 

7.  Solve  the  equation      /—  =  b      for  'x'. 

8.  A  grocer  mixes  coffee  worth  88  cents  a  pound  with  coffee  worth  6  8 
cents  a  pound  to  make  20  pounds  of  blend  worth  75  cents  a  pound. 
How  many  pounds  of  the  88  cent  coffee  should  he  use? 

9.  A  cylindrical  bar  made  of  a  certain  metal  is  5  feet  long  and  2  inches 
in  radius.  Another  bar  of  the  same  metal  is  4  feet  long  but  3  inches 
in  radius.  If  the  longer  bar  weighs  50  pounds,  what  is  the  weight  of 
the  shorter  one? 


[7.04]  [7-81] 

10.  If  A  varies  inversely  as  the  square  of  B,    and  A  =  1  when  B  =  9,   then 
B  =  when  A  =  4. 

11.  Graph  {(x,  y):     |x  +  2|  <    ljf^}  r\    {(x,  y) :     |x  +  2|  +  y  <  0}. 

12.  If  a  man  was  x  years  old  y  years  ago,   how  old  will  he  be  z  years 
from  now? 

13.  Which  of  the  given  numbers  is  the  largest? 

IA\     l  m\      3  (C\     ^5  m,     (0.5)2  ,E.     4/5 

(A)   j  (B)    io  <c)   -TT  {D)   "oTT"  (E)     2 

14.  Simplify. 

(a)  (5a2  +  ab  +  c)  +  (4c  -  2az  -  ab) 

(b)  (7k2  +  k  -  2m2)  +  (5k  -  3m2  -  2k2) 

(c)  (-5mn  +  n3  +  m2)   -  (2m2  -  n3  +  5mn) 

15.  T «_  ^S  What  is  the  sum  of  the  meas- 

r  ures  of  the  angles  ZP,  ZQ,  ZR, 

ZS,   andZT? 

16.  What  is  the  distance  between  the  points  (1,    9)  and  (6,    -3)? 

17.  A  nut  mixture  is    l/3   peanuts,    3/l0   almonds,    and    l/5    cashews. 
The  remainder  is  pecans,    and  there  are  five  pounds  of  these.    How 
many  pounds  does  the  mixture  weigh? 

18.  How  many  pounds  of  filberts  worth  $1.03  a  pound  should  be  added 
to  10  pounds  of  pecans  worth  $.  76  a  pound  to  make  a  mixture  worth 
$.  85  a  pound? 

19.  Solve  this  system  of  equations.     \      6x  +  y+3  =  0 

3x  -  2y  +  9  =  0 


[7-82] 


[7.04] 


20. 


21. 


V 


ioo 


V. 


t  = 


A 


100(V-Vo) 

V         -  V 
yioo       o 


^ 


Given:     quadrilateral  ABCD  is  a 
parallelogram, 

AE  =  EB,    DF  =  FC 

Find:       the  ratio  of  the  area-measure 
of  the  shaded  region  to  that 
of  the  given  parallelogram 


Derive  a  formula  for  t 
in  terms  of  A,    V,    and  Vr 


J 


22.  What  is  the  greatest  number  of  circular  regions,    each  with  an  area 
of   13  square  inches,    which  can  be  stamped  from  a  rectangular 
sheet  of  steel  9  inches  by  13  inches? 

23.  A  motorist  buys    14  gallons  of  gasoline  to  bring  his  tank  from  20% 
full  to  90%  full.     What  is  the  capacity  of  the  tank? 


24.    Simplify, 
(a) 


m  -  1 


m  +  1 


(b) 


4  -t 


5  -t 


(c) 


10 


+ 


2x  -  y         y  -  2x 


25.    A  merchant  who  had  a  certain  number  of  bags  of  flour  passed 

through  thirteen  gates  of  thirteen  cities.     The  curious  levy  at  each 
gate  was  one  half  of  what  he  possessed  at  the  moment.     The  officer 
in  charge  at  each  gate  was  his  friend.     After  the  levy  was  paid, 
each  officer  gave  him  one  bag  as  a  gift.     To  his  surprise,    he  still 
had  exactly  the  same  number  of  bags  after  he  had  completed  his 
journey.     How  many  bags  did  he  have?     [Chih-yi  Wang,    Mathematics 
Magazine,    Nov.  -  Dec,    1959] 


26.    When  successive  discounts  of  30%  and  10%  are  applied  to  the  list 
price  of  an  article,    the  selling  price  comes  to  $44.  10.     What  is 
the  list  price? 


[7.04]  [7-83] 

27.  A  gardener  uses    25  pounds  of  seed  for  a  lawn  50  feet  by  90  feet. 
How  many  pounds  of  seed  is  this  per  square  yard? 

28.  Simplify. 

<a>  <!+!><t  +  t>  <b>  "-h&th  -rrr> 

29.  Bill  and  Emily  are  traveling  on  their  bicycles  at  6  miles  per  hour 
and  5  miles  per  hour,    respectively.     They  start  at  the  same  place 
and  time,   and  travel  in  the  same  direction.     If  after  traveling  3 
hours,    Bill  turns  back,   how  far  from  their  starting  point  will  they 
meet? 

30.  A  rectangle  and  a  circle  have  the  same  perimeter.     Derive  a  for- 
mula for  the  area-measure,    K,    of  the  circle  in  terms  of  the  dimen 
sions,    S.  and  w,    of  the  rectangle. 

31.  Complete:     V     — ^ —  =   ■=-  - 

0^      0   .  x  +  62.008 

32.  Solve  :      x  = 


33.    Simplify. 


1.5749 


I+i 

2  „  v       4        3  ,    v     12  ,  1%     125 


(a)   0.8tJ  (b)  1"1  (c)    ~  (d) 

T*4 


5  *"'    ~        11  vw    74  v~'     375 

1     -     -rr' 


34.  Solve  this  system  of  equations. 

-14y  +  4x  =  28 
-2x  +  7y  =  -14 

35.  If  tangerines  sell  5  for  16£,   how  much  do  Z\   dozen  cost? 

36.  If  a  radius  of  a  circle  is  x  inches  long  and  a  side  of  a  triangle  with 
the  same  area  is  y  inches  long,    how  many  inches  long  is  the  altitude 
to  the  given  side? 


[7-84] 


[7.05] 


7.05    The  relation  greater  than  for  the  positive  integers.  --From  our 
basic  principles  for  real  numbers  it  is  possible  to  prove  that 


and 


(1)  there  is  no  least  positive  number, 

(2)  between  any  two  positive  numbers  there  is  a  third. 


[Given  a  positive  number,    how  can  you  find  a  smaller  one?      Given  two 
positive  numbers,   how  can  you  find  one  between  them?] 

Do  (1)  and  (2)  remain  true  if  'number'  is  replaced  by  'integer'?     Is 
there  a  positive  integer  less  than  1?     Is  there  a  positive  integer  between, 
say,    5  and  6?      Using  our  basic  principles  we  can  show  that  the  answer 
to  both  of  these  questions  is  'no'. 

The  first  question  is  settled  by  the  following  theorem: 


This  theorem  can  be  proved  easily  by  mathematical  induction.     You  will 
be  asked  to  do  so  in  the  next  set  of  exercises. 
The  second  question  is  settled  by: 


(*) 


V      V    [n  >  m 

m     n  L 


n  >   m  +   1] 


For,   according  to  (*),   if  a  positive  integer  is  greater  than  5  then  it  is 
greater  than  or  equal  to  6- -that  is,   if  it  is  greater  than  5,   it  is  not  less 
than  6.     So,    6  is  the  next  positive  integer  after  5. 

How  can  we  prove  (*)?      We  note  that  (*)  is  equivalent  to: 

V      V    [n  >  m  =>  n  -  m  :     1] 
m     n  L  —     J 

And,   we  could  use  Theorem  104  to  prove  this  once  we  have  proved: 


Theorem  105. 


V      V    [n  >  m 

m     n  L 


n  -  m  e  I+] 


[Explain.]     [Theorem  105  is  assumption  (3)  on  page  7-47.] 


[7.05]  [7-85] 

So,    let's  prove  Theorem  105.     A  straight -forward  induction  will  do 
the  job.     To  do  so,   we  must  prove: 


and: 


(i)      Vn  [n  >    1  =>  n  -   1  el+] 


(ii)     V        [V    (n  >   m  =>  n  -  m  e  T)  => 
'       m     L   n 


V     (n  >   m  +   1=>   n  -  (m  +   1)  €  I+)] 


n 


Consider  (i).     By  Theorem  87,    if  p  >    1  then  p  f  1.     [Explain.]     So, 
we  could  prove  (i)  if  we  could  prove: 

(a)  Vn  [n/*   1=>  n  -   1  el+] 

And,    we  could  prove  this  if  we  could  prove: 

(b)  V     [n  =   1  or  n  -   1  €  I+]  [Explain.] 

Now,    (b)  is  easy  to  prove  by  induction.     Here's  how. 

Clearly,    since   1  =  1,    it  follows  that 

1=1     or     1    -   1  e I+ 
Also,    since  (p  +  1)   -   1  =  p  and  p  e  I+,    it  follows  that 

p  +   1  =   1     or    (p  +  1)   -  1  €  I+. 
Hence  [by  conditionalizing  and  generalizing], 

V     [(n  =    1     or    n  -  1  e  I+)  =>    (n  +   1  =   1     or     (n  +   1)  -  1  e  T)] 

Thus,    by  mathematical  induction,    (b) . 

SO;,    we  have  established  (i) .      Now,    on  to  (ii). 

Suppose  that  Vn  [n  >  p  '■=>   n  -  p  €  I+].      Further,    suppose  that 
q  >  p  +   1.     Then,    q  -   1   >  p.     Since  p  +   1  >   p  >L   1 ,    it  also  follows  that 
q  >    1.     So,    from  (i),    q  -  I  e  I+  and,    from  the  inductive  hypothesis, 
(q  -    1)    -  p  e  I  +  .     But,    (q  -   1)   -  p  =  q  -  (p  +   1).     So,    q  -   (p  +   1)  e  T.    Hence, 
Vn  [n  >   p  +   1  =>   n  -  (p  +   1)  €  I*].       Consequently,    (ii). 

So,    from  (i)  and  (ii),    by  mathematical  induction,    Theorem   105. 


[7-86] 


[7.05] 


EXERCISES 


A.    Prove. 


1.     Theorem  104 


V    n  +  1  >    1 
n 


B.     1.     Show  that  (a)  and  (b)  on  page  7-8  5  are  equivalent  by  deriving  (b) 
from  (a)  and  (a)  from  (b). 

2.  Prove:    V    if  n  4  1  then  n  >  2       [Hint.   Use  (a)  and  Theorem  104.1 

n  '  —  L J 

3.  Show  that  1  is  the  only  positive  integer  between  0  and  2. 
[Hint.     Use  Exercise  2.] 


C.    1.     Use  Theorems  104  and  105  to  prove  (*)  on  page  7-84 
2.     Prove  the  converse  of  (*) . 


J,      «A.      «A. 


*-,v    +,<* 


V       T      *X 


(*)  and  its  converse  give  us  : 


Theorem  106. 


V      V    [n  >  m  +  1    <=>    n  >  ml 

m     n  L     —  J 


o,    *^    *•> 
'iv    *ix    *in 


D^.     1.     You  know  how  to  prove  '5  e  I  *  and  *6  e  I  '.     Accepting  these  theo 
rems  prove  that,    for  each  x,    if  5  <  x  <  6  then  x  tf  1*  . 

2.     Prove:      VV      [n  <  x  <  n  +  1  "=>  x  fi'  I+] 

n    x    l  *       J 


IS.    Prove  each  of  the  following. 
1.     V    n/t   1 


n 


2.     V     V    [n  <  m  +  1    <=>    n  <  ml 
m    n  L  —       J 


[Theorem  107a] 
[Theorem  107b] 


rF_.    Prove  (1)  and  (2)  on  page  7-84. 


[7.05]  [7-87] 

LOWER  BOUNDS  AND  LEAST  MEMBERS 

Consider  the  set  P  of  all  positive  numbers.     We  have  noted  earlier 
that  P  has  no  smallest  member—that  is,    there  is  no  positive  number 
which  is  less  than  every  other  positive  number.     But,    there  is  a  real 
number  x  such  that,    for  each  y  €  P,    x  <_  y.     Name  one  such  real  number. 
Name  another.     How  many  are  there?      Each  such  real  number  is  called 
a  lower  bound  of  P . 

You  have  seen  that  P  does  not  have  a  least  member  but  that  it  does 
have  many  lower  bounds.     Does  P  have  a  greatest  lower  bound? 

Consider  the  set  K  of  all  positive  integral  multiples  of  3.     Does  K 
have  a  least  member?      Does  K  have  a  lower  bound? 

Consider   {x:    x  >   6}.     Does  this  set  have  a  least  member?      Does 
it  have  a  lower  bound? 

Consider  {n:  n  >  6}.  Does  this  set  have  a  lower  bound?  Does  it 
have  a  least  member?     [Recall  what  the  domain  of  4n'  is.] 

Consider  the  set  of  negative  integers.       Does  it  have  a  least  member? 
Do  you  think  it  has  a  lower  bound?      Do  you  think  the  set  of  positive 
integers  has  an  upper  bound?      [To  settle  these  last  two  questions,    we'll 
need  another  basic  principle.] 

By  now  you  probably  have  a  good  idea  of  what  is  meant  by  *a  lower 
bound  of  a  set  of  real  numbers'    and    'a  least  member  of  a  set  of  real 
numbers'.      But,    for  definiteness,    here  are  formal  definitions: 

V..  V     [x  is  a  lower  bound  of  S    <==>    V        _  x  <   yl 
S    x    L  y  €  S       —   7i 

VQ  V     [x  is  a  least  member  of  S   <^=> 
o    x 

x  is  a  lower  bound  of  S  and  x  e  S] 

Use  the  first  definition  to  prove  that  each  number  which  is  less  than 
some  lower  bound  of  a  set  is  also  a  lower  bound  of  the  set. 

Using  both  definitions  and  Theorem  93,    you  can  prove  that  a  set 
cannot  have  two  least  members.     Do  so. 

Suppose  that  S  is  a  set  of  real  numbers.     If  S  has  a  least  member, 
must  S  have  a  lower  bound?      If  S  has  a  lower  bound,    must  it  have  a 
least  member? 

Suppose  that  T  and  S  are  sets  of  real  numbers  such  that  T  C_  S.  If 
S  has  a  lower  bound,  does  T?  If  S  has  a  least  member,  does  T?  If  T 
has  a  lower  bound,    does  S?      If  T  has  a  least  member,    does  S? 


[7-88] 


[7.05] 


THE  LEAST   NUMBER  THEOREM 

A  set  which  has  a  least  member  is  certainly  not  empty.     But,    as 
you  have  seen,    there  are  many  nonempty  sets  of  real  numbers  which  do 
not  have  least  members.     One  of  the  most  useful  theorems  about  I+  says 
that  none  of  its  subsets  is  of  this  kind: 


Theorem   108. 

Each  nonempty  set  of  positive  integers 
has  a  least  member. 


This  theorem  is  often  called  the  least  number  theorem.     [An  equivalent 
formulation  is:    Each  set  of  real  numbers  which  contains  a  positive  inte- 
ger contains  a  least  positive  integer.] 

We  shall  prove  the  least  number  theorem  by  showing  that  the  only 
set  of  positive  integers  which  has  no  least  member  is  the  empty  set. 

Suppose  that  S  is  a  set  of  positive  integers  which  has  no  least  mem- 
ber.    Since   1  is  the  least  positive  integer,    and  since  S  is  a  set  of  positive 
integers,    1  is  a  lower  bound  of  S.     Since  S  has  no  least  member,    1  i  S. 
So, 

(i)      VjneS=>    1<    n]. 


n 


Suppose  now  that  V     [n  e  S  ■=>   p  <   n].     By  Theorem   106,    it  follows  [using 
the  substitution  rule  for  biconditional  sentences]    that 

V     [n  e  S  =>   p  +   1  <   n] 


n 


--that  is,    that  p  +   1  is  a  lower  bound  of  S.     So,    since  S  has  no  least 
member,    it  follows  that  p  +   1  (  S.     Hence,    V    [n  e  S  ~=>  p  +   1  <   n]. 
Consequently, 

(ii)     V     [~V    (n  e  S  =s>   m  <   n)  =>    V    (n  e  S  =>   m  +   1  <   nfl. 
mLn  n  J 

From    (i)    and   (ii)    it  follows,     by  mathematical  induction,    that 

'   m  <  n]. 

In  particular,    V     [n  e  S  ==>  n  "     n] .      Since  V    x  i-  x,    it  follows  that 
V    n  g   S.  --that  is,   that  no  positive  integer  belongs  to  S. 

The  least  number  theorem  is  assumption    (5)  on  page  7-47.      It  is 
interesting  to  recall  how  we  used  this  theorem  back  in  Unit  4.     We  were 


V      V    [n  e  S 

m     n  L 


[7.05]  [7-89] 

trying  to  show  that  there  is  no  positive  rational  number  whose  square  is 
8.     We  argued  that  if  there  were  such  a  rational  number,    V8,   then  there 
would  be  positive  integers  p  and  q  such  that  v8  •  q  =  p--that  is,    that 
{q:   V"8  *q  €  I+}  is  not  empty.     From  this  supposition  and  Theorem  108  it 
follows  that  there  is  a  least  positive  integer  q  such  that  V8  •  q  €  I+.    Then 
we  proceeded  to  derive  a  contradiction,   and  so  concluded  that  there  is 
no  positive  rational  number  whose  square  is  8.     [Notice  that  up  to  now 
we  have  not  shown  that  there  is  any  positive  number  whose  square  is  8. 
All  we  know  is  that,   if  there  is  such  a  number,   it  is  not  a  rational  num- 
ber.    In  a  later  unit  we  shall  adopt  a  final  basic  principle  which  will 
make  it  possible  to  prove  that  each  positive  real  number  has  a  positive 
square  root.  ] 

THE  COFINALITY  PRINCIPLE 

A  few  paragraphs  back  we  raised  the  question  of  whether  there  is  a 
lower  bound  for  the  set  of  negative  integers.     By  Theorem  94,   this  is 
equivalent  to  asking  if  there  is  an  upper  bound  for  the  set  of  positive 
integers.     And,    from  our  intuitive  knowledge  of  the  real  numbers,   we 
would  be  inclined  to  say  'no'.     That  is,    we  believe  that  there  is  no  real 
number  which  is  greater  than  or  equal  to  each  of  the  positive  integers.    In 
still  other  words,    we  believe  that,   for  each  real  number,   there  is  a 
positive  integer  which  is  greater  than  it.     As  a  matter  of  fact,   this  is 
so,    but  it  cannot  be  deduced  from  the  basic  principles  we  have  set  down 
up  to  now.     So,    let  us  now  take  this  as  a  basic  principle. 


Cofinality  Principle 


V    3      n  >  x 
x      n 


[How  do  you  read  '3    '?] 


When,    in  a  later  unit,    we  adopt  our  final  basic  principle  for  the  real 
numbers,    we  shall  see  that  the  cofinality  principle  then  becomes  a 
theorem. 

From  the  cofinality  principle  it  follows  that    V    3     n>  -x.     [Explain.] 
So,    Vx3nx>    -n.     Hence,    Vx3nx^-n.     But,  this  is  just  to  say  that  no 
real  number  is  a  lower  bound  of  the  set  of  negative  integers. 


[7-90]  [7.05] 

EXERCISES 
A.    Give  an  example  of  a  set  S  of  real  numbers  which  has 

1.  both  an  upper  bound  and  a  lower  bound; 

2.  a  lower  bound  but  no  upper  bound; 

3.  an  upper  bound  but  no  lower  bound; 

4.  neither  an  upper  nor  a  lower  bound; 

5.  a  greatest  member  but  no  upper  bound; 

6.  an  upper  bound  which  belongs  to  S; 

7.  a  least  upper  bound  which  does  not  belong  to  S; 
**&.     a  lower  bound  but  not  a  greatest  lower  bound. 


B.     1.     Prove:    V    .    n  3     x  >    - 

—  x  >   0      n  n 

2.     Suppose  that  &,  =  I+  and  that,   for  each  n,    f     =   — . 
«*r  f  n        n 

(a)  Does  &f  have  an  upper  bound?      A  lower  bound? 

(b)  Does  (ft-  have  a  greatest  member?      A  least  member? 


»    i    i  w        nil, 


1  '  '  (-1) 

3.     Repeat  Exercise  2  with     —     replaced  by 


n  n 


•  ■   * 


1   *  *  n  -   1   ' 

4.     Repeat  Exercise  2  with     —     replaced  by 


n  n 


C.  1.     What  does  Theorem  108  tell  you  about    (n:    3       xn  =  m)  if  x  is 

—  }  v  m  J 

(a)    a  positive  rational  number?  (b)    an  irrational  number? 

*2.     Suppose  that  m  >  n.     Show  that  there  is  a  p  such  that  0  <  m  -  np  <  n. 
[Hint .    Consider   {q:   3     q  =  m   -  np}.] 

D.  1.     Suppose  that,    for  each  n,    f     =  n(n  +   1)  and  g     =  (n  +   -=-)2. 

(a)  Prove:    V „[f     =  g    =>  f  =  g  ] 

n  L  n       &n  n  +  l       en  +  iJ 

(b)  In  (a)  you  gave  part  (ii)  of  an  inductive  proof  of  the  generali- 
zation 4Vnfn  =  gn'.  Is  f4  =  g4?     Does  your  answer  contradict 
the  result  you  obtained  in  (a)?      Explain. 


[7.05]  [7-91] 

2.     Consider  the  following  generalization: 

(*)  V     V    [n  <   m  ^^  n  =  m] 

x    '  m    n  L     —  J 

(a)  Give  a  counter  example  for  (#). 

(b)  Criticize  the  following  purported  proof  of  (*) : 

(i)      From  Theorems   104  and  93  it  follows  that 

V     [n  <    1=>  n  =  l]< 

n  L     —  J 

(ii)    Suppose  that  V    [n  <  p  =>  n  =  p].      Further,    suppose 
that  q  <  p  +   1.     Then,    q  -    1  £  p.     So,   from  the 
inductive  hypothesis,    q-  1  =  p,    and  q  =  p  +   1. 
Hence,    V     [n  <  p  +   1  =>   n  =  p  +  1],     Consequently, 


n 


V 

m 


V    [n  <   m=>  n  =  ml  =>   V     [n  <  m  +   l=>  n  =  m  +   l] 
n  L     —  J  n  L     —  J 


(iii)  So,    from  (i)  and  (ii),    by  mathematical  induction, 

V     V    fn  <  m  =>  n  =  m]. 
m    n  L     —  J 

(c)    Compare  (b)  with  the  proof  of  Theorem   105  on  page  7-85. 


MISCELLANEOUS  EXERCISES 

1.     For  each  n,    C     is  the  number  of  subsets  of  an  n-membered  set. 

n 

(a)  Find  a  recursive  definition  for  C.     [Hint .     Suppose  that  SQ  is  an 
n-membered  set  and  that  eQ  ^  SQ.     Then,    SQ  w  {eQ}  is  an 

(n  +   l)-membered  set,    S.     How  many  subsets   of  S  do  not  contain 
eQ  ?      How  many  do  contain  eQ?] 

(b)  Show  that  each  nonempty  set  has  the  same  number  of  odd- 
membored  subsets  as  it  has  even-membered  ones.     [Check  this 
result  in  the  table  you  completed  on  page  7-73.] 

(c)  A  ladder  which  reaches  just  to  the  roof  of  a  building  has    10  rungs 
By  how  many  routes  can  a  beetle  climb  along  the  ladder,    from 
the  ground  to  the  roof,    supposing  that,    on  each  route,   he  never 
retraces  his  path? 


[7-92]  [7.05] 

2.  Each  function  defined  below  has  I+  as  domain.     Give  a  recursive 
definition  for  each  and  then  use  mathematical  induction  to  derive 
the  given  explicit  definition  from  the  recursive  definition. 

(a)    f     =  7n  +  1  (b)    fn  =  7n  +  9  (c)    fR  =  -n 

(d)    f     =  6  -  5n  (e)    f     =  n2  (f)    f     =  2n2  -  5 

N    '     n  n  n 

3.  Each  of  the  following  generalizations  is  false.     Give  a  counter- 
example for  each. 

'^Vo^  =  5  (b)  v*^-*-1 

(c)  Vo   lZk  +  k    =k  (d»    Vx/3  -IFTT  =  ~2 

4.  If  the  length-  and  width-measures  of  a  rectangle  are  doubled,    by 
what  per  cent  does  the  area-measure  increase? 

5.  The  surface  area-measure  of  a  solid  cube  is   16  times  that  of  a 
second  cube.     If  the  cubes  are  made  of  the  same  substance  and  the 
larger  cube  weighs  80  pounds,   what  does  the  smaller  one  weigh? 

6.  Simplify. 

(a)  8rs  -  2r2  +  7s2  +  (2r2  -  3rs  -  s2)  -  (r2  -  s2) 

(b)  7x(2x2  -  3x  +  2)  -  5x(4x  -  3x2  +  1)  +  (x  -  2)2 

(c)  (3a  -  2b)(a  +  b)  -  4(a  +  2b)(a  -  3b)  -  5(a  -  b)(a  +  b) 

7.  How  much  longer  does  it  take  a  train  to  travel  1  mile  at  55  miles 
per  hour  than  at  60  miles  per  hour? 

8.  AABC  and  AADC  are  inscribed  in  a  circle  for  which  AC  is  a  diam- 
eter.    If  AB  =   1,   AC  =  5,    and  AD  =  3,   find  CB  and  CD. 

9.  Find  an  ordered  pair  (x,  y)  such  that  x  -  y  =  8  and  5x  +  2y  -  5  =  0. 


[7.05]  [7-93] 

10.  hn  =  E  +   -=-mv2   )  Derive  a  formula  for  V  in 

.  /  terms   of  h,    n,    E,    and  e. 

eV  =   imv2  J 

11.  Solve  the  equation  'm  =   r-  '    for    ln'. 

1    -    - 

n 

3 

12.  Suppose  that  A  =    c"B.     What  is  the  value  of  A  for  an  argument  for 

which  A  and  B  have  the  same  value? 


13.    Find  the  coordinates  of  the  midpoint  of  the  segment  whose  endpoints 
have  coordinates  {y,    -^-)    and    (-5-,    y) . 


x       x 
14.    For  each  positive  integer  x,    3     •  9     = 


(A)     27X  (B)     272x  (C)     33x  (D)     93x  (E)     32x 


3  1 

15.    If  you  subtract   ^-  from  a  number  and  get   -=-,    what  would  you  get  if 

3 

you  added   —  to  the  number? 

7  4 


16.  If  a  pipe  3  inches  in  diameter  can  drain  a  tank  in  x  minutes  then  a 
pipe  4  inches  in  diameter  can  drain  the  tank  in  minutes. 

17.  If  it  takes   24  days  for  20  people  to  consume  a  certain  stock  of  food, 
how  long  would  it  take  25  people  to  consume  it? 

18.  Solve:    x%(800)  =  32 

19.  Simplify. 

,    .      a2  +  7a  +   12    ^    a2  +  4a  -  5  ,.  .     rs  +  r     .     r2s  +  r2 

(a)       ^  +  2a  -   3     X    "im <b>     — sT~    T    ~ 3i~ 

20.  If  (0,    3)  and  (1.  5,    0)  belong  to  a  linear  function  f,    which  ordered 
pair   of  f  has  equal  components? 


[7-94] 


[7.06] 


7.06    The  integers.  --Up  to  now,    in  our  study  of  real  numbers,   we  have 
introduced  some  basic  principles  to  single  out  the  positive  real  numbers 
and  some  other  basic  principles  to  single  out  the  positive  integers.     We 
also  adopted  the  basic  principle  (G)  in  order  to  introduce  the  relation  >, 
and  the  cofinality  principle  to  help  specify  how  the  positive  integers  occur 
among  the  real  numbers. 

The  next  step  in  our  program  is  to  study  the  set,  I,  of  integers -- 
positive,  negative,  and  0.  We  already  know  what  real  numbers  these 
are,   and  we  formulate  this  knowledge  in  a  basic  principle: 


(I)      V     [x  e  I    <=>    (x  e  I+  or  x  =  0  or  -x  e  I+)] 


So,    to  say  that  a  real  number  is  an  integer  is  to  say  that  it  is  a  positive 
integer,    or  is  0,    or  its  opposite  is  a  positive  integer. 

Our  first  theorem  gives  us  another  way  of  characterizing  the  set  of 
integers.     Read  it  carefully  and  see  if  you  believe  what  it  says. 


Theorem   109. 

V     [x  e  I    <= 
x  L 

->    3      3      x  =  m  ■ 

m      n 

-n] 

This  theorem  tells  us  that  a  real  number  is  an  integer  if  and  only  if  it  is 
a  difference  of  positive  integers.  Using  the  basic  principle  (1)  it  is  easy 
to  prove  that  this  is  so.     Here  is  a  proof. 

In  the  first  place,    suppose  that  m  and  n  are  positive  integers.      If 
m  >  n  then,    by  Theorem  105,    m  -  n  €  I+;    so,    by  (I),    m-nel.     If  m  =  n 
then  m  -  n  =  0;    so,  by  (I),    m  -  n  e  I.     Finally,  if  n  >   m  then,  by  Theorem 
105,   n  -  m  €  I+.     But,    n  -  m  =   -(m  -  n) .     So,    if  n  >   m  then  -(m   -  n)  e  I+. 
Hence,    by  (I),    m  -  n  €  I.     Since,    in  any  case,    m>   norm  =  norn>   m, 
it  follows  that  each  difference  of  positive  integers  belongs  to  I. 

In  the  second  place,    suppose  that  a  real  number  a  is  an  integer.    By 
(I),    either  a  e  I*  or  a  =  0  or   -a  e  I+.     Suppose  that  a  e  1* .     Then,  by  (I+2), 
a  +  1  €  I+.     Since,    by  (1^),    1  e  I+,  and  since  a  =  (a  +   1)   -   1,    it  follows  that 
a  is  a  difference  of  positive  integers.     Suppose  that  a  =  0.     Since  1  e  I+ 


[7.06] 


[7-95] 


and  1  -  1  =  0,   a  is,   again,    a  difference  of  positive  integers.      Finally, 
suppose  that  -a  e  I+.      Then,   as  before,    -a  +  1  e  I+,     1  €  I+,    and  a  = 
1  -  (-a  +  1).     So,    again,    a  is  a  difference  of  positive  integers.     Hence, 
each  integer  is  a  difference  of  positive  integers. 

Just  as,    for  the  purpose  of  stating  generalizations  about  positive 
integers,  it  was  convenient  to  introduce  special  variables  [4m',    4n',    4p', 
4q']   whose  domain  is  I+,    so,   in  stating  generalizations  about  integers  it 
will  be  convenient  to  use  special  variables  whose  domain  is  I.     For  these 
we  shall  choose  the  letters  4i',    4j',    and  4k'. 

Among  such  generalizations  are  those  which  say  that  I  is  closed 
with  respect  to  opposition,  addition,  subtraction,  and  multiplication. 
[Is  I  closed  with  respect  to  division?] 


Th 

eorem 

110. 

a. 

V. 

J 

-j 

e  I 

b. 

V. 

J 

\ 

k  +  j  e 

I 

c . 

V. 

J 

\ 

k  -  je 

I 

d. 

V. 

J 

\ 

kj  e  I 

Let's  prove  the  first  two  parts  of  this  theorem. 

Suppose  that  j  is  an  integer.      Then,    by  Theorem  109,    there  are 
positive  integers  m  and  n  such  that  j  =  m  -  n.      So  [by  Theorem  33], 
-j  =  n  -  m.     Hence,  by  Theorem   109,    -j  €  I.     This  completes  the  proof 
of  part  a. 

Suppose  that  j  and  k  are  integers.     Then,    by  Theorem  109,    there 
are  positive  integers  m,    n,    p,    and  q  such  that 


k  =  m 


n 


and 


j  =  p  -  q 


So,    k  +  j  =  (m  -  n)  +  (p  -  q)   =  (m  +  p)  -   (n  +  q).     Since  I+  is  closed  with 
respect  to  addition  [Theorem   102],    m  +  p  e  I+  and  n  +  q  e  I+.     Hence,    by 
Theorem   109,    k  +  j  e  I.     This  completes  the  proof  of  part  b. 
Now,    prove  parts  c  and  d. 


[7-96] 


[7.06] 


Here  are  other  theorems  about  integers  which  you  can  prove: 


Theorem   112. 

V.  V    [k  +   1  >    i    <= 

=>    k  >  j] 

The  proof  of  Theorem  111  involves  the  same  techniques  used  in  proving 
Theorem  110.     To  prove  Theorem  112  use  Theorems    111  and  104,    and 
theorems   on  >  . 

EXPLORATION  EXERCISES 

A.     1.     Suppose  that  S  =   {k:    k  >    -7}.     Describe  the  set,    S    ,    obtained  by 
subtracting  -5  from  each  member  of  S. 

2.     Let  t       be  the  function  [mapping],    whose  domain  is  I,    such  that 
t       (k)  =  k  -   -5.      Complete  this  picture  to  show  the  mapping  t_g. 


-8-7-6-5-4-3-2-10       1       2       3       4       5       6       7       8       9 

<e-| , 1 \ 1 , 1 1 1 1 , , , 1 1 1 1 ,_> 


I*-! 


H h 


4-> 


-8-7-6-5-4-3-2-10       1       2       3       4       5       6       7       8       9 


3.     What  is  the  range  of  t_    ?     Does  t        have  an  inverse?     Onto  what 
set  does  t_5  map   {k:    k  >    -5}? 


4.     For  a  given  integer  j,    let  t.  be  the  function  such  that,    for  each 
integer  k,  t.  (k)  =  k  -  j. 

(a)  What  is  the  range  of  t.? 

(b)  If  i  >  k,    what  can  you  say  about  t.  (i)  and  t.  (k)? 

J  J 


[7.06] 


[7-97] 


(c)  Look  at  Part  H  on  page  7-41.  The  results  there  hold  just  as 
well  for  functions  whose  domain  is  I.  So,  what  do  Exercises 
1  and  2  of  Part  H  tell  you  about  t.? 

5.     (a)    Describe  the  set  obtained  by  applying  t_10   to  each  member 
of  {k:    k  >    ~10}„ 

(b)  Describe  the  set  obtained  by  applying  t27   to  each  member 
of  {k:    k  >   27}. 

(c)  Describe  the  set  obtained  by  applying  t.   to  each  member 
of  {k:    k  >  j}. 


B.     1.     Let  r  be  the  function,    whose  domain  is  I,    such  that   r(j)  =  -j. 
Draw  a  picture  like  that  in  Exercise  2  of  Part  A  to  show  the 
mapping  r.     Does  r  have  an  inverse? 


2.     For  each  set  listed  below,    describe  the  set  obtained  by  applying 
r  to  each  of  its  members. 

(b)    {j:    j  <   5} 

(d)    {j:     -6  <  j  <   5} 

(0     0:    j  <  k} 


(a)    {j:    j  >   -1} 

(c)    {j:     -5  <  j  < 

-4} 

(e)    {j:    j  <   -8} 

3.     Look  at  Theorem  94.     Use  it  to  complete  the  following  statement 

V    V.  [r(i)  >   r(j)  ] 

J 


4.     (a)    Pick  a  real  number.     By  the  cofinality  principle,    you  know 
that  there  is  an  integer  which  is  greater  than  this  real  num- 
ber.    Can  you  prove  that  there  is  an  integer  which  is  less 
than  this  real  number?     [You  can  do  this  by  using  the  cofi- 
nality principle  to  get  an  integer  which  is  greater  than  the 
opposite  of  the  chosen  real  number.  ] 

(b)   Suppose  that  a  set  S  has  a  lower  bound.     Prove  that  it  has  an 
integral  lower  bound. 


[7-98] 


[7.06] 


THE  LEAST  NUMBER  THEOREM  FOR  INTEGERS 

Consider  a  nonempty  set  S  of  integers.     If  S  contains  only  positive 
integers  then,   as  we  have  proved  [Theorem  108],   S  has  a  least  member. 

On  the  other  hand,    we  know  that  if  S  contains  negative  integers,   it 
may  not  have  a  least  member.     Give  an  example  of  such  a  set.    Now  give 
an  example  of  a  set  of  integers,    some  of  which  are  negative,   which  does 
have  a  least  member.     Can  you  guess  a  theorem? 


Theorem  113. 

Each  nonempty  set  of  integers  which 
has  a  lower  bound  has  a  least  member. 


For,    suppose  that  S  is  a  nonempty  set  of  integers  which  has  a  lower 

bound.     Then,    by  the  cofinality  principle,    there  is  an  integer  j  which  is 

less  than  each  member  of  S.     [Explain.] 

Now,    let  t.  be  the  function,   whose  domain  is  I,    such  that  t-(k)  =  k- j. 
J  J 

Let  S'f  be  the  set  obtained  by  applying  t.  to  each  member  of  S.      Then, 

S>c  C^  I+   [Explain],      So,    by  Theorem  108,    S  ,s  has  a  least  member  m. 

Since  m  e  S*,   there  is  a  k  6  S  such  that  t  •  (k)  =  m.     Since  m  is  a  lower 
bound  of  S*,   k  is  a  lower  bound  of  S  [Why?].      So,    S  has  a  least  member,  k 

The  proof  of  Theorem  113  illustrates  a  general  method  for  deducing 
theorems  about  integers  from  analogous  theorems  about  positive  integers. 
In  Part  A  of  the  preceding  Exploration  Exercises  you  discovered  certain 
properties  of  the  functions  t;.     For  a  given  integer  j,   the  essential  prop- 
erty of  t-  is  that  it  maps  the  set  of  integers  greater  than  j  on  the  set  of 


positive  integers  in  such  a  way  that  order  is  preserved.    So,  for  example, 
lower  bounds  of  a  subset  S  of   {k:   k  >  j}  are  mapped  onto  lower  bounds   of 
the  image  of  S.    And,  by  Part  H  on  page  7-41,    it  follows  that  each  lower 
bound  of  the  image  of  S  is  the  image  of  some  lower  bound  of  S.    In  general, 


[7.06] 


[7-99] 


because  ti  has  this  property,    anything  you  can  say  about  the  positive 
integers  which  has  only  to  do  with  their  order,    you  can  also  say  about 
the  set  of  integers  greater  than  j. 

For  example,    the  principle  of  mathematical  induction  gives  us  a 
method  for  proving  theorems  concerning  {k:    k  >  0}.      Using  the  prop- 
erties of  the  function  t.        ,    for  any  j,    we  can  transform  the  principle 

J 
of  mathematical  induction  [see  page  7-49]  into  a  principle  which  enables 

us  to  prove  theorems  concerning  {k:  k  >  j  -  1  }  --that  is,  {k:  k  _>  j}. 
Theorem  112  tells  us  that,  for  each  k,  k  +  1  is  the  least  integer  >  k. 
So,   the  PMI  tells  us  that,    for  each  set  S, 


if 


(the  least  integer  >   0)  €  S 
and 


k>  0 


[k  e  S  — >   (the  least  integer  >   k)  e  S] 


then 


Vk>okeS' 


It  is  easy  to  see  that  the  validity  of  this  result  depends  only  on  the  way 
in  which  the  positive  integers  are  ordered.     Since  the  members  of 
{k:    k  >  j   -   1}  are  ordered  in  the  same  way,    we  may  replace  the  40's  in 
the  statement  above  by  lj   -   l's.     Since 

(the  least  integer  >  j  -   1)  =  j, 
k  >  j   -   1    <=^>    k  >  j, 
and  (the  least  integer  >   k)  =  k  +   1, 

we  have  the  following  theorem: 


Theorem   114. 


(]  e  S  and  Vk  > 


±) 


[k  e  S  =>  k  +  1  6  S]) 


V1S  .keS 


We  can  use  this  theorem  to  give  us  mathematical  induction  principles 
for  proving  theorems  about  all  integers  greater  than  or  equal  to  some 
specified  integer  j.     For  example,    in  case  j  =   1 ,    the  corresponding 
instance  of  Theorem   114  is  equivalent  to  the  PMI  itself. 

In  Part  B  of  the  Exploration  Exercises  you  worked  with  the  function 
r  such  that,  for  each  j,  r(j)   =   -j  .     This  function  maps  I  onto  itself  in  such 


[7-100] 


[7.06] 


a  way  that  order  is  reversed.     So,  for  example,  lower  bounds  of  a  set  S 
of  integers  are  mapped  by  r  onto  upper  bounds  of  the  image  S     of  S. 
Similarly,   upper  bounds  of  S  are  mapped  onto  lower  bounds  of  S*.     From 
this  and  Theorem  113  we  have: 


Theorem  115. 

Each  nonempty  set  of  integers  which  has  an  upper  bound 
has  a  greatest  member. 


Also,   since,   as  Theorem  114  tells  us,   we  can  do  mathematical 
induction  "upward"  from  any  given  integer,   so  we  can  do  mathematical 
induction  "downward"  from  any  given  integer.     Using  the  order-revers 
ing  property  of  r  we  can  obtain  from  Theorem  114: 


Theorem  116. 


(jcSand  Vk£j[keS=>  k-1  eS])=>  Vk<.  keS 


From  Theorems   114  and  116  we  obtain  a  theorem  of  mathematical 
induction  for  the  set  of  all  integers : 


Theorem  117. 


(0  €  S  and  V,    [k  €  S  =>  (k  +  1  €  S  and  k  -  1  e  S)])  =>  V  k  e  S 


1. 


EXERCISES 

Complete  the  following  recursive  definition  of  the  numbers,   D   ,    of 
diagonals  of  an  n -sided  polygon: 

V   .   -    D     ,       =D     + 
n  >^  3      n  +  i  n 

[Hint .     Draw  a  diagonal  joining  a  vertex  to  a  next  but  one.  ] 


2.      Discover  an  explicit  definition  of  D: 


n  >  3      n 


3.       Use  the  appropriate  instance  of  Theorem  114,    and  derive  the  explicit 
definition  from  the  recursive  one. 


[7.06]  [7-101] 

MISCELLANEOUS  EXERCISES 
A.     1.     Here  is  a  recursive  definition  for  a  function  f: 

=    f 


nn+i  n     n  +  2 

(a)  Compute  f2,   f3,    f4,    and  f5. 

(b)  Guess  an  explicit  definition  for  f,    and  try  to  derive  it  from 
the  recursive  definition  by  mathematical  induction. 

n     +     1 

2.  If,    for  each  n,    g     =  then,    for  each  n,    g  =    — JTT' 

3.  If,    for  each  n,    gn  =  n(n  +   1)  then,    for  each  n,    g  =  gn  + 

4.  Complete  these  generalizations  about  the  positive  integers. 

/    »    ^         n  1 

[*}      n   n+l         (n+l)(n+2)    "    n+2 

(b)    V     n+1    +   I =   

K    '      n   n  +  2         (n+2)(n+3)         n+  3 

.    .    w     n  +  2    j  1 


n   n  +  3         (n+  3)(n  +  4) 

u,    w     n  +  3    ,  n  +  4 

n   n  +  4  n  +  d 


B.     1.     Complete. 

(a)    V    x3  +   1  =  (x  +   1)(  )  (b)    Vv  x3  -   1  =  (x  -   1)(  ) 

■A.  X 

2.  If   —  =   —  +   —    find  the  value  of  s  which  corresponds  with  the  value 

r         s  t  r 

6  for     r    and  4  for    t  . 

3.  If  a  cubic  centimeter  of  water  weighs    1  gram,    how  many  kilograms 
of  water  will  fill  a  rectangular  container  45  cm.    by  35  cm.  by  8  cm.? 

i 

4.  If  the  square  of  the   sum  of  two  numbers  is    16  and  the  square  of 
their  difference  is  4,    find  their  product. 


[7-102]  [7.06] 

THE  GREATEST  INTEGER  FUNCTION 

One  consequence  of  the  cofinality  principle  is  that,    given  a  real 
number,  there  is  an  integer  less  than  or  equal  to  it.    That  is,  for  each 
x,    {k:   k  <.  x}  f  0.     Since  the  nonempty  set  {k:   k  <^  x}  has  an  upper  bound 
[Explain.],   it  follows  from  Theorem  115  that  it  has  a  greatest  member. 

For  each  of  the  sets  listed  below,    give  its  greatest  member. 

(1)  {k:  k  <  4.3}  (2)  {k:  k  <   2y} 

(3)  {k:  k  <    13/3}  (4)  {k:  k  <    tt} 

(5)  {k:  k  <   8}  (6)  {k:  k  <   -1.3} 

(7)  {k:  k<?r/2}  (8)  {k:  k<2/3} 

(9)  {k:  k<    -0.5}  (10)  {k:  k<2223/llll} 

Instead  of  writing: 

the  greatest  member  of  {k:    k  <^  x} 
it  customary  to  write: 

Read  'HxJ'  as  'brackets  x'  or  as  'the  integral  part  of  x'.    So,  for  exam- 
ple,  instead  of  asking:    What  is  the  greatest  member  of  {k:    k  <  4.3}? 
we  can  ask:    What  is  [4.  3  J? 

Now  complete  the  following  sentences: 

(H)     [[7. I]    =  (12)     [[0I|    =  (13)     E-5.2D    = 

(14)    [7T+  1J   =  (15)    I-6]|    =  (16)    [[0. 62]   = 

Here  is  a  list  of  the  integers. 

...,    -6,    -5,    -4,    -3,    -2,    -1,   0,     1,    2,    3,   4,    5,    6,    ... 

(17)    Where  would  you  draw  a  line  to  separate  the  integers  which  are  less 
than  or  equal  to  ir  from  those  which  aren't? 

(1.8)    Where  would  you  draw  a  line  to  separate  the  integers  which  are  less 
than  or  equal  to   ([  -n  J   from  those  which  aren't? 

(19)    Repeat  (17)  and  (18)  for  the  real  numbers    /I,     >/l,     V4,    and  -/IT. 


[7.06]  [7-103] 

Questions  (17)  -(19)  suggest  the  following  theorem  which  follows 
immediately  from  the  definition  of  '[[...  ]]': 

v   \  [k  1  ttxH   <==>    kl  xl  [Theorem  118a] 


EXERCISES 

A.     1.     What  is  the  greatest  member  of    {k:    k  <  4.3}?      What  is  the 
greatest  member  of  {x:    x  <  4.  3}? 

2.  Which  of  the  following  do  not  stand  for  integers? 

(a)  [[7.5]]  (b)    [[7.5  +  7.5]  (c)    [  7.  5  ]  +  |[7.  5  ] 
(d)    19+2.7])                   (e)9  +  [2.7j  (f)|[9]  +  2.7 

(g)    12.9.6]  (h)    2[9.6]  (i)    Ij'9.61  (j)    i|[9.6] 

3.  (a)    Use  Theorem  118a  to  prove:    V    [[  x  ]  <  x 

[Hint.   By  Theorem  118a,   the  integer  [[  c  ]  is  less  than  or 

equal  to  c  if  and  only  if  it  is  less  than  or  equal  to  [[  c  J. 
But,    .  .  .  ] 

(b)  Use  part  (a)  to  prove:    V    2  {[  x  ]]  <  2x 

(c)  Use  Theorem  1 1  8a  and  part  (b)  to  prove :    V    2[[xj£[[2x]] 

A 

(d)  Use  part  (a)  to  prove:    V     y{[x]]  <    yx 

(e)  Prove  or  disprove:    V     y{[x]]  £  Hyx]j 

(f)  Which  of  the  following  is  a  theorem? 

(i)    vx  vk>0kttxli- HkxB 

(")     Vx  Vy>Q  ylx]<    [yxl 

(g)  Tell  which  of  the  following  is  a  theorem  and  prove  it. 
(i)      Vx  Vk  Jx]]  +  k  <   ffx  +  kj 

(ii)     V    V    [xj  +  y  <   Ix  +  y] 

*    y 

(h)    Prove  or  disprove:     VV     JxJ  +  dyJ^Ix  +  yJ 

x    y 


[7-104]  [7.06] 

B.    The  set  of  ordered  pairs, 

{(x,  y):    y  =  [x]}, 

is  a  function  which  maps  each  real  number  onto  the  greatest  integer 
not  greater  than  the  real  number.     So,    it  is  sometimes  called  the 
greatest  integer  function. 

1.     Make  a  sketch  of  the  greatest  integer  mapping. 

_4       -.3       _2       -1        0  12  3  4         5  6 

1 1 1 1 j , 1 1 1 , , 


H 1 h 


-4-3-2-10123456 
2.     Graph  4y  =  [xj\  3.     Graph  4y  =  [x  +  1  J'. 

*'-      <A,     *J* 
T-     T     "lN 

You  probably  discovered  in  Part  A  that 

Vx  j[x+  I]]    =    [x]|+  1. 

Let's  try  to  prove  this. 

In  Exercise  3(g)  of  Part  A  you  used  Theorem  118a  to  prove  a  theorem 
which  tells  you  that 

Vx  [xj  +  1  <  |[x+  lj. 

So,    by  Theorem  93,    all  we  have  to  prove  is  that 

Vjx]  +  1  >   Ix+  1]. 

Evidently,  to  prove  this,   it  would  be  helpful  to  have  a  theorem  like  Theo- 
rem 118a  but  which  would  start  with: 

VxVk[k>  Ix]    «=> 

Theorem  118a,    with  the  help  of  Tiieorem  88,   tells  us  that,    for  any  real 
number  c  and  any  integer  k, 

(*)  k  >  [cj    <=>    k  >  c.  [Explain.] 

This  tells  us  about  >  rather  than  >.  But,  >  -sentences  about  integers 
can  be  translated  into  > -sentences  with  the  help  of  Theorem  112.  In 
fact,   for  integers  k  and  j, 

(**)  k  >  j    <=>    k  +  1   >  j. 


[7.06] 


[7-105] 


By  (*),    since  k  +  1  is  an  integer, 

k+  1  >  [cj   <=>    k  +  1  >  c.  [Explain.] 

So,    since  [cj  is  an  integer,   it  follows  from  (*)  and  (**)  that 

k  >   [cj    <=>    k  +  1  >  c. 


Consequently, 


Now  we  can  prove  : 


V   V,    [k  >   |[x  J    <=>    k  +  1  >  x].       [Theorem  118c] 


x    k 


VxIx]  +  1  >   [x+  1] 
Since,   for  any  c,    [[  c  J  +  1  is  an  integer, 

He  J  +  1  >   |[c  +  1] 


<==>   ([c  J  +  1)  +  1  >   c  +  1 
<=>     [[c]  +  1  >   c 
<=>     II  cj  >  Ic]. 


Theorem   118c 

\  Why? 

Theorem  1 18c, 
|[  c  ]}  is  an  integer 


So,    since  each  real  number  is  greater  than  or  equal  to  itself, 

ffcj  +  1  >  Ic  +  I]]. 

Consequently,    V    [x]+l>[[x+l]|. 

Notice  that  Theorems  118a  and  118c  can,    with  the  help  of  Theorem 
93,    be  combined  into: 


Theorem   118e. 


VxVk[k  =  JxJ    <=>    k  <  x  <  k+  f) 


This  theorem  gives  us  a  quick  way  of  proving  that  a  given  integer  is  the 
integral  part  of  a  given  real  number. 


O-      o^     «J, 
■T*      "V      T 


C.    Use  Theorem  118e  to  prove: 

VVV.  fl>  +  jB  =  [xj  +  j  [Theorem  119] 

[Hint.     Since,    for  any  real  number  c  and  any  integer  j,    [[  c  ]]  +  j  is 
an  integer,  all  you  need,  by  Theorem   118e,  is  to  show  that 

Ic]  +  j<c  +  j<(|[c]|  +  j)  +  l. 
Use  Theorem  1 1  8e  to  show  that  [c]l<c<|[c]|+l.] 


[7-106] 


[7.06] 


D.    Exercise  3(h)  of  Part  A  tells  you  that  the  integral  part  of  the  sum  of 
two  real  numbers  is  at  least  as  great  as  the  sum  of  the  integral  parts 
of  the  real  numbers.     Can  it  be  greater?     By  how  much?     Prove  this. 


E.     1. 


Consider  the  real  numbers  93  and  18.     It  is  possible  to  find  an 
integer  k  such  that 

18k<93<    18(k+l).  [Find  it.] 

In  fact,    given  any  real  numbers  a  and  b,    b  >  0,   there  exists  an 
integer  k  such  that 

kb  <  a  <   (k  +  l)b 

Prove  that  such  an  integer  exists.     [Hint.     In  the  example  about 

(19  3"* 

93  and  18,    did  you  see  that  the  sought -for  integer  is 


18 


?] 


2. 


In  Exercise  1  you  proved  the  following  theorem: 

V  V   .  n  3,    ky  <  x  <  (k  +  l)y 
x    y  >  0.      k     ' 

Now  prove : 

(a)  VxVy>03k°^X-ky<   V 

(b)  VxV   >      ^  3z  (x  =  ky  +  z  and  0  <   z  <  y)  [Theorem  120] 


3.     Use  the  cofinality  principle  on  page  7-89  to  prove: 

V   V   .   „   3     ny>x  [Theorem  121] 

x    y  >  0       n     7  L  J 


F.     1.     Graph  both  *y  =  HxJT  and  *y  =  tt  -*]'  on  the  same  chart 


2.     Graph  4y  =  -|[-x]|\ 


3.     Complete: 

Vx  -ff-xj  =the 


integer  k  such  that  x   [Theorem  122] 


4.     The  U.S.   Post  Office  has  a  book  rate  of  9  cents  for  the  first 

pound  and  5  cents  for  each  additional  pound  or  fraction  of  a  pound. 
Find  the  formula  for  the  postage  c,  in  cents,  for  a  parcel  of  books 
which  weighs  w  pounds. 


[7.06] 


[7-107] 


G.    On  a  picture  of  the  number  Line,    graph  the  solution  set  of 

[5.36J  +  [x]    =    [5.36  +  xj 


V«*  VV  ^l- 

'Is      "|>      "1N 


Since ,   for  each  x,    [xje  I  and  [  x  ]  <  x  <   |[x]|+  1,   it  follows  that, 
for  each  x,  there  is  an  integer  k,  namely  ffx]|,  such  that  0  <  x  -  k  <    1. 
Instead  of  writing  'x  -  [xj'  it  is  customary  to  write  '{[xj'    [read  as 
'braces  x'  or  as  'the  fractional  part  of  x']. 

V    x  =  [xj+  fxj,     where  [  x  J  €  I  and  0  <  ffx}  <    1 


The  set  of  ordered  pairs, 


{(x,  y):    y  =   «  }, 


is  a  function,   and  is  sometimes  called  the  fractional  part  function 


•Jt*      O^      <J^ 
"T*       "lx      *jx 


H.     1.     Graph  'y  =  {fxj\ 

2.     State,   and  prove,    a  theorem  about  the  fractional  part  function 
analogous  to  the  theorem  about  the  greatest  integer  function 
which  you  proved  in  Part  C. 


I.      Solve. 

1.     [5.36])  +  [x]  =  [5.36  +  xj 


2.     [5.36J  +  [xj  <   [5.36  +  xj 


-sir 


J^.     Compute . 

75621' 


1. 


4. 


10 


[75621 

I    104 


10 


10 


7.     V 


k>  5 


75621 


2. 


5. 


75621 


\L  io^ 


75621 
105 


10 


-]] 


10 


10 


10 


3. 


6. 


75621 

'75621 
106 


10 


10 


Ox 


[7-108] 


[7.06] 


8.    Find  the  positive  integer  n  which  satisfies  the  following  six  sen- 
tences. 


10] 


10< 


10 


10 


=  7 


=  5 


102 


105 


10 


10 


•j. 


=  3 


=  5 


n 


10: 


k>  5 


10 


n 
101 


=  0 


10 


=  0 


9.    Complete  without  doing  any  dividing  or  multiplying. 


(a) 


357829630572164 


10 


n 


10 


(b) 


1T68257934625TI  . 
[I!  1035  jf 


10.  Repeat  Exercises   1-6  with  each  *10'  replaced  by  an  48'.      Then, 
formulate  a  theorem  like  that  in  Exercise  7. 

11.  Recall  the  results  of  Exercises   1-6  and  that 

75621  =  l_  +  2«  10  +  6/  102  +  5_-  103  +  7-  104. 

Now,   interpret  the  result  of  Exercise   10  by  completing  and  checking 
75621  =  5  + 


12.    Use  the  facts  that 

15278  =  4+5-7  +  3-  72  +  2  •  73  +  6-  74 


and  that 


15278  =  2  +  1  •  12  +   10  •  122  +  8-  123 


to  complete  the  following: 
15  27811    - 


(a) 


(c) 


15278 


(b) 


(d) 


15278 
123 


15278 
125 


nL  Vl^  O-p 

T*      "Jv     *'l> 


12 


12 


In  doing  the  exercises  in  Part  J  you  discovered  a  technique  for 
'splitting"  each  positive  integer  into  multiples  of  powers  of  a  given 


[7.06] 


[7-109] 


integer  greater  than  1.      When  the  given  integer  is  10,  the  splitting 
leads  to  the  customary  decimal  notation  for  positive  integers.     As  you 
probably  guessed,    or  perhaps  know,   it  is  possible  to  use  bases  other 
than  10  in  a  place  value  system  of  names  for  integers.     For  example, 
the  integer  whose  name  in  the  decimal  system  is  '75621'  is  named 
'223545'   in  the  base-8  system.      Let's  find  the  base-6  name  for  this 
integer.     According  to  the  procedure  illustrated  in  the  exercises,  the 
units  digit  is  obtained  by  simplifying: 


ilnty  0  ■  I2'"  ♦  0 ']  ■ 


3 

o" 


=  3 


The  sixes  digit  is  obtained  by  simplifying: 

75621 
62 


Instead  of  computing  75621    v   36  directly  by  long  division,   we  can  make 

use  of  the  fact  that  75621   4-   6  =  12603  +  f . 

o 


75621 


12603  + 


2100  +  £  +  ^g-V 


o"  +   3o"      6 


-! 


=    3 


Now  there  is  a  short  cut  here.     Instead  of  computing 

3 


12603  + 


we  could  have  computed 


12603 


[Check  this.  ] 


This  suggests  a  very  simple  procedure  for  finding  the  digits. 


[7-110] 


[7.06] 


6 
6 


75621 


12603 


2100 


350 


58 


6 

6 


9 

1 
0 


3 
0 
2 
4 
3 
1 


\  Remainders 


j 


So,   the  base-6  name  for  75621  is  *  13420 3 3' --that  is, 

75621  =  3+3-6  +  0-62  +  2-63  +  4-64+3'65+  1  •  66 

[Check  this.  ] 

The  short  cut  used  here  is  a  consequence  of  the  theorem: 

75621 


1L6P " l  M 


Similarly,  the  digits  obtained  in  Exercises   1-6  of  Part  J  by  evaluating 

'75621 


10' 


10 


for  the  values  1,   2,    3,   .  .  .  ,   6  of  4p',   can  be  obtained  by  evaluating,   in- 
stead: 

r~  tT 

75621 


]_10 


p-1 


J) /no 


10 


Evaluate  the  two  expressions  for  the  value  3  of  'p'. 

The  short  cut,   in  general,    depends  on  the  theorem 


V   V 
x    m 


& 


m 


JIM. 


m 


m 


To  understand  this  theorem,   consider  a  real  number  c  and  a  positive 

c 
integer  p.    Since  —  is  the  sum  of  its  integral  part  and  its  fractional  part, 


(*) 


I**© 


—  >>p      and    0  < 


[7.06] 


[7-111] 


In  words:    one  can  divide  a  real  number  c  by  a  positive  integer  p  in  such 
a  way  as  to  obtain  an  integral  partial  quotient,       —  L   and  a  nonnegative 
remainder,  (f~T)p»   less  than  p.     [What  has  been  said  up  to  now  holds 
for  any  positive  number  p,   integral  or  not.]    The  theorem  says  that  the 
integral  part  of  the  remainder  on  dividing  a  real  number  by  a  positive 
integer  is  the  remainder  on  dividing  the  integral  part  of  the  real  number 
by  the  positive  integer. 


To  see  why  this  is  so,    return  to  (*)  and  note  that,   since 
it  follows  from  (*),   by  Theorem  119,   that 


pel, 


(#*) 


H- 


c 
P 


Now,   since  0  <  ((— )l,p  <  p.   it  follows  that  0  < 


0  < 


<  p,    so  that 


l<  i. 

P 


Hence,  from  (**)  ["dividing  by  p"], 

'gel 

p 


<  M  < 

""      P 


+  1 .       Conse- 


quently, 

From  this  and  (**), 

We  combine  these  two  results  into  two  theorems: 


Theorem  124. 

V   V      0  <    i 
x    m       — 

(fi*i\ 

}- 

=  E«l  - 

>- 
X 

m 

_     j 

m  <  m 

iml 

In  a  later  unit  we  shall  use  the  second  of  these  two  theorems  to 
prove  that,   for  each  m  >   1,   each  positive  integer  has  a  base-m  place 
value  name. 

Use  the  algorithm  based  on  Theorem  123  to  find  the  base-9  name 
for  75621. 


[7-112]  [7.06] 

MISCELLANEOUS  EXERCISES 
A.    1.     Complete  and  prove  the  generalizations  about  the  positive  integers 
(a)     V     n(n  t  l)   +   (n  +  1)  =  t±±M 1 

{b)     V     n(n  *  '><"  +  2>   +  (n  +  l)(n  +  2)  =  <n+1><     t     H 1 


<c)     Vn  n(n+l)(n+3)(nt4)   +  (ntl)|nH)(nt3)  = 


2.  If,  for  each  n,  g  =  n(n  +  l)(2n  +  1)  then,  for  each  n,  g 
(n+l)(     )(     ). 

3.  If,  for  each,.p,  f  =  p(2p  -  l)(2p  +  1)  then,  for  each  p,  f 

(         )(         K         ). 


B.    1.     The  ancient  Babylonians  used  the  following  incorrect  formula 
for  finding  the  area-measure  of  a  quadrilateral: 

K   _    (a  +  c)(b  +  d) 

where  K  is  the  area-measure,   and  a,   b,   c,   and  d  are  the  meas- 
ures of  consecutive  sides. 

(a)  If  the  formula  is  applied  to  a  parallelogram  or  a  trapezoid, 
does  it  give  an  overestimate  or  an  underestimate  of  the 
area -measure? 

(b)  For  which  class  of  quadrilaterals  is  the  formula  correct? 

2.  The  number  of  television  sets  in  a  certain  city  is  48000.     If 
this  means  that  there  are  13.9  television  sets  per  1000  of  popu- 
lation,  what  is  the  population  of  the  city  ? 

3.  All  but  one  of  the  following  numerals  stand  for  the  same  num- 
ber.    Which  one  does  not? 

(A)    |  (B)     1.125  (C)    ^  (D)    |i  (E)    || 


[7.06]  [7-U3] 

4.  The  vertices  of  a  triangle  are  A(0,   8),    B(0,  4),    and  C(5,    1). 
What  is  the  area -measure  of  AABC? 

5.  Simplify. 

(a)   (4&*b^(-3aV)  (b)    -3p2q7*7p4q6 

(c)    (20xy2z3)(4x2y3z)  (d)    -8r2s5t  •  -8s¥u 

6.  If  1  child  is  born  every  x  seconds,  how  many  children  are  born 
in  a  half -hour? 

7.  By  selling  an  article  at  30%  above  the  cost  price,   a  dealer  makes 
a  profit  of %  of  the  selling  price. 

8.  45  seconds  is  what  per  cent  of  2  hours? 

9.  Simplify. 

2  +   5  i+  I 

(a)  0.6  *   |  (b)  |  +  £  -  3L  (c)    L^  (d)    ^JL 


8 


10.    Solve  this  system  of  equations. 


f  x  +  y  +  48 

x  =     1.474?9 

6.3*4*0  =  y+3-024 

11.  If  80  light  bulbs  cost  k  dollars,  how  many  can  you  buy  for  75  cents? 

12.  A  team  is  scheduled  to  play  30  games  during  the  season.     If  it  loses 
20%  of  the  first  15  games,  how  many  of  the  remaining  games  must 
it  win  to  make  the  season  win  average  at  least  90%  ? 

13.  Mr.  Sanchez  plans  to  travel  350  miles  in  8  hours  of  driving.  If  he 
averages  40  miles  per  hour  during  the  first  5  hours,  what  must  he 
average  during  the  remaining  3  hours? 


[7-114]  [7.06] 

14.    Solve  the  equation  4S  =   j  (a  +  i)'  for  '*'. 


15.  Suppose  that  one  amoeba  divides  into  two  every  30  seconds.     Under 
these  conditions,    it  takes  24  hours  to  fill  a  test  tube.     If  you  start 
with  two  amoebas,   how  long  would  it  take  to  fill  the  same  test  tube? 

16.  Simplify. 
(a)(r^r-^]xTT^:T  ,b)    <      *  2      >-    *2-2 


x-4        x-3J        ll-2x  ''    la+1        a+2 


4  1         1         1  ' 
17,    Solve  the  equation     —  =  —  +  —     for     r'. 

r        p        q 


18.  If  a  car  travels  x  miles  in  y  hours,   what  is  its  rate  in  feet  per 
second? 

19.  One  method  used  by  the  Egyptians  to  find  the  area-measure  of  a 
circle  was  to  compute  the  area-measure  of  a  square  whose  side- 
measure  is  8/9  of  the  diameter  of  the  circle.     Does  this  method 
give  an  overestimate  or  an  underestimate  of  the  area-measure  of 
the  circle? 

'  ab 

20.  Solve  the  equation     —  =  d     for  4b*. 

21.  If  the  sum  of  two  numbers  is   10  and  the  sum  of  their  reciprocals 
is  5,    what  is  their  product? 


3 

22.  Which  of  the  given  numbers  is  not  ■=-  ? 

(A)V?        <b)  7f        (c)  7f        (D)  i*?        (E)  ^ 

23.  Give  a  geometric  argument  to  show  that,   for  each  a  >  0  and  for 
each  b  >  0,    Va*  +  b*    <  a  +  b. 


24.    If  a  first  number  is  40%  of  a  second,    what  per  cent  of  the  second 
is   125%  of  the  first? 


[7.06] 


[7-115] 


DIVISIBILITY 

You  know  that  m  is  a  factor  of  n  with  respect  to  I+  [or:   m  divides  n, 
or:   m|n]   if  and  only  if  there  is  a  positive  integer  p  such  that  n  =  mp: 

V     V    [m  In   <=>    3     n  =  mp] 
m    n  L      '  p  rj 

So,    2  |4  and  6|l2,   but  5^9-     And,    since  n=  1  »n  =  n»  1,   it  follows  that 


Theorem  125. 


V    [1  Jn  and  n  jn] 


EXERCISES 


A.      True -or -false? 

1.    5 130         2.    l|78 
6.    7|21  7.    Zl j 168 

11.    3|59        12.    3 (63  +  59 
15.    2 1 103 •  104 


3.  82|80  4.    80|82  5.    8|  17 

8.  7  1 168  9.    8|80  +  888       10.    3  |63 

13.  7|14'8273591  14.    2|94«95 

16.  6|94-95«96  17.    6  1 103  •  104  •  105 


B^.      Prove  each  of  the  following. 

Sample.       V     V     [m  |n=>  m  <  n] 


[Theorem  126a] 


Solution.     Suppose  that  m  Jn.     It  follows  that  there  is  a  q  such 
that  mq  =  n.     By  Theorem  104,    1  <  q  and,   by 
Theorem  101,    m  >  0.     Hence,    using  Theorem  86e, 
it  follows  that  m  =  m  •  1  <  mq  =  n.     So,   if  m  jn  then 

■  m  <  n]. 


m  '•  n.     Consequently,    V     V    fm  In 

~  ^  J        m    n  L      ' 


1.     V     V   V    [(m|nandn|p) 
m    n    p  L        '  ,r 


m 


|p] 


2.     V     V     [(m  |n  and  n  |m)  =>  m  =  n] 

[Hint.   Use  the  theorem  in  the  Sample.] 


[Theorem  126b] 
[Theorem  126c] 


Exercises    1  and  2  say  that    I   is  transitive  and  antisymmetric 


3.     V     V   V     [(mlnandmlp) 
m    n    p  l        '  ' r' 


m (n  +  p] 


[Theorem  126d] 


[7-116] 


[7.06] 


4. 
5. 
6. 


V  V   V    [(m  In  and  m  In  +  p) 
m    n    p 

V  V   V    [m|n=>  mp|np] 
m    n    p  l      ' 


m 


|p] 


[Theorem  126e] 
[Theorem  126f] 


V    2|n(n  +  1)    [Hint .    For  part  (ii)  of  an  induction  proof,   note  that 
(p  +  l)(p  +  2)  =  p(p  +  1)  +  2(p  +  1).     Now,   use  Theorem  126d,  above.] 


7.     Vn  6|n(n  +  l)(n  +  2) 


'8.     V    6  n(n2  +  5) 
n      ' 


*9.     For  each  odd  n,    24|n(n2  -  1) 


THE  DIVISIBILITY  RELATION 

The  relation  j  has  much  in  common  with  the  relation  <.  : 


V  In 

n      ■ 

V  n  In 

n      ' 


V™  V«  ^  Km  ln  and  n  Ip) 
m    n    p  ' 


m 


|p] 


V     V    [( m  I  n  and  n  |  m)  =i>  m  =  n] 

m    n  l        '  '  J 


V     V    V    [mln  < — >  mplnp] 
m    n    p  l      '  r  i    rj 


V  1  <  n 
n      ~ 

V  n  <  n 
n      ~~ 

V  V   V    [(m  <:  n  and  n  <:  p)  r=^>  m  <,  p] 

m    n    p  l        ~  ~~  r  rj 

V     V    f(m  <  n  and  n  <  m)  =>  m  =  n] 

m    n  ~" 

V     V    (m  <  n  or  n  <  m) 

m    n         "~  — 

V  VV    [m  «:  n  <=>  m  +  p  '■  n  +  p] 
mnpL—  r  —  rj 

V  V    V    [m  <  n  <=>  mp  *'■  np] 
mnpL_  r"_      r-j 


The  two  gaps  in  the  left  column  indicate  two  important  differences  between 

j  and  <.     Another  difference  is  that  there  is  no  theorem  about   J  which 

corresponds  with  4V     V    [m  <  n   < — >    m  <  n  +  1]'. 
r  m    n  L      —  J 

In  spite  of  these  differences,  the  first  four  theorems  about   |  do  show 

that,   like  <,    J  orders  the  positive  integers  in  an  interesting  way.    To  see 

what  this  means,   note  that  <.  orders  the  positive  integers  which  are  less 

than  or  equal  to  a  given  one  in  a  very  simple  way.     For  example,   those 

which  are  less  than  or  equal  to  6  are  1,    2,    3,   4,    5,   and  6.     Since  1  <   2, 

2  <_  3,    3  <_  4,   4  <_  5,   and  5  <  6,   we  may,   knowing  that  <  is  reflexive  and 

transitive,  indicate  the  < -order  of  these  integers  schematically  by: 


On  the  other  hand,   the  positive  integers  which  are  divisors  of  6  are  1, 
2,    3,   and  6.     Since  1  |2,    l|3,    2|6,    and  3|6,   we  may,   knowing  that   j  is 


[7.06] 


[7-117] 


reflexive  and  transitive,   indicate  the   (-order  of  these  integers  by: 

2 


1 


The  diagram  tells  us  that  1  \Z,    1  |3,    2|6,   and  3  J6.     Knowing  that   |  is 
transitive,   we  conclude  that  1  J 6,   and,   knowing  that   j  is  reflexive,   we 
conclude  that  1  1 1,    2|2,    3J3   and   6  1 6 . 

For  certain  choices  of  n  [for  example,   for  n  =  8],    j  orders  the 
factors^of  n  in  the  same  simple  way  [linearly]  in  which  <,  orders  the 
positive  integers  which  are  less  than  or  equal  to  n.      However,    for 
"most'*  choices  of  n  this  is  not  the  case.    As  examples,   consider  24, 
whose  factors  are  1,    2,    3,   4,   6,    8,    12,   and  24: 


30,   whose  divisors  are  1,    2,    3,    5,   6,    10,    15,  and  30 

2 6. 


1 


0- 


^30 


15 


and  60,   whose  factors  are  1,    2,    3,   4,    5,   6,    10,    12,    15,    20,    30,   and  60 


[7-118]  [7.06] 

EXERCISES 

A.  Here  are  the  divisibility-ordering  diagrams  for  the  divisors  of  8 
and  for  the  divisors  of  243. 

1 2 4 8 

1 3 9 27 81 243 

Give  ten  more  examples  of  positive  integers  whose  diagrams  are 
linear. 

B.  The  diagram  for  6  is  two-dimensional.     So  are  the  diagrams  for  12 
and  24.      Draw  the  diagram  for  48;   it  is  two-dimensional,    also. 
Other  two-dimensional  diagrams  are  those  for  18,    54,    36,    108, 
and  215. 

C.  1.     Draw  the  diagram  for  1125.     [Hint.    1125  =  32  •  53] 

2.  Draw  the  diagram  for  500. 

3.  Give  three  more  examples  of  positive  integers  whose  diagrams 
have  the  same  shape  as  the  diagrams  in  Exercises   1  and  2. 

4.  Draw  the  diagrams  for  23  •  34  and  24»  54. 

5.  For  each  number  listed  below,  tell  how  many  factors  it  has. 
(a)    32-53                (b)    22-53  (c)    23  •  34  (d)    24-56 
(e)    5b-73                 (f)     251-380             (g)    751-1180              (h)    23  •  3 
(i)     13-17               (j)     28                        (k)     317  (i)    43 
(m)43«105               (n)    142»73               (o)    l4-25-32  (p)    24-35-52 

D.  The  diagrams  for  30  and  60  are  three-dimensional.     Characterize 
the  positive  integers  whose  diagrams  are  three-dimensional. 

E_.    If  a  positive  integer  has  just  two  factors,   its  diagram  looks  like 
this: 

1  D 


[Such  numbers  are,    of  course,   prime  numbers.]    If  a  positive 


a 


[7.06]  [7-119] 

integer  has  just  four  factors,   its  diagram  looks  either 

like:      1 p p2  p3  or  like :         1  ^  ^pq 

q 

[What  are  the  smallest  such  positive  integers?] 

Give  all  possible  diagrams  for  a  positive  integer  which  has  just 

1.     three  factors  2.     five  factors  3.     six  factors 

4.     eight  factors  5.     twelve  factors  6.     sixteen  factors 

HIGHEST  COMMON  FACTOR 

It  is  one  of  the  important  properties   of  <  that,    given  two  positive 
integers,    one  is  less  than  the  other.     So,    by  transitivity,   the  positive 
integers  which  are  less  than  or  equal  to  both  of  two  positive  integers 
are  just  those  which  are  less  than  or  equal  to  the  smaller  of  the  two. 
For  example,   since  12  <   30    [and  since  <.  is  transitive], 

V  [(p  <   12  and  p  <   30)  <=>  p  <    12]. 

The  situation  with  regard  to  the  relation   j  is  less  simple,   and  more 
interesting.     Given  two  positive  integers  one  of  which  divides  the  other, 
the  positive  integers  which  divide  both  are,    by  the  same  argument  used 
above,   just  those  which  divide  the  smaller  of  the  two.     But,    given  two 
positive  integers,    it  is  not  usually  the  case  that  one  of  them  does  divide 
the  other.     To  see  what  happens,    let's  again  consider  12  and  30.     Since 
12  1 60  and  30  1 60,   all  the  divisors  of  12  and  all  those  of  30  are  indicated 
in  the  diagram  on  page  7-117  which  shows  how   j  orders  the  divisors  of 
60.     From  this  we  see  that  the  set  of  divisors  of  12  is    {1,    2,    3,   4,    6,  12} 
and  the  set  of  divisors  of  30  is    {1,    2,    3,    5,    6,    10,    15,    30}.     The  set  of 
common  divisors  of  12  and  30  is  the  intersection  of  these  two  sets.     So, 
it  is 

{1,    2,    3,   6}. 

This  is  just  the  set  of  divisors  of  6.     So, 

V  [(p|l2   and  p|30)    <=>    p|6]. 


[7-120]  [7.06] 

Just  as  12  is  the  greatest  integer  less  than  or  equal  to  both  12  and  30, 
6  is  the  "most  divisible"   integer  which  divides  both  12  and  30.       6  is 
called  the  highest  common  factor    [HCF;    or:    the  greatest  common 
divisor,    GCD]  of  12  and  30. 

Find  the  HCF  of  15  and  24,     of  25  and  50,    of  21  and  27,     of  14  and 
33,    and  of  23  •  3s    and   27  •  34. 

A  positive  integer  d  is  the  HCF  of  m  and  n  when 

V    [(p|m  and  p|n)    <=>    p|d]. 

Notice  that  this  means  [only  if -part]  that  each  common  divisor  of  m  and 
n  divides  d  and  [if -part]  that  d  divides  both  m  and  n.     Using  Exercise  2 
of  Part  B  on  page  7-115,   it  is  easy  to  show  that  no  two  integers  have 
more  than  one  HCF.      [Do  so  now,   by  showing  that  if  each  of  d1  and  d2 
is  an  HCF  of  m  and  n  then  d1  |d2  and  d2  j d. x .  ]     However,    we  have  yet  to 
show  that  each  two  integers  do  have  an  HCF.     In  doing  so,   we  shall  dis- 
cover an  important  property  of  highest  common  factors. 

Exercises  3  and  4  of  Part  B  on  page  7-115  suggest  a  procedure  to 
use  in  searching  for  the  HCF  of  two  positive  integers.     For  they  tell  us 
that  the  divisors  of 

n  +  p  and  n 
are  just  the  divisors  of 

n  and  p. 

So,   the  HCF  of  n  +  p  and  n  is  the  HCF  of  n  and  p. 

For  example,    let's  find  the  HCF  of  30  and  12.     Since  30  =  12  +  18, 

HCF(30,  12)  =   HCF(12,    18). 

Since  18=12  +  6,  HCF(18,  12)  =   HCF(12,   6). 

Since  6J12,  HCF(12,  6)  =   6. 

So,  HCF(30,  12)  =    6. 

In  obtaining  the  first  two  of  the  displayed  equations,    we  subtracted 
12  from  30  twice  and  found  that 

30  -  12*2   =   6. 

This  told  us  that  HCF(30,    12)  =  HCF(12,   6).      Since 

12-6*2   =   0, 

HCF(12,    6)  =  6.     So,   HCF(30,    12)  =  6. 
[Notice  that  30-  1  +  12  •  -2  =  HCF(30,    12).] 


[7.06]  [7-121] 

Let's  try  this  with  some  larger  numbers,    say  9672  and  4147. 
Since  9672  -  4147  X  2   =    1378, 

we  see  that         HCF(9672,    4147)    =   HCF(4147,    1378). 
Since  4147  -  1378  X  3    =    13, 

it  follows  that   HCF(4147,    1378)    =  HCF(1378,    13). 
But,  1378  -  13  X  106    =   0. 

So,   HCF(1378,    13)  =  13.     Hence,   HCF(9672,    4147)  =  13. 

In  finding  HCF(30,    12)  we  discovered  that  there  are  integers  i  and  j 
such  that 

HCF(30,    12)  =  30i  +  12j. 

[These  integers  are  1  and  -2,    respectively.]    In  other  words,   HCF(30,  12) 
is  an  integral  linear  combination  of  30  and  1 2 . 

Is  HCF(9672,    4147)  an  integral  linear  combination  of  9672  and  4147? 
That  is,    are  there  integers  i  and  j  such  that    13  =  9672i  +  4147j?      Our 
investigation  of  this  question  will  suggest  a  method  of  proving  that  each 
two  positive  integers  do  have  an  HCF. 

We  begin  by  noting  that  both  9672  and  4147  are  integral  linear  com- 
binations of  9672  and  4147. 

9672  =  9672  X  1  +  4147  X  0 
4147  =  9672  X  0  +  4147  X  1 

We  also  note  that  1378  is  the  remainder  upon  dividing  9672  by  4147,   and 
that  13  is  the  remainder  upon  dividing  4147  by  1378.     So,   we  can  show 
that  13  is  an  integral  linear  combination  of  9672  and  4147  if  we  can  show 
that  the  remainder  upon  dividing  one  integral  linear  combination  of  these 
two  numbers  by  a  second  such  linear  combination  is,    again,    an  integral 
linear  combination  of  the  two  numbers  [Explain.] 

Let's  show  this.     Suppose  that  p  and  q  are  integral  linear  combina- 
tions of  some  two  integers  m  and  n.       That  is,    suppose  that  there  are 
integers  i,   j,   k,   and  l  such  that   p  =  mi  +  nj    and    q  =  mk  +  ni .      The 
remainder  upon  dividing  p  by  q  is 

p  -  q£E]  =  (mi  +  »j>  -  (mk  +  nl)|[£]  .  m(i  -  k[| ])  +  »(j  -  ||[£  }). 
Since    '  *-    €  I  and  since  I  is  closed  with  respect  to  multiplication  and  sub- 

ILCU1 

traction,    it  follows  that  the  remainder  is  an  integral  linear  combination 


[7-122]  [7.06] 

of  m  and  n. 

So,    since  1378  is  the  remainder  upon  dividing  9672  by  4147,    both  of 
which  are  integral  linear  combinations  of  9672  and  4147,    so  is   1378. 
And,    since  13  is  the  remainder  upon  dividing  4147  by  1378,    both  of  which 
are  integral  linear  combinations  of  9672  and  4147,    so  is   13. 

We  have  proved  that  there  are  integers  i  and  j  such  that 

13  =  9672i  +  4147j.  [Find  two  such  integers.] 

Our  examples  suggest  to  us  that  if  two  positive  integers  have  a 
highest  common  factor  then  it  is  an  integral  linear  combination  of  the 
two  integers.     To  show  that  two  positive  integers  m  and  n  do  have  an 
HCF,    let's  consider  the  set  S  of  all  positive  integers  which  are  integral 
linear  combinations  of  m  and  n. 

S  =  {p:    3^-    p  =  mi  +  nj} 

As  we  have  seen,    m  e  S,   n  e  S,    and  if  p  and  q  belong  to  S  then  if  the 
remainder  upon  dividing  p  by  q  is  positive,   it  also  belongs  to  S. 

Since  S  is  a  nonempty  set  of  positive  integers,   it  has  a  least  member 
d.    Suppose  that  p  is  any  member  of  S.    Since  d  e  S,    the  remainder  upon 
dividing  p  by  d  is  either  0  or  belongs  to  S.    This  remainder  is  p  -  d  II  ~  II. 
Since 

0<p-d[|]<d, 

it  follows  that  the  remainder  is  less  than  d.    Since  d  is  the  least  member 
of  S,  the  remainder  is  0.      That  is, 

So,    d|p.    We  have  shown  that  d  divides  each  member  of  S.    In  particular, 
d  |m  and  d  |n. 

On  the  other  hand,    since  d  e  S,   there  are  integers  i  and  j  such  that 

d  =  mi  +  nj. 

So,    each  common  divisor  of  m  and  n  divides  d. 

Hence,    each  two  positive  integers  m  and  n  do  have  a  highest  common 
factor.        In  fact,    HCF(m,    n)    is  the  least  member  d  of  the  set  S  of  those 
positive  integers  which  are  integral  linear  combinations  of  m  and  n. 


V     V    3.  3.    HCF(m,  n)  =  mi  +  nj  [Theorem   127] 

m    n      i      j  j  \.  j 


[7.06]  [7-123] 

The  algorithm  for  computing  the  highest  common  factor  of  two 
positive  integers  m  and  n: 

m  -  nq2   =   r1 
n    -  riq2=  r2 

ri  "  ^3=  r3 

is  nothing  more  than  a  convenient  way  of  getting  smaller  and  smaller 
members  of  S  until  you  finally  find  the  one  which  divides  each  member 
of  this  set.     This  is  the  last  nonzero  remainder. 

Example  1.      Find  HCF(3705,    2618). 

Solution. 

1_ 

2618) 3705  3705  -  2618*1  =  1087 

2618 

1087 

2 

1087)2618  2618  -   1087-2  =  444 

2174 

444 

2 

444)  1087  1087  -  444 • 2  =  199 

888 

199 

2 

199)144  444-199-2  =  46 

398 

46 


46)  199  199  -  46-4  =  15 

184 

15 


_3 

15  )  46  46  -  15  •  3  =  1        -< HCF 

45 

1 


[7-124] 


[7.06] 


Example  2.      In  Example   1  we  found  that  1  is  the  highest  common 
factor  of  3705  and  2618.     So,    we  know  that   1  is  an 
integral  linear  combination  of  3705  and  2618.    Find 
integers  i  and  j  such  that 

3705i  +  2618j  =  1. 

Solution.     The  required  integers  can  be  found  by  using  the 

equations  obtained  in  the  Solution  for  Example  1 . 
One  way  is  to  substitute  from  the  first  equation  into  the 
second  ['3705  -  2618*  V  for  '1087'],  then  from  the  first  and 
second  equation  into  the  third,    etc. 

Another  way,   perhaps  easier,   is  the  following: 


3705  =  3705 

2618  =  3705 

3705  -  2618-  1  =  1087  =  3705 

2618  -  1087-  2  =     444  =  3705 

1087  -     444-2  =      199  =  3705 

444  .      199-2  =        46  =  3705 

199   -        46-4  =        15  =  3705 

46  -        15-3  =  1  =  3705 


1  +  2618-0 

0  +  2618- 1 

1  +  2618- -1 
-2  +  2618- 3 
5  +  2618- -7 
-12  +  2618-17 
53  +  2618-  -75 
-171  +  2618-242 

t  t 


^EXERCISES 

A.    Use  the  algorithm  to  find  the  highest  common  factor  for  each  pair 
of  positive  integers.     [This  algorithm  is  sometimes  called  Euclid's 
algorithm  for  finding  greatest  common  divisors.] 

1.     7469,    2387  2.     5320,    4389 


3.     8749,    11143 


4.     4147,    10672 


B.    Reduce  each  fraction  to  lowest  terms. 


1. 


2387 
7459 


2. 


4389 
5320 


3. 


8749 
11143 


4. 


2574 
5049 


[7.06]  [7-125] 

C.    For  each  of  the  following  equations,   find  a  pair  of  integers  which 
satisfy  it. 

1.     27i  +  17j  =  1  2.     38i  +  43j  =  1 

3.     270i  +  170j  =  10  4.     38i  -  43j  =  1 

5.     270i  +  170j  =11  6.     38i  +  43j  =  2 

^EXPLORATION  EXERCISES 

A,  For  each  of  the  following  linear  equations,  tell  whether  its  graph 
contains  a  point  with  integral  coordinates.     If  it  does,  give  the  coor- 
dinates of  one  such  point;   if  it  doesn't,   say  so. 

1.     13x+10y=l  2.     42x  +  60y  =  6 

3.     13x  +  lOy  =  2  4.     42x  +  60y  =  5 

5.     525x  +  231y  =  42  6.     7469x  +  2387y  =  0 

B.  Each  of  the  following  linear  equations  is  satisfied  by  (0,   0).      For 
each  equation,   find  several  other  pairs  of  integers  which  satisfy  it. 

1.     6x  +  lOy  =  0  2.     3x  +  5y  =  0 

3.     13x+17y  =  0  4.     13x  -  17y  =  0 

5.     17x+13y  =  0  6.     1322x  -  1263y  =  0 

C^.     1.     (a)   Is  it  the  case  that,   for  each  integer  k,    (10k,    -6k)  satisfies 
the  equation  46x  +  lOy  =  0'? 

(b)  It  is  the  case  that,   for  each  ordered  pair  of  integers  which 
satisfies  46x  +  lOy  =  0*,   there  is  an  integer  k  such  that  the 
ordered  pair  is  (10k,    -6k)? 

(c)  Repeat  (a)  and  (b)  for  the  equation  *3x  +  5y  =  0'  and  *(5k,  -3k)*.. 


•^ 
*"i^ 


In  answering  Exercise  1(a)  of  Part  C,    you  probably  saw  that  the  set 
of  all  ordered  pairs  (10k,  -6k)  is  a  subset  of  the  set  of  ordered  pairs  of 


[7-126] 


[7.06] 


integers  which  satisfy  *6x  +  lOy  =  0'.     That  is,  you  probably  saw  that 

{(x,  y):  3k  (x  =  10k  and  y  =  -6k)}  C  {(x,  y)  €  I  X  I:   6x  +  lOy  =  0}. 

But,   when  you  did  Exercise  1(b),   you  should  have  discovered  that 

{(x,  y):  3k  (x  =  10k  and  y  =  -6k)}  /*  {(x,  y)  e  I  XI:   6x  +  lOy  =  0}. 

For  example,  (5,  -3)  belongs  to  the  second-named  set  but  not  to  the  first. 

2.     True-or -false? 

[x,y):   3k(x=  12k  and  y  =  -15k)}  C  {(x,y)e  IXI:    15x+12y  =  0} 

>,  y):  3k(x=  12k  and  y=  -15k)}  =  {(x,  y)  €  I  X  I:    15x+12y  =  0} 

[x,  y):  3k(x  =  4k  and  y=  -5k)}  =  {(x,  y)  €  I  X  I:    15x  +  12y  =  0} 

Ix,  y):  3k(x=-4k  and  y  =  5k)}  =  {(x,y)  6  I  X  I:   15x  +  12y  =  0} 

[x,y):  3k(x  =  4k  and  y  =  5k)}  =  {(x,  y)  €  I  XI:   15x-  12y  =  0} 

;x,y):  3k(x  =  584k  and  y=  -626k}  =  {(x,  y)  e  I  XI:  626x  +  584y=0} 


(a) 

{( 

(b) 

{( 

(c) 

{( 

<d) 

{( 

<e) 

{( 

(£) 

{( 

3.  Change  the  left  side  of  the  sentence  in  Exercise  2(f)  to  make  a 
true  sentence. 

4.  Complete  the  following  statement: 

{(x,  y):   3k(x=      and  y  =    )}  =  {(x,  y)  6  I  X  I:    1 162x  +  938y  =  0} 
[Hint.    HCF(1162,   938)  =  14] 


_D.    1.     How  many  ways  can  you  make  change  for  75£  using  nothing  but 
nickels  and  dimes? 

2.  Repeat  Exercise  1  for  73£. 

3.  Repeat  Exercise  1  for  $3.75,   but  this  time  use  nothing  but  quarters 
and  halves . 


[7.06]  [7-127] 

^DIOPHANTINE  PROBLEMS 

In  order  to  solve  the  exercises  of  Part  D  of  the  Exploration  Exercises 
you  had  to  find  how  many  ordered  pairs  (x,   y)  of  nonnegative  integers 
satisfy: 

(1)  5x  +  lOy  =  75 

(2)  5x  +  lOy  =  73 
and  : 

(3)  25x  +  50y  =  375 

Problems  which  amount  to  finding  the  integral  solutions  of  equations  are 
called  'Diophantine  problems'  after  the  Greek  mathematician,   Diophantus 
[circa  250  A.D.]. 

Equation  (1)  is  equivalent  to  'x  =  15  -  2y'  and  it  is  obvious,   by  sub- 
stitution,  that  the  nonnegative  integral  solutions  are   (15,   0),    (13,    1), 
(11,   2),   (9,    3),   (7,   4),  (5,    5),   (3,   6),   and  (1,    7).     So,   there  are  eight 
ways  to  make  change  for  75 £  using  nothing  but  nickels  and  dimes. 

In  Exercise  2  of  Part  D,   you  probably  answered,   without  having  to 
do  much  thinking,   that  you  can't  make  change  for  73£  using  nothing  but 
nickels  and  dimes.     Looking  at  equation  (2)  you  see  that,  for  all  ordered 
pairs  (x,  y)  of  integers,    5J5x  +  lOy.     Since  5/[73,  there  are  no  integers 
x  and  y  such  that  5x  +  lOy  =  73. 

Equation  (3)  is  equivalent  to  equation  (1)- -making  change  for  75£ 
using  nickels  and  dimes  is  essentially  the  same  problem  as  making  change 
for  $3.75  using  quarters  and  halves. 

Here  is  a  fourth  exercise  like  those  of  Part  D: 

A  bit  is  a  coin  worth  12y£.     In  how  many  ways  can  you 
make  change  for  75£    using  only  nickels  and  bits? 

This  problem  amounts  to  finding  the  number  of  nonnegative  integral 
solutions  of: 

5x  +  12~y  =  75 

This  equation  is  equivalent  to: 

(4)  2x  +  5y  =  30 

As  we  shall  see,   we  can  find  all  the  integral  solutions  of  equation 
(4).      Then,    we  can  solve  the  nickel-and-bit  problem  by  counting  the 


[7-128]  [7.06] 

number  of  nonnegative  integral  solutions.     The  first  step  is  to  notice 
that  (3,    -1)  is  a  solution  of  the  equation: 

2x  +  5y  =  1 

So,    (3  •  30,    -1  •  30)   is  a  solution  of  (4).      Since  2  •  90  +  5  •  -30  =  30,    it 
follows  that  if  2x  +  5y  =  30  then 

2x  +  5y  =  2*90  +  5*  -30. 
So, 

(5)  2(x  -  90)  +  5(y  +  30)  =  0. 

Now,   if  x  and  y  are  integers,  then  so  are  x  -  90  and  y  +  30.    Your  work 
in  Part  C  of  the  Exploration  Exercises  suggests  that,  since  HCF(2,  5)  =  1, 
the  integral  solutions  of  (5)  are  just  the  solutions  of: 

3k  (x  -  90  =  5k  and  y  +  30  =  -2k) 

In  order  that  x  >  0  and  y  >  0  we  must  have  90  +  5k  >  0  and  -30  -  2k  >  0 
--that  is,    k  must  be  an  integer  such  that 

-18  <  k  <  -15. 

So,  there  are  four  ways  to  change  75£  using  nothing  but  nickels  and 
bits . 


'•x     *"|H     *v 


The  key  points  in  finding  the  integral  solution  of  (4)  were,    first, 
finding  some  solution  of: 

2i  +  5j  =  1 

and,    second,   finding  all  solutions  of: 

2i  +  5j  =  0 

We  have  already  proved  that,  because  H(2,  5)  =  1,  the  first  equation 
does  have  solutions.      And  the  Euclidean  algorithm  gives  us  a  way  of 
finding  one.      [It  is  often,   as  in  this  case,    easier  to  guess  solutions  for 
such  equations  than  it  is  to  use  the  algorithm --but,    one  always  has  the 
algorithm  to  fall  back  on.  ] 

We  now  need  to  check  that,   again  because  H(2,    5)  =  1,   the  second 
equation  is  equivalent  to  the  sentence: 

3,    (i  =  5k  and  j  =  -2k) 


[7.06] 


[7-129] 


This  is  a  consequence  of 


Theorem  129. 


V     V    rHCF(m,  n)  =   1 
m    n  L 


V.  V.  [mi  +  nj  =  0  <=>  3k(i  =  nk  and  j  =  -mk)]] 


In  order  to  prove  Theorem   129,    it  is  helpful  to  prove,    first: 


Theorem   128, 

V     V   V    [(HCF(m,  n)  =   1  and  m  Ink)  = 
m    n    k  L                                                  ' 

>   m|k] 

To  say  of  two  positive  integers  m  and  n  that  H(m,  n)  =   1  amounts  to 
saying  that  m  and  n  have  no  common  factor  other  than  1.     Two  such 
integers  are  commonly  said  to  be  relatively  prime.     Theorem  128  says 
that  if  a  positive  integer  divides  a  product  and  is  relatively  prime  to 
one  factor  then  it  divides  the  other  factor. 

Notice  that,   in  view  of  the  application  we  wish  to  make  of  Theorem 
129,    we  have  extended  the  meaning  of  '  |  '  so  that,    for  example,    43j— 6* 
is  to  make  sense.     In  fact,    we  shall  define    4  |'    so  that  it  is  true  that 
3|-6: 

=>  3k  J  =  mk] 


V     V.  [m  1  j 
m    j  L      ,J 


Now,    let's  prove  Theorem   128. 

Suppose  that  HCF(m,  n)  =  1  and  that  m  |nk.    It  follows  that  there  are 
integers  i  and  j  such  that 

(*)  mi  +  nj  =   1 

and  an  integer  kQ  such  that 

(**)  nk  =  mk0. 

From  (*)  it  follows  that  mki  +  nkj  =  k. 

From  this  and  (**),    mki  +  mkQj  =  k.      So, 

k  =  m(ki  +  k0j). 

Hence  [since  I  is  closed  with  respect  to  multiplication  and  addition], 

m       k. 


[7-130]  [7.06] 

We  can  now  prove  Theorem  129: 

V     V    [HCF(m.  n)  =  1=> 

m    n  <- 


V.  V.  [mi  +  nj  =  0  <=>  3k  (i  =  nk  and  j  =  mk)]] 

To  begin  with,    we  notice  that,    whether  HCF(m,  n)  =   1  or  not, 

m(nk)  +  n(-mk)  =  mnk  -  mnk  =  0. 

So,    for  any  integers  i  and  j, 

3,    (i  =  nk  and  j  =  -mk)1^   mi  +  nj  =  0. 

Now,    suppose  that  HCF(m,  n)  =  1 .     Suppose  that  mi  +  nj  =  0.    Then, 
mi  =  n  •  -j  and,  since  m  Jmi,  it  follows  that  m  jn  °  -j.    Since  HCF(m,  n)  =  1, 
it  follows,   by  Theorem  128,   that  mj-j.     So,    by  definition,   there  is  an 
integer  k  such  that  -j  =  mk.     Since  j  =  -mk  and  mi  +  nj  =  0,  mi  -  mnk  =  0. 
Hence,    since  m  ^  0,    i  =  nk.     Consequently  [assuming  that  HCF(m,  n)  =  1], 

mi  +  nj  =  0  ^=>  3,    (i  =  nk  and  j  =  -mk). 

Combining  the  two  results  displayed  above,    and  generalizing,    we 
have: 

V.  V.  [mi  +  nj  =  0  <=>  3     (i  =  nk  and  j  =  -mk)] 

1    J  ^ 

So,    recalling  the  assumption  that  HCF(m,  n)  =  1 ,    we  see  that  we  have 
proved  Theorem  129. 


J,      +.K      .A. 
T»      .y.      ..,> 


We  have  seen  how  the  Euclidean  algorithm  and  Theorem  129  can 
be  used  to  find  all  integral  solutions  (x,  y)  of  linear  equations  of  the 
form: 

mx  +  ny  =  p 

Such  an  equation  has  an  infinite  number  of  integral  solutions  if 
HCF(m,  n)  |p,    and  no  integral  solutions  if   HCF(m,  n)/j/p.      [Finding 
integral  solutions  of  such  equations  is  sometimes  called  solving  linear 
Diophantine  equations .  ] 

Example.     Solve:    35i  -  42j  =  14 

Solution.     The  given  equation  is  equivalent  to: 
(1)         5i  +  6--j  =  2, 


[7.06]  [7-131] 

and  HCF(5,  6)  =  1 .      First,   we  find  a  solution  of: 

(2)  5i  +  6--j  =  1 

Using  this  solution,   we  can  immediately  find  a  particular 
solution  of  (1).     Then,   Theorem  129  gives  us  all  solutions  of: 

(3)  5i  +  6-  -j  =  0 

By  combining  these  with  the  particular  solution  of  (1),   we  get 
all  solutions  of  (1). 

By  inspection  [or  the  Euclidean  algorithm], 

(2')  5-  -1  +  6-  1  =  1. 

So, 

(1')  5-  -2  +  6-2  =  2. 

Thus,   (-2,    -2)  is  a  particular  solution  of  (1). 
By  Theorem  129,   (3)  is  equivalent  to: 

(3')  3k  (i  =  6k  and  j  =  -5k) 

Consequently,   the  solutions  of  (1)  are 

(-2  +  6k,    -2  -  5k),   for  k  €  I. 

^EXERCISES 
A.    Find  all  solutions  of  each  of  the  following  equations. 

1.     65i  +  77j  =  200  2.     50i  -  63j  =  75  3.     33i  +  1 9 j  =  250 

B_.    For  each  of  the  following  equations  tell  how  many  of  its  solutions 
are  pairs  of  positive  integers. 

1.     78x  -  117y  =  97  2.     51x  +  85y  =  1037 

C^.    Find  the  least  integer  p  for  which  *5x  +  7y  =  p*  has  just  nine 
nonnegative  integral  solutions. 

JD.    Solve  the  following  problems. 

1.     Mr.   Schwartz  hands  a  storekeeper  a  dollar  bill  and  4  pennies 

when  paying  for  a  49-cent  article.     The  storekeeper  has  7  nickels 


[7-132]  [7.06] 

and  6  dimes.     In  how  many  ways  can  the  change  be  arranged? 

2.  When  Mr.   Perez  cashed  a  check  at  the  bank,    the  teller  mistook 
the  number  of  dollars  for  the  number  of  cents,    and  vice  versa. 
Without  noticing  the  error,    Mr.    Perez  pocketed  the  money  and 
left  the  bank.    He  purchased  an  article  for  $3.50  and  then  noticed 
that  he  had  twice  as  much  money  left  as  the  check  had  called  for. 
For  what  amount  had  the  check  been  written? 

3.  A  stick  is  divided  into  sections,  all  of  the  same  length,  by  drawing 
36  red  lines  across  it.     It  is  also  divided  uniformly  by  28  green 
lines.      Counting  from  a  given  end  of  the  stick,    which  red  line  comes 
the  least  distance  after  one  of  the  green  lines?      Which  red  line 
comes  the  least  distance  before  one  of  the  green  lines? 

4.  A  manufacturer  uses  three  kinds  of  components --googels,    goggels, 
and  gaggels--in  assembling  a  biggel.     For  each  kind  of  component, 
the  unit  cost  is  a  whole  number  of  cents.      Four  googels  and  five 
goggels,   together,    cost  25£  more  than  three  gaggels.    Two  googels, 
four  goggels,    and  three  gaggels  cost  38£.     Each  biggel  contains 
seven  googels,    eleven  goggels,    and  six  gaggels.     What  is  the  total 
cost  of  the  components  of  a  biggel? 

5.  Three  robbers  stole  a  sack  of  spade  guineas.     As  they  were  about 
to  divide  their  loot,    l/2   to  the  first  man,     l/3   to  the  second,    and 
1/6  to  the  third,   they  were  surprised  by  the  King's  foresters.     The 
robbers  were  able  to  escape  with  all  the  money,    each  man  carrying 
a  portion  of  it.     Meeting  again  in  a  safer  place,    each  robber  turned 
over  a  part  of  what  he  had  carried  away;    the  first  man  kept  half  of 
what  he  had,   the  second,   two-thirds,    and  the  third,   five-sixths. 
This  money  was  then  shared  equally  among  all  three.     As  it  turned 
out,   each  robber  got,   in  all,   the  same  amount  as  he  would  have,    if 
the  foresters  had  not  interfered.     What  is  the  smallest  number  of 
coins  which  the  sack  could  have  contained? 


[7-133] 


REVIEW  EXERCISES 


1.     Consider  the  number  system  consisting  of  the  set  S  of  three  numbers 
0,    1,   and  2,   and  the  two  operations  ft  and  <I>  which  are  defined  in  the 
following  tables  : 


a 

0 

1 

2 

0 

0 

1 

2 

l 

1 

2 

0 

2 

2 

1 

0 

1 

4> 

0 

1 

2 

0 

0 

0 

0 

1 

0 

1 

2 

2 

0 

2 

1 

In  the  following  questions,    'x\    'y',   and  'z'  are  variables  whose 
domain  is  S. 


(a)    True-or-false? 
(1 


(3 

(4 

(5 

(6 

(7 

(8 

(10 

(12 

(13 

(14 


V    V    x  ■&  y  =  y  ft  x 
x    y  7        7 


(2)     VV    x<ty=y4>x 

x    y  7        7 


VVV  x  ft  (y  ft  z)  =  (x  ft  v)  ft  z 

x    y    z  7  7 

VVV  x  «$•  (y  <t>  z)  =  (x  4>  y)  <!>  z 

x    y    z  7  '  y  3 ' 

VVV  (x  ft  y)   4>  z  =  (x  <fr  z)  ft  (y  <d>  z) 

V  V    V  x  <J>  (y  &  z)   =  (x  <t>  y)   ft  (x  <J>  z) 

X      y       Z 

V  V    V  (x  <fc  y)  ft  z  =  (x  ft  z)  <t>  (y  ft  z) 

X      y       Z 


3      V     v  ft  x  =  V 

x     y  7  7 

V    V    3     x  ft  z  =  v 
x    y      z  7 

V/nV3     x  4>  z  =  y 
x/  0     y      z  7 

VVV    [x  ft  y  =  x  ft  z 
x    y    z  ' 

VVV    [x4>y  =  x<tz 
x    y    z  L  7 


(9)     3      V    y  4>  x  =  y 

x     y  7  7 

(11)     VV    3     x  4>  z  =  y 
x    y      z  7 


y  =  z] 
y  =  z] 


(b)    Notice  that    V   V    x  ft  y  =  the  remainder  on  dividing  x  +  y  by  3. 

X      V 


Complete:      V   V    x  <$>  v  - 

r  x    y  7 


[7-134] 

2.     Suppose  that  S  =  {0,    1,    2,    3,    4,    5}   and  that,    for  each  xe  S   and 
each  ye  S, 

x  ft  y  =  the  remainder  on  dividing  x  +  y  by  6, 
and  x  &  y  =  the  remainder  on  dividing  xy  by  6. 


(a)    Complete  the  following  tables 


ft 

0 

1 

2 

3 

4 

5 

0 

1 

2 

3 

4 

5 

. ,  . 

.  ,   . 

* 

0 

1 

2 

3 

4 

5 

0 

1 

2 

3 

4 

5 

(b)   (1)   Is  ft  commutative? 

(2)  Is  <i>  commutative? 

(3)  Is  the  same  number  in  S  listed  twice  in  any  row  of  the  table 
for  ft? 

(4)  Your  answer  for  part  (3)  tells  you  of  a  principle  for  ft.     What 
principle? 

(5)  Is  each  number  in  S  listed  in  each  row  of  the  table  for  ft? 

(6)  Your  answer  for  part  (5)  tells  you  of  a  theorem  about  ft,    State 

the  theorem: 

V   V    3 
x    y      z 

(7)  Find  all  solutions  of  each  of  the  following  equations. 

(i)    5  <t>  x  =  3  (ii)    2  *  x  =  0  (iii)    2  4>  x  =  3 


(8)    Is  there  a  cancellation  principle  for  4>  ? 


[7-135] 


3.     Simplify. 

(a)  (7x3  -  3x2  +  2x  -  5)  +  (2x3  +  7X2  -  5x  +  1) 

(b)  (8x4  +  2x2  -  x  +  3)  +  (9x4  +  3x3  -  2x  +  7) 

(c)  (8y  -  2y3  +  4y4  -   1)  +  (7  -  2y*  +  Qy*  -  7y4) 

(d)  (9a3  -  3a2  +  2a  -  5)  -  (7a3  +  5a2  -  6a  +  7) 

(e)  (5x4  -  3x3  -  2)  +  (7x3  -  3x2  +  1)  -  (8X2  -  2x3  +  7) 

(£)  (3x2  -  2x  +  l)(5x2  +  3x  -  2)  -  (3x  -  2  +  5x2)(3x2  -  2x  +  1) 


4.     Expand  and  simplify. 

(a)    (7x2  -  2x  +  l)(5x  -  3)  (b)   (8x4  -  x2  +  3)(2x3  -  3x  +  1) 

(c)   (2y6  +  7y3  +  3y)(4y3  +  2^  +  7)  (d)   (5a4  +  3a  +  4)(7a3  +  2a2  +  2) 

(e)    (5x3  +  3x  +  2)(  10x2  +4)  (f)    (4x4  +  3X2  +  x  -  4)(5x3  +  8x  +  2) 


Simplify  by  using  the  division -with -remainder  algorithm. 
lOx5  -  15x4+  4x3  -  12x2  +  23x-  21 


(g) 


2x  -  3 


...    4x4  +  2X3  -8X2  +  2 
(h)  2TT1 


5. 


If  the  volume-measure  of  a  circular  cone 
2x  .  3       is  (16x3  -  28x2  -  24x  +  45)tt  and  the  radius 
of  the  base  is  2x  -  3,   what  is  the  measure 
of  the  height? 


6.  Show  that  4x  -  2'  is  a  factor  of  *3x3  -  2X2  +  7x  -  30'  without  using  the 
division-with-remainder  algorithm. 

7.  One  factor  of  l6x4  +  15x3  -  8x2  -  6x  +  35'  is  *2x  +  5'.    What  is  another? 

8.  The  equation  46x4  -  x3  -  82x2  +  81x  +  36'  has  four  roots.    Two  of  them 
are  3  and  -4.     What  are  the  other  two? 


9.     Use  definitions  such  as  '4  =  3  +  1'  and  algebra  theorems  to  prove 
each  of  the  following. 


(a)    3  +  2  =  5 


(b)    5  -  2  €  P 


[7-136] 


10.    Prove  these  generalizations. 


(a)    V     q(q  +  4)(q  +  5)    +         +  2)        +  5)   .    (q  +   l)(q  +  5)(q  +  6) 


(b)    v      (q  -    l>q(q  +   l)(q  +  2)    +  q(q  +   1)(q  +  2)   =    q(q+lHq+2)(q+3) 


,        v     q(q  -  l)(2q  +  5)    +  {q  +  1}2  .  x  =    (q  +   DqUq  +  7) 
q  6  o 


(d)   V      q(q  +  l)    + 
ya)      q    2(2q  +   1) 


(q  +  IV 


_    (q  +    l)(q  +  2) 


q   2(2q  +   1)     '    ( 2q  +  l)(2q  +  3)  2(2q  +  3) 


(e)    V     q(6q2+15q+ll)    +  (3q  +  4)2  =    (q  +  D[6(q  -H)2  +   15(q+l)  +   11] 


11.    For  each  of  the  listed  sets,    tell  whether  or  not  it  is  closed  with 
respect  to  each  of  the  given  operations. 


(a)  the  set  of  positive  integers 

(b)  the  set  of  negative  integers 

(c)  the  set  of  nonzero  rational  numbers 

(d)  the  set  of  nonnegative  numbers 

(e)  the  set  of  irrational  numbers 

(f)  the  set  of  nonzero  integers 

(g)  the  set  of  real  numbers 


+  X  -  -r 


12.    (a)   Suppose  the  domain  of  'x'  and  4y'  is  the  set  of  all  people  and  '>' 
means  the  same  thing  as  'is  older  than'. 

(1)  Which  of  Theorems  86a,    86b,    and  and  86c  still  hold? 

(2)  Does  Theorem  87  still  hold? 


(b)    Suppose  the  domain  of  'x'  and  4y'  is  the  set  of  all  people  and  *>' 
means  the  same  thing  as  4is  the  father  of. 

(1)  Which  of  Theorems  86a,    86b,    and  86c  still  hold? 

(2)  Does  Theorem  87  still  hold? 


[7-137] 


13.    Prove  each  of  the  following  theorems. 

(a)   V   V   V   V    [(u  -  v)(x  -  y)  <  0   <=>    (v  -  u)(x  -  y)  >  0] 
x    y    u    v  L  ' 


<b>   Vx>0Vy>0 


1 


1 


=>       -    >    X 

y 


14.    True-or-false? 


(a)    V   V    not  both  u  >  v  and  v  >  u 
u    v 


(b)  VxVy[x  -  3  >   y=>  x  -  y  >   0] 

(c)  V   V   V    [(x  >  y  and  x  <   z)  =>   z  >  y] 

x    y    z  L  7 


(d)    V     2x  >  x 

x         ~~ 


(e)  y 


x^O 


Jl  <  I 

x2    -    x 


"] 


(f)  V  <     ,  V  .  . 

x<  -2     y  >  1 

(g)  Vu  (u  +  2)2  >  u2 


4-x2   >    4-x2 
x  xy 


(h)    V   V     u2  +  v2  >  uv 

x     '        U     V  "" 


(i)    Vu>0Vv>0 
0)    VuVvCu2>  V 


<       =>    u  "  v    >    u  "  vl 

VV  VU  J 

U  >    V  >    -u J 


2     <*- 


15.    Prove: 


(a)   V     1  -   I  +    .      * 

n  n        (n  +  \)c 


<   1  - 


1 


n+  1 


(b)    V     ~  -  -r  <   -4t 
'      n    n        n*         n  +  1 


(c)  y 


n 


n    n2+  1 


(d)   y 


1 


1 


< 


1 


n    n+  1        (n+  l)2         n+2 


16.    Suppose  that  quadrilateral  ABCD  is  a  nonsquare  rectangle,  and  that 
quadrilateral  BDEF  is  a  square.      Show  that  the  area-measure  of 
the  rectangle  is  less  than  the  area-measure  of  the  square. 


17.    Prove  that  if  a,    b,    c,    and  d  are  nonnegative  and  c  +  d  <  a  <  b  then 
ad  +  be  <  ab. 


[7-138] 

18.  Prove  that  the  equation  'x2  +  1  =  0'    is  not  satisfied  by  any  real 

number. 

19.  Prove:      V   V    x4  +  y4  >   2xV 

x    y  3     ~~  ' 

20.  Arrange  in  order  from  largest  to  smallest. 

1  l         n       "6         9  7         14         9  2  1 

5'     -5'        '      29'     25*     30'     50'     17'     -7'     2 

21.  Solve  each  of  the  following  inequations. 

(a)    3x  -  2  >  9  -  5x  (b)    5(x  -  2)  <   3(4  -  x) 

(c)   x2  +  7x  >  44  (d)    ^j    <   2 

22.  In  a  basket  of  oranges  and  grapefruit,   the  number  of  grapefruit  is 
2  greater  than  the  number  of  oranges.      If  the  number  of  oranges 
were  doubled,    there  would  still  be  less  than  a  dozen  pieces  of  fruit 
in  the  basket,    but  if  the  number  of  grapefruit  were  doubled,   there 
would  be  more  than  a  dozen.     How  many  oranges  and  grapefruit  are 
there  in  the  basket? 

23.  In  each  of  the  following  exercises  you  are  given  a  set  S  and  a  map- 
ping f.     In  each  case  consider  the  generalization: 

Ve[eeS=^  f  (e)  €  S], 

and  answer  'true'  or  'false'. 

(a)  S  is  the  set  of  even  numbers;   f  :  n  -*  n  +  12 

(b)  S  =  {x:    3     x  =  2n  -  1};    f  :  n  —  n  +  12 

(c)  S  =  {m:    3     m  =  3n};    f  :  n  —  n  +  20 

n  * 

(d)  S  =  {m:     Vn  m  £  3n};    f  :  n  -*  n  +  20 

(e)  S  =  {i:     3     i  =  3j};    £:i  — i  +  6 

(f)  S  =  {i:     V     if  3j};    f  :  i  -*  i  -  6 

(g)  S  =  {x:    x  >  5};    f  :  x  —  x  +  1 


[7-139] 

(h)   S  =  {x:  x  is  a  prime  number};   f  :  x  -*-  x  +  2 

1  +  x 

(i)    S  =    1,  3;    f  :x-  i~-^ 

(j)    S=  {1};    f:x-I±*. 

(k)   S  =  {x:  x  <  0};    f  :  x  —  -i-j-*- 

(i)   S  =  {x:  3p  3q^Q  qx  =  p};    f  :  x  -  i-±-*- 

(m)  S  =  {x:  V    /  Q  qx  4  I};    f  :  x  —       2  * 

(n)   S  =  {(x,  y) :  x  >  0  and  y  >  0};    f  :  (x,  y)  -  (x,   y  +  2) 

[Now,   repeat  part  (n)  for  Quadrant  II,    Quadrant  III,   and 
Quadrant  IV.] 

(o)   S  =  {(x,  y) :  x  =  7};  f  :  (x,  y)  -  (x,   y  +  2) 

(p)   S  =  {(x,  y) :  x  <  y  and  3.   y  =  2i};    f  :  (x,  y)  -  (x,   y  +  2) 

(q)   S  =  {(x,  y):  y  >  x2};   f  :  (x,  y)  -  (2x,  4y) 

(r)   S  =  Quadrant  I;   f  :  (x,  y)  —  (x  -  2,   y  -  3) 

[Repeat  part  (r)  for  each  of  the  other  three  quadrants.] 

(s)  S=  {(x,  y):   2y-3x  =  7};   f  :  (x,  y)  -  (x  -  2,   y  -  3) 

(t)  S  =  {(x,  y):   2y-3x=  7}  w  {(x,  y) :   3x  -  2y  =  9};   f  :  (x,  y)  -  (x  -  2,  y  -  3) 

(u)  S  =  {(x,  y) :   4x  -  5y  =  7  or  4x  -  5y  =  14};  f  :  (x,  y)  —  (x  -  2,  y  -  3) 

(v)  S  =  {(x,  y) :   4x  -  5y  =  7}  r\  {<x,  y) :   4x  -  5y  =  14};  f  :  (x,  y)  -  (x  -  2,  y  -  3) 

(w)  S  =  {(x,  y) :   3p  4x  -  5y  =  7p};    f  :  (x,  y)  -  (x  -  2,   y  -  3) 

(x)  S  =  {(x,  y):  y  >    |x|};   f  :  (x,  y)  -  (-x,    2y) 

(y)  S  =  {(x,  y):   x2  +  y2  =   l};   f  :  (x,  y)  -    (^L+JL,     *jJL\ 

(z)    S  =  {(x,  y):  xy  =  0  or  y2  =  x2};   f  :  (x,  y)  -  (££4=I>     ^j^) 


[7-140] 


24.    Here  are  the  first  four  numbers  in  the  sequence  C, 


•  »  • 


13 


(a)  Study  the  formation  pattern  and  try  to  describe  it  in  a  recursive 
definition  of  C. 

(b)  Guess  an  explicit  definition  of  C,   and  derive  it  by  mathematical 
induction  from  the  recursive  definition. 

25.    Here  are  the  first  four  numbers  in  the  sequence  D. 


•  ••  • 

•  •  • 


•  •  • 

•  •♦• 
•••••••• 

•••••••••• 

•••••• 

•  •  • 


i 


8 


21 


40 


(a)  D     =   1  and,   for  each  n,    D  =  S     ,      +  4T    ,   where  S  and  T  are 
%   '       l                                                   n+in+i  n 

the  sequences  of  square  numbers  and  triangular  numbers,    re- 
spectively.      Use  the  recursive  definitions  of  S  and  T  to  find  the 
recursive  definition  of  D: 

Di   =    1 

V      D  =    D     + 

n       n  +  i  n 

(b)  Derive  an  explicit  definition  of  D. 


26.    Let  f  be  a  function  which  is  defined  recursively  by: 


V       f     .       =  f     +  3 

n       n+  l  n 


[7-141] 

(a)  Compute   f3,    f6,    and  f12  -  f10. 

(b)  Discover  an  explicit  definition  of  f  and  derive  it  from  the  recur 
sive  definition. 


27.  (a)   Given  n  points  on  a  circle,  how  many  chords  have  these  points 

as  end  points  ? 

(b)  Given  7  points  on  a  circle,    such  that  no  three  of  the  chords  which 
have  these  as  end  points  are  concurrent  inside  the  circle,   how 
many  intersections  do  the  chords  have  inside  the  circle? 

(c)  Repeat  part  (b)  with  'n'  in  place  of  47'. 

28.  (a)   In  how  many  ways  can  you  put  seven  silver  dollars  in  three 

pockets  ? 

(b)   Solve  part  (a)  under  the  additional  condition  that  no  pocket  be 
empty. 


135 
29.    Show  that,   for  each  n,   n  >    -«-  if  and  only  if  n  >  5 


30.    Give  an  example  of  a  set  of  real  numbers  which  has 

(a)  a  lower  bound  but  no  upper  bound; 

(b)  a  lower  bound  but  no  least  member. 


1    -  n 

31.    Does    {x:    3     x  =  — j—}  have  a  least  member?    A  greatest  member? 
1  n  n*     '  & 

A  least  upper  bound? 


32.    Suppose  that  f  and  g  are  defined  by: 

f  x  =  4  f  Bl  =   2 

and:  / 

VfL=2f+3  V      g     L      =    3g 

nn+i  n  1^  n    fen  +  l  &n 

Prove  each  of  the  following  theorems  by  mathematical  induction 

[or  Theorem  114]. 

(a)   V     g     >  2  (b)    V   w   g     -  f     >   1 

n°n—  *   '      n  >  4   &n        n 


[7-142] 


33.    Compute. 

(a)   13.11   +   12.9] 
(c)    [9.  28]]   +    49.28}} 


(b)    J -3.  5  J    -    J  2*1 
(d)    H»]H   +    €1*11 


34.    What  ordered  pairs  belong  to  {(x,  y) :   4y  -  3x  =  0}  r\  {(x,  y) :  y  =  [xj}? 


35.    Solve: 

(a)    [4.8]   +   11*1   <    7.6 


(b)    J2.891  +  Ixl  <   [2.89  +  xl 


36.    Prove: 


(a)    Vx[xl    <      x+  4-  (b)    V 


X+   2 


X+    2 


<  |[xl  +  1 


37.  Solve:    n|n2  +  1 

38.  For  each  number  listed,   tell  how  many  factors  it  has. 

(a)    23'34  (b)    1000  (c)    32  •  52  •  72 

39.  Find  ail  the  integer  solutions  of  *31x  -  25y  =  6'. 

40.  What  is  the  highest  common  factor  of  298  and  746? 

41.  A  farmer's  livestock  consists  of  just  cows  and  chickens.  The  total 
number  of  heads  and  legs  is  699.  If  he  has  more  than  200  chickens 
and  more  than  15  cows,  how  many  of  each  animal  does  he  have? 

42.  If  a  10-ounce  can  of  car  wax  costs  60  cents  and  a  26 -ounce  can  costs 
$1.43,   how  much  do  you  save  per  ounce  by  buying  the  larger  size? 


43.    Simplify. 


(b)       i-x 


2y+  1         3 


[7-143] 


2  7 

44.    Solve  the  equation:      >2x.  \)Z    "    (2x  -  1)   +   ^  =  ° 


s  +  1  r 

45.    If  r  and  s  are  positive  integers,   which  is  larger,    — - —  or  ,  ? 

s  r  t  i 


46.  Suppose  that  AABC  is  right-angled  at  C.     If  the  measures  of  the 
medians  from  A  and  B  are  13  and  2V  34 ,   respectively,   what  is  the 
perimeter  and  what  is  the  measure  of  the  third  median? 

<-> 

47.  A    yS^"  "^Xt*  Given:     PT  is  a  tangent, 

PA  =  12, 
PT  =  6 


Find:        PB 


48.  A  handtruck  can  hold  either  10  boxes  of  plums  or  15  boxes  of  cherries 
If  a  stock  clerk  wants  to  load  it  with  as  many  boxes  of  plums  as  cher- 
ries,  what  is  the  largest  number  of  boxes  of  each  kind  he  can  load? 

49.  Solve  these  equations: 

(a)    24=0.25%(x)  (b)    ~%(l600)=x 

o 


50.    Simplify, 
(a)     £*Ll 


i-i.i 


y/Tl  +   V48  /u\"V/l  1  ,   \  9      5 

— —  (b)    V25     "    TTq  (c)     5 

/8   +    vT2  Lb  lb9  1+4 


18 


51.  The  width  of  a  rectangle  is  75%  of  the  length.     If  the  dimensions  are 
increased  in  such  a  way  that  the  new  rectangle  is  similar  to  the  old 
one,   but  its  perimeter  is  increased  by  21  and  its  area-measure  is 
increased  by  63,   find  the  dimensions  of  the  original  rectangle. 

52.  Solve  these  equations: 


[7-144] 

53.    The  hypotenuse  of  a  right  triangle  is   1  foot  longer  than  one  of  the 
legs  and  its  perimeter  is  6.     What  is  its  area? 


54.    Simplify. 

,    .    3  5  7 

(a)   4    "   II  +    20 


{b)  T  +  — 


3m 

To" 


(c) 


1.2  X  0.  1 
62 


55.    There  are  n  players  in  an  elimination -type  checkers  tournament. 
How  many  individual  matches  must  be  played  to  determine  the 
winner? 


56.  r-    ^-srT-^  The   latitude   of  Chicago  is  approxi- 
mately 42°  ,   and  the  radius  of  the 
earth  is  about  3960  miles.      How  far 
is  Chicago  from  the  earth's  axis? 

57.  Pete  walks  from  A  to  B  at  the  rate  of  4  miles  per  hour  and  rides 
from  B  to  A  at  the  rate  of  7  miles  per  hour.      If  the  round  trip 
took  Sj  hours,   what  is  the  distance  between  A  and  B? 

58.  Find  four  consecutive  integers  such  that  the  product  of  the  two 
smallest  differs  from  the  product  of  the  two  largest  by  the  sum  of 
the  four. 


59.    Solve  the  system 


60.    What  is  the  area  of  an  isosceles  triangle  whose  base  is   10  inches 
long  and  whose  congruent  sides  are  each  13  inches  long? 


61.    A  man  inherits    1/3    of  his  father's  estate,    and  his  sister  inherits 

l/5    of  it.     The  balance  of  $14000  was  left  to  a  charitable  institution, 
How  many  dollars  did  the  man  inherit? 


[7-145] 


BASIC  PRINCIPLES  AND  THEOREMS 

Commutative  principles  for  addition  and  multiplication 

VVx  +  y  =  y  +  x  VVxy=yx 

x    y  7       7  x    y     7       7 

Associative  principles  for  addition  and  multiplication 

VVVx+y+z  =  x+(y+z)  VVV    xyz  =  x(yz) 

xyz  7  '  xyz7  7    ' 

Distributive  principle  [for  multiplication  over  addition] 

VVV    (x  +  y)z  =  xz  +  yz 
xyz  7'  7 

Principles  for  0  and  1 


V    x 
x 

+  0  = 

X 

V 

X 

Xl    = 

=    X 

1/0 

Principle 

of 

Opposites 

Principle  for 

Subtraction 

V    x 

X 

+  - 

x  =  0 

V   V    x  - 
x    y 

y  =  x  + 

-y 

P: 

rinciple 

of  Quotients 

V   V 
x    y 

/o 

X 

7y 

=   X 

* 

1-  VxVy  Vz  x(y  +  z)  =  xy  +  xz  [page  2-60] 

2.  Vx  lx  =  x  [2-61] 

3-  vv  Y.  V.  V.  ax  +  bx  +  ex  =  (a  +  b  +  c)x  [2-61] 
x    a    d    c 

4'  VxVy  VaVb  <ax><by>  =  <ab)<xY)  [2-61] 

5-  VxVy  Va  Vb  <a  +  x>  +  (t>  +  y)  =  (a  +  b)  +  (x  +  y)  [2-61] 

6'  VxVyVz  fx  =  y==>  x  +  z  =  y  +  zl  [2-64] 

7'  VxVyV2  tx+  z  =  y  +  z=>  x  =  y]  [2-65] 

8*  VxVyVz  [x  =  y=:>  z  +  *=  z  +  y]  [2-66] 


[7-146] 


9. 
10. 
11. 
12. 
13. 
14. 
15. 
16. 
17. 
18. 
19. 
20. 
21. 
22. 
23. 
24. 
25. 
26. 
27. 
28. 
29. 
30. 


VVV    fz  +  x  =  z  +  y^^  x  =  yl 
x    y    z  L  7  ' J 

V  V    [x  =  y  =5>   -x  =  -y] 

x    y  L  7  /J 

VVV     fx  =  y  =^>  xz  =  yzl 

x    y     z  L  7  7    J 

VVV     fx  =  y  =£»    zx  =  zyl 

x    y    z  L  7  7J 

V  V    V   V    [(u  =  v  and  x  =  y)  =>  u  +  x 
u    v    x    y  L  7 

V  V    V   V    f(u  =  v  and  u  +  x  =  v  +  y)  — 
u    v    x    y  L  J 


V    xO  =  0 
x 


V    V     [x  +  y  =  0  =>    -x  =  y] 

x    y  L         }  J  J 

V X    =    X 

X 


V  V     -(x  +  y)  =  —x  +  -y 

x    y  7  7 

V  V     -(x  +   -y)  =  y  +  -x 

x    y  7  7 

V  V     -(xy)   =  x  •  — y 
x    y  '  ] 

VxVy  -(xy)   =   -xy 

V  V     [x  =  — y  =>    -x  =  yl 

x    y  L  J  7J 

V  V     —x  *  —  y  =  xy 

x    y  '  J 

V  V     — xy  =  x  •  — y 

x    y         7  ' 

VxVyVz  -x(y  +  z)  =  -(xy)  +  -(xz) 

VVV     -x(-y  +  -z)  =  xy  +  xz 
x    y    z  7  '  7 


[page 


v  +  y] 
x  =  y] 


V    x 
x 


1    =    -X 


V     -x  =  -lx 

X 


V    V     (x  +  y)  +   -y  =  x 
x    y  7  7 

VV     (x  +  y)-y  =  x 
x    y  '  7 


2-66] 

[2-66] 

[2-66] 

[2-66] 

[2-66] 

[2-66] 

[2-66] 

[2-68] 

[2-69] 

[2-69] 

[2-69] 

[2-69] 

[2-69] 

[2-69] 

[2-70] 

[2-70] 

[2-70] 

[2-70] 

[2-70] 

[2-70] 

[2-71] 

[2-71] 

31. 
32. 

33. 

34. 

35. 

36. 

37. 

38. 

39. 

40. 

41. 

42. 

43. 

44. 

45. 

46. 

47. 

48. 

49. 

50. 

51. 

52. 


VVV    x  -  yz  =  x  +  -yz 
x    y    z  '  ] 

V  V    x  -  y  +  y  =  x 
x    y  J       J 

V  V     -(x  -  y)  =  y  -  x 
x    y  "        7 

VVV    x+(y-z)=x  +  y-z 
x    y    z  w  #  y 

VVV    x-(y+z)=x-v-z 
x    y    z  '  7 

VVV    x-(y-z)=x-y+z 
x    y    z  w  /  7 

V  V    V    x+(y-z)=x-z  +  y 
x    y    z  7  '  ' 

VV  Vz  x(y  -  z)  =  xy  -  xz 
Vx  Vy  Vz  (x  -  y)z  =  xz  -  yz 
VxVy  Vz  x  -  (-y  -  z)  =  x  +  y  +  z 

Vx  Vy  Vz  Vu  X  "  (y  "  Z  "  U)  =  x  "  V  +  z  +  u 


V    0   -  x  =  -x 
x 


V    x  -  0  =  x 

X 


VxVyVzX+  z  "  ^  +  z)  =  x  "  y 
VxVy  Vz  X  "  Z  "  (y  '  z)  =  x  "  V 
Va  Vb  Vc  Vd  (a  "  b)  +  (c  "  d)  =  (a  +  c>  "  <b  +  d) 

VxVyVz'Z+y  =  X=^   z  =  x  -  y] 

vxvyvz/o  [xz  =  yz=>  x  =  y] 

V  V    ,rt  V    [zy  =  x=*>   z  =   X 1 

x    y/0      z  L    y  y J 

w      x 

V  —  =  x 

x    1 


Vo  l  =  1 


VX 

X      -  1 


[7-147] 

[page  2-71] 
[2-72] 
[2-72] 
[2-73] 
[2-73] 
[2-73] 
[2-73] 
[2-74] 
[2-74] 
[2-74] 
[2-74] 
[2-75] 
[2-75] 
[2-75] 
[2-75] 
[2-75] 
[2-89] 
[2-90] 
[2-91] 
[2-91] 
[2-91] 
[2-91] 


[7-148] 


53.    V    ,n    -  =  0 


54. 
55. 
56. 

57. 

58. 

59. 

60. 

61. 

62. 

63. 

64. 

65. 

66. 

67. 

68. 

69. 


V  V    /n  [-  =  0=>  x=  0] 
x    y/0  Ly  J 

V  V    [(x  /0andy/0)^>xy/0] 
x    y 

V  V    [xy  =  0  =>  (x  =  0  or  y  =  0)] 
x    y 

ww  ww  xiu        xv  +  uy 

x    y/=0     u    vfO    y       v  yv 

ww  ww  x       u        xv  -  uy 

x    y/*0     u    v p  0    y       v  yv 


x    y^O     u    v^O    y     v 


V  V    ,„  V   V 


yv 


V  V    /     V    ,       —  =  — 

xy/0     z/cOyz        y 

x    y^O     z^O    y  :     yT"z 

V  V  V    ,n    *£  =  ly 

x    y     Z/eO      z  z ' 


*    w  x  1 

x    y/*0    y  y 


x    y/*0      y 


V  ,nV  V     xX±2£E  =  y+z 

x/ 0     y    z  x  ' 

V  V    ,     V  V   j     V   ,     ^  =  (*  *  2?u 
x    y/*0     u    v^O     z/*0    yv        (y -f  z)v 

v  v  v  ,n  £  +  £  =  ^±x 

xyz/Oz        z  z 


V   V 


V   V 


u 


'     V      /        V      /        V      V      /         +    

x    y/0     z^O     u    vpQ    yz        vz 

V  V  V    ,n   i  -  X=  iUlX 
x    y    z/tO    z        z  z 


xv  +  uy 
yvz 


70-    VxVo  V<)VUVW0    ^ 

71.    V   V  V    ,_   x+  i=  *£±X 
x    y    z^O  z  z 


u 
vz 


xv  -  uv 
yvz 


[page  2-91] 

[2-91] 

[2-91] 

[2-91] 

[2-92] 

[2-92] 

[2-93] 

[2-94] 

U-95] 

[2-96] 

[2-97] 

[2-97] 

[2-97] 

[2-98] 

[2-99] 

[2-99] 

[2-100] 

[2-100] 

[2-100] 

72.    V   V    /n  V    /n  x-r  *-  =  x- 


x    y  ^  0     z  ^  0 


73     V   V    /     V    /     V    /       — 
fJ<      x    y^O     u^O     v^O    y 

74.    V,^V,--!-=2- 
x  ;=  0     y/=Ox/y        x 


75.    V   V    ,n  V    ,.    *-  -r  z 

x    y/=  0     Z/=  0    y 


u 

v 


XV 

yu 


x 

yz 


76.    V   V    /n 
x    y/O 


77.    V   V    yn 
x    y/*0 


78.    V   V    ,n 
x    y/O 


x 

y 

X 

y 

■X 

•y 


-X 

y 

X 

^y 

X 

y 


79.    Vx[x^O=>  -x/*0] 


80.     -0=0 


Ox     Ox     Ox 

'iN    "j"    "i" 


(Px  )     V^  [x  ^  0  =>   either  x  €  P  or  -x  €  P] 
(P2  )     V^  not  both  x  c  P  and  -x  e  P 
(P^  )     V^  V„  [(x  €  P  and  y  e  P)  =>  x  +  y  e  P] 

>   xy  €  P] 


s'    x-yi  -y 


(P4)     VxVy  [(xe  P  andyeP) 


Ox        Ox        Ox 
T»       "4X       '4V 


81.  Of/P 

82.  1  e  P 


fVxeP 
[i  >  o] 


x^O] 


[7-149] 

[page  2-101] 

[2-101] 

[2-101] 

[2-101] 

[2-103] 

[2-103] 

[2-103] 

[7-18] 

[7-18] 

[7-22] 
[7-22] 
[7-23] 
[7-24] 

[7-23] 
[7-23] 


Ox      Ox      Ox 

*-,x    *.,«.     „,* 


(G)        V    V     [y  >  x 
x    y  LJ 


y  -  x  e  P] 


Jx         Ox         Ox 

',*    -v>    „,.» 


[7-30] 


83.    V    [x  >  0 


==»    x 


€P] 


84.    V   V    [y  >  x  <=>    y  -  x  >  0] 


[7-30] 
[7-31] 


[7-150] 


85.    V    [x  <  0   <=>    -x  >  0] 


[page  7-32] 


86     a.  V   V    [x/y^>(x>yory>x)] 

—  x    y  c 

b.  V   V    not  both  x  >  y  and  y  >  x 

—  x    y 

c.  V   V    V    f(x  >  y  and  y  >  z)=>  x  >   z] 

—  x    y     z  L 

d.  VVV    [x>y=>x+z>y+z] 

—  x    y    z  L  ' 

e.  VVV    [(z  >  0  and  x  >  y)  =*>  xz  >  yz] 

—  x    y    z 


> 


[7-32] 


87.    V    x/x 
x 


[VxVy(x  =  y=>  x^  y)] 


88.  V   V    [y  >.  x   <=>  x^  y] 

x    y  L/  — 

89.  VVV    [x  +  z  >  y  +  z   <=>  x  >  y] 

x    y    z  L 


90.    V    x  +  1  >  x 
x 


[7-33] 
[7-33] 
[7-33] 
[7-35] 


91.  V   V   V   V    [(x  >  y  and  u>v)=>x  +  u>y  +  v] 

x    y    u    v 

92.  VVV    f(x  >  y  and  y  >   z)  =>  x  >   z] 

x    y    z  "  """ 

93.  V   V    [(x  >  y  and  y  >  x)  =>  x  =  y] 

x    y         —  '  — 

94.  V  V    [-x  >   -y  <=>    y  >  x] 

x    y  L 


[7-35] 
[7-35] 
[7-35] 
[7-35] 


95.    a.     VxVy  Vz>  Q  [xz  >  yz    <=>    x  >  y] 
^'     VxVy  Vz  <  0  fXZ  *  yZ    <=^>    x  >  yl 


[7-36] 


96      a.     V    V    [xy  >   0 
—         x    y  L    7 


( [x  >  0  and  y  >  0]  or  [x  <  0  and  y  <  0]  )  ] 


b.     V   V    [xy  <  0  <=>  ( [x  >  0  and  y  <  0]  or  [x  <  0  and  y  >  0] )  ] 
—         x    y  L    ' 

[7-36] 


97     a.     V 


x2  >  0 


±'  vx/0 

b.  VxVy  [x/*y=S>  x2  +  y2>    2xy] 

c,  V^_x+i>2 
—        x>  0  x  — 


[7-38] 
[7-39] 
[7-40] 


[7-151] 


,2    _   ,,2 


x  =  y] 


98  *•  Vx>0Vy>otX    -* 

b-  V   V   .    n  [y2  >  x2  =>  y  >  x  >   -y] 

—  x    y  >  0 

c-  V    ^  n  V    [y  >  x=>  y2  >  x2] 

—  x>  0     y     7  7  J 


*\ 


\     [page  7-38] 


-> 


99  a.     V   V   V    ,n   [-  >   £ 
—        x    y    z/£  0    L  z         z 


^  xz  >  yz] 


£"     Vx^0  ([i  >  0    <=>    x  >  0]  and  [~  <  0  <=>  x  <0]) 


[7-41] 


100. 


Vx>0Vy/0  Vz>0[y>   Z^    I>    ?J 


j-    vu    o„ 

'IN       «V       'lx 


[7-41] 


(V) 

<V> 


[domain  of  'm',    4n',    *p*,   and  4q'  is  I+] 

1  €l+ 

Vn  n  +  1  e  I+ 

Vg  [( 1  €  S  and  Vr  [n  €  S  =>  n  +  1  e  S] )  =>  Vn  n  €  S] 


[7-49] 


Oy        „^         xU 

*-j>.  ^,X  *•,>. 


101.     I+C  P 


102.     V     V    m  +  n  €  I* 
m    n 


103.     V     V    mneT 
m    n 


104.     V    n  >  1 
n       — 


105.     V     V    [n  >  m  =>  n  -  mc  I+] 
m    n  L  J 


[Vn  n  €  P] 


106.     V     V    [n  >  m  +  1 
m    n  l     — 


n  >   m] 


[7-49] 
[7-56] 
[7-56] 
[7-84] 
[7-84] 
[7-86] 


107  a.  V    n  f-   1 

—  n 

b.  V     V    [n  <   m  +  1    <=>    n  <   m] 

— ■  m    n  L                                        —       J 


[7-86] 


[7-152] 


108. 


Each  nonempty  set  of  positive  integers  has  a  least  member. 

[V    [0  /  S  C  I*=>  3m      s   Vn  e  s   m  <  n]  J  [page  7-88] 


O,        0'        -J, 

"ix    ^r    "ix 


(C)    V     3     n  >  x 


x      n 


(I)    Vx[xel 


(x  e  I+  or  x  =  0  or  -x  €  I+)  ] 


s.1,       s,«,       si, 

•V    'J%    'lx 


109.    V    [xe  I  <=>   3_  3    x 

x  m      n 


=  m  -  n  ] 


[domain  of  V,    *j\   and  'k'  is  I] 


110  a.     V.   -j  e  I 


b.     V.  Vkk  +  jel 

c .   y.  vk  k  -  j  €  i 

d.    V.VkkjCI 


[7-89] 
[7-94] 


[7-94] 


*\ 


>       [7-95] 


J 


111.     V.  Vk[k>  j 


k-j€l*] 


112.     V.  V.    [k+  1  >  j   <=>    k  >  j] 


[7-96] 
[7-96] 


113.  Each  nonempty  set  of  integers  which  has  a  lower  bound  [7-98] 
has  a  least  member. 

114.  V.  V     [(j  eS  and  Vfc  >  .   [k  e  S  =>  k  +  1  e  S] )  =>  Vk>  .  k  e  S]    [7-99] 

115.  Each  nonempty  set  of  integers  which  has  an  upper  bound      [7-100] 
has  a  greatest  member. 


116.     V.  Vg  [(j  €  S  and  Vk<  .  [k  €  S=>  k-  1  e  S})=>  VR<  .  k  e  S]      [7-100] 


117.     Vg[(0  e  S  and  Vk[k  €  S^=>  (k+  1  €  S  and  k  -  1  e  S)]) 


Ox     Ox      O. 

*,%  *,H.  «y. 


V    |[x  J  =  the  greatest  integer  k  such  that  k<  x 


Vx  «xj  =  x-[xj 


Ox      Ox      Ox 

<-,N         #-p  X-,% 


118  a.  V   V.    [k  <  [xj   <=>   k  <  x] 

•— •  X     K 

b.  VxVk[k>  IxJ   <=^   k>  x] 

c.  VxVk  [k  >  [xj   <=>   k  +  1  >  x] 

d.  V  V,    [k  <  ffxfl   <=>   k  +  1  <  x] 

— *  X     K 

e.  V   V.    [k  =  [xj   <=>    k  <  x  <  k  +  1] 

—  X     K 


[7-153] 

Vkkes]^ 

[page  7-100] 

[7-102] 
[7-107] 

[7-103] 
[7-104] 


"N 


>       [7-105] 


J 


119.     VxV.([x  + J]]  =  [xj+j 


120.     VV,ft3,3      [x  =  ky  +  z  and  0  <  z  <  y] 
xy>0kzL  7  —  Ji 


121.     V   V    .   A   3     ny  >  x 
x    y>  On7 


122.     V    -  j[  -xj   =  the  least  integer  k  such  that  k  >  x 


123.     V  V 
x    m 


x 

m 


M 


m 


and 


m 


124.     V 


'x*mo<S¥} 


m   =   [xj    - 


m 


x 

m 


-m 


m    ^    m 


Ox      Ox      Ox 
"ix     *V     T» 


V     V.   [m  I  j   <=>    3,    j  =  mk] 

m    j   l      tJ  k  J  J 


*'x      Ox      Ox 
V"     "<"*     *V 


125 


V    (1  In  and  n In) 


[7-105] 
[7-106] 
[7-106] 
[7-106] 

[7-111] 
[7-111] 

[7-115  and  7-129] 
[7-115] 


m 


[7-154] 


126   a. 
b. 

£. 

d. 
e. 
£. 


V  V    [mln^^  m  <  n] 
m    n  L      '  ~~     J 

V  V   V    [  (m  I  n  and  n  I  p)  ==>  m  I  p] 
m    n    p  l        '  ,r  irj 

V  V    [  ( m  I  n  and  n  I  m)  =>  m  =  n] 
m    n  lx      '  '     '  J 

VmVnVp  [(mln  and  mlp)  ==c>  mln  +  Pi 

VmVnVp  ^mln  and  mln  +  P)  ^  mlpl 
VmVnVp  [m|n=>  mp|np] 


"\ 


>        [page  7-115] 


y 


[7-116] 


127.     V     V    3.    3.    HCF(m,  n)  =  mi  +  nj 
m    n     l     j 


[7-122] 


128.     V     V   V.    [(HCF(m,  n)  =  1  and  mink) 
m    n    k  L  '  '      ' 


m 


Ik] 


[7-129] 


129.     V     V    [HCF(m,  n)  =  1 
m    n  L 


V.  V.    [mi  +  nj  =  0  <=>  3     (i  =  nk  and  j  =  -mk)]  ]       [7-129] 
1     J  K 


TABLE  OF  TRIGONOMETRIC  RATIOS 


[7-155] 


Angle 

sin 

cos 

tan 

Angle 

sin 

cos 

tan 

1° 

.0175 

.9998 

.0175 

46° 

.7193 

.6947 

1.  0355 

2° 

.0349 

.9994 

.0349 

47° 

.7314 

.6820 

1.0724 

3° 

.05  23 

.9986 

.05  24 

48° 

.7431 

.6691 

1.  1106 

4° 

.0698 

.9976 

.0699 

49° 

.7547 

.6561 

1.  1504 

5° 

.0872 

.9962 

.0875 

50° 

.7660 

.6428 

1.  1918 

6° 

.  1045 

.9945 

.  105  1 

51° 

.7771 

.6293 

1.2349 

7° 

.  1219 

.9925 

.  1228 

5  2° 

.7880 

.6157 

1.2799 

8° 

.  1392 

.9903 

.  1405 

53° 

.7986 

.6018 

1.  3270 

9° 

.  15  64 

.9877 

.  15  84 

54° 

.8090 

.5878 

1.3764 

10° 

.  1736 

.9848 

.  1763 

55° 

.  8192 

.5736 

1.4281 

11° 

.  1908 

.9816 

.  1944 

5  6° 

.8290 

.5592 

1.4826 

12° 

.2079 

.9781 

.2126 

57° 

.8387 

.5446 

1.5399 

13° 

.2250 

.9744 

.2309 

5  8° 

.  8480 

.5  299 

1.  6003 

14° 

.2419 

.9703 

.  2493 

59° 

.8572 

.5150 

i. 6643 

15° 

.25  88 

.9659 

.2679 

60° 

.8660 

.5000 

1.  7321 

16° 

.  2756 

.9613 

.2867 

61° 

.8746 

.4848 

1.8040 

17° 

.2924 

.95  63 

.305  7 

62° 

.8829 

.4695 

1.  8807 

18° 

.3090 

.9511 

.3249 

63° 

.8910 

.4540 

1.9626 

19° 

.3256 

.9455 

.3443 

64° 

.8988 

.4384 

2.  0503 

20° 

.3420 

.9397 

.3640 

65° 

.9063 

.4226 

2.  1445 

21° 

.3584 

.9336 

.3839 

66° 

.9135 

.4067 

2.  2460 

22° 

.3746 

.9272 

.4040 

67° 

.9205 

.3907 

2.3559 

23° 

.3907 

.9205 

.4245 

68° 

.9272 

.3746 

2.4751 

24° 

.4067 

.9135 

.445  2 

69° 

.9336 

.3584 

2.  605  1 

25° 

.4226 

.90  63 

.4663 

70° 

.9397 

.3420 

2.  7475 

26° 

.4384 

.8988 

.4877 

71° 

.9455 

.3256 

2.9042 

27° 

.4540 

.8910 

.5095 

72° 

.9511 

.3090 

3.0777 

28° 

.4695 

.8829 

.5317 

73° 

.9563 

.2924 

3.2709 

29° 

.4848 

.8746 

.5543 

74° 

.9613 

.275  6 

3.4874 

30° 

.5000 

.8660 

.5774 

75° 

.9659 

.2588 

3.7321 

3Je 

.5150 

.85  72 

.  6009 

76° 

.9703 

.2419 

4.  0108 

32° 

.5299 

.  8480 

.  6249 

77° 

.9744 

.2250 

4.3315 

33° 

.5446 

.8387 

.  6494 

78° 

.9781 

.2079 

4.  7046 

34° 

.5592 

.8290 

.  6745 

79° 

.9816 

.  1908 

5. 1446 

35° 

.5736 

.8192 

.7002 

80° 

.9848 

.1736 

5.6713 

36° 

.5878 

.8090 

.  7265 

81° 

.9877 

.  1564 

6.  3138 

37° 

.  6018 

.7986 

.7536 

82° 

.9903 

.1392 

7.  1154 

38° 

.6157 

.7880 

.7813 

83° 

.9925 

.  1219 

8. 1443 

39° 

.6293 

.  7771 

.8098 

84° 

.9945 

.1045 

9.5144 

40° 

.6428 

.7660 

.8391 

85' 

.9962 

.0872 

11.4301 

41° 

.65  61 

.7547 

.8693 

86° 

.9976 

.0698 

14.3007 

42° 

.6691 

.7431 

.9004 

87° 

.9986 

.0523 

19.0811 

43° 

.  6820 

.7314 

.93  25 

88° 

.9994 

.0349 

28. 6363 

44° 

.6947 

.7193 

.9657 

89° 

.9998 

.0175 

57.  2900 

45° 

.7071 

.7071 

1.0000 

[7-156] 


TABLE   OF  SQUARES  AND  SQUARE   ROOTS 


n 

1 
2 

3 
4 

5 
6 
7 
8 
9 

10 
11 
12 
13 
14 

15 
16 
17 
18 
19 

20 
21 
22 
23 
24 

25 
26 
27 
28 
29 

30 
31 
32 
33 
34 

35 
36 
37 
38 
39 

40 
41 
42 
43 
44 

45 
46 
47 
48 
49 


1 

4 

9 

16 

25 
36 
49 
64 
81 

100 
121 
144 
169 
196 

225 

256 
289 
324 
361 

400 
441 
484 
529 
576 

625 
676 
729 
784 
841 

900 

961 

1024 

1089 

1156 

1225 
1296 
1369 
1444 
1521 

1600 
1681 
1764 
1849 
1936 

2025 
2116 
2209 
2304 
2401 


1.000 
1.414 
1.732 
2.000 

2.  236 
2.449 
2.646 
2.828 
3.000 

3.  162 
3.317 

3.464 
3.606 
3.742 

3.873 
4.000 

4.  123 
4.243 
4.359 

4.472 
4.583 
4.690 
4.796 
4.899 

5.000 
5.099 

5.  196 
5.292 
5.385 

5.477 
5.5  68 

5.  657 
5.745 
5.831 

5.916 
6.000 
6.083 

6.  164 
6.  245 

6.325 
6.403 
6.481 
6.557 
6.  633 

6.708 
6.782 
6.856 
6.928 
7.000 


yflOn 

3.  162 
4.472 
5.477 
6.325 

7.071 
7.746 
8.367 
8.944 
9.487 

10.000 
10.488 
10.954 
11.402 
11.832 

12.247 
12. 649 
13.038 
13.416 
13.784 

14. 142 
14.491 
14.832 
15. 166 
15.492 

15.811 
16. 125 
16.432 
16.733 
17.029 

17.321 
17.607 
17.889 
18. 166 
18.439 

18. 708 
18.974 
19.235 
19.494 
19.748 

20.000 
20. 248 
20.494 
20.736 
20.976 

21.213 
21.448 
21.679 
21.909 
22. 136 


50       2500       7.071        22.361 


n 

51 
52 
53 

54 

55 
56 
57 
58 
59 

60 
61 
62 
63 
64 

65 
66 
67 
68 
69 

70 
71 
72 
73 
74 

75 
76 
77 
78 
79 

80 
81 
82 
83 
84 

85 
86 
87 
88 
89 

90 
91 
92 
93 
94 

95 
96 
97 
98 
99 


n 


2601 
2704 
2809 
2916 

3025 
3136 
3249 
3364 
3481 

3600 
3721 
3844 
3969 
4096 

4225 
435  6 
4489 
4624 
4761 

4900 
5041 
5184 
5329 
5476 

5625 
5776 
5929 
6084 
6241 

6400 
6561 
6724 
6889 
705  6 

7225 
7396 
75  69 
7744 
7921 

8100 
8281 
8464 
8649 
8836 

9025 
9216 
9409 
9604 
9801 


7.141 
7.211 

7.  280 
7.348 

7.416 
7.483 
7.550 
7.616 
7.681 

7.746 
7.810 
7.874 
7.937 
8.000 

8.062 

8.  124 
8.  185 
8.246 
8.307 

8.367 
8.426 
8.485 
8.544 
8.  602 

8.660 
8.718 
8.775 
8.832 
8.888 

8.944 
9.000 
9.055 
9.110 
9.165 

9.220 
9.274 
9.327 
9.381 
9.434 

9.487 
9.539 
9.592 
9.644 
9.695 

9.747 
9.798 
9.849 
9.899 
9.950 


VTOn 

22.583 
22.804 
23.022 
23.238 

23.452 
23.664 
23.875 
24.083 
24.290 

24.495 
24.698 
24.900 
25. 100 
25.298 

25.495 
25.690 
25.884 
26.077 
26.268 

26.458 
26.  646 
26.833 
27.019 
27.203 

27.386 
27.568 
27.749 
27.928 
28.  107 

28.284 
28.460 
28.636 
28.810 
28.983 

29. 155 
29.326 
29.496 
29.665 
29.833 

30.000 
30. 166 
30.332 
30.496 
30.659 

30.822 
30.984 
31.  145 
31.305 
31.464 


100  10000  10.000   31.623